NEB/PMDC STEP-1 PAST PAPERS
DR.ASIF ALI KHAN
YANGTZE UNIVERSITY
NEB/PMDC STEP-1 PAST PAPERS
DR.ASIF ALI KHAN
YANGTZE UNIVERSITY
02 JUNE 2016
MCQS OF NEB PMDC STEP-1
Community Medicine
1. Dental caries is due to lack of – Fluoride
2. Contain High Calories – Fat and Oil
3. Communication in School – Two way
4. Death rate declining, birth rate declining, stage? low expanding
5. Not included in Maternity Cycle – Weaning
6. Muscle stretching of factory worker, management – Ergonomics or Training
7. Not Included in PHC – Disequality
8. Not a prevention of Malaria – Vaccination
9. Polio schedule not included in EPI – 1 year
10. Pregnant Lady with anemia give specific protection – Secondary Protection/ Early diagnosis and Tx
11. 10 % of Population – Simple Cross sectional/ Stratified Cross Sectional
12. Topographic Accident – Avalances
13. Poor Overcrowded House – Rheumatic Heart Disease (Unemployment and overcrowding is associated with
RHD)
14. Best Prevention of Non-Communicable Disease – Health Education/Genetic Counseling
15. Not risk factor for infant – 3 kg weight/ prematurity...
16. marasmus
17. Best teaching method in school, two way communication
18. Which one is not at risk of malnutrition 3 kg baby weight? at birth
19. Death rate decline, late expending
20. Polio vaccine not include in EPI schedule, zero at birth
NEB/PMDC STEP-1 PAST PAPERS
DR.ASIF ALI KHAN
YANGTZE UNIVERSITY
8 JANUARY 2012
MCQS OF NEB PMDS STEP-1
1. In a hypertensive patient which of the following will least likely to reduce the hypertension
A. Calcium intake
B. Potassium intake
C. Moderation in alcohol
D. Weight loss
E. Increase in physical activity
2. An example of acute inflammation is
A. Pneumonia
B. Tuberculosis
C. Sarcoidosis
D. Encephalitis
3. In Pakistan aedes egypti mosquito transmit
A. Dengue fever
B. Yellow fever
C. Leishmaniasis
D. Japanese encephalitis
4. Which of the following is the true regarding BCG vaccination:
A. BCG is not fully protective in adults and it is a live attenuated vaccine
B. It is protective in children’s
C. It is a live attenuated vaccine
D. Produce passive immunity
5. For the para follicular cells in thyroid and the parathyroid cells, best explanation is
A. Calcitonin and PTH
6. The hormone that controls the level of calcium and phosphorus in blood is secreted by
A. Thyroid gland
B. Parathyroid gland
C. Pituitary gland
D. Thymus
KEY: B, Parathyroid gland is tiny glands. It controls the body’s calcium level. Each gland is about the size of a
grain of rice.
7. In a conflict between two equally competence medical witness the court will fallow
A. The witness who seen the injured in the beginning
B. The witness who favor in defense
C. The witness who seen the injured in the end
8. If the vertical length of a window will increase what will happen in the light / shadows coming through it?
NEB/PMDC STEP-1 PAST PAPERS
DR.ASIF ALI KHAN
YANGTZE UNIVERSITY
9. The privileges of a register medical practioner is
A. can advertise in news paper
B. can voluntarily appear in court
10. In younger children’s tetracycline is not used because
A. It is hepatotoxic
B. Cause bone calcification
11. In a child with bloody diarrhea which of the following organism will be in the stools found in stools lab
exam.
A. E-coli
B. Shigella
C. Campylobacter
12. In the adrenal gland mineralocorticoids comes from
A. Zona fasciculate
B. Adrenal cortex
C. Zona glomerulosa
D. Adrenal medulla
E. Zona Reticularis
13. The size of particle to cause pneumoconiosis
A. 0.5 to 3 micron
B. 4 to 7 micron
C. 10 to 12 micron
14. A patient with Bradykinesia, this is described as
A. Cerebellar lesion
B. Levy bodies
C. Parkinsonism
D. Substancia nigra
15. An abnormality in coordination movement is termed as
A. Ataxia
B. Cerebellar lesion
C. Dysdiadochokinesia
16. Long term memories stored in which part of the brain
A. Thalamus
B. Cerebellum
C. Cerebral cortex
D. Hippocampus
17. Chop wound in?
A. Revolver
B. Short gun
NEB/PMDC STEP-1 PAST PAPERS
DR.ASIF ALI KHAN
YANGTZE UNIVERSITY
18. Cookie cutter in fire arm injury?
» Also known as: Rat hole injury / Cookie cutter phenomenon/ Moth eaten appearance of wound of entry
» In case of shot gun fire, up till 1 meter (3feet) all the pellets make a single large hole.
» After 3 feet the edges of the wound will become crenated.
» From 2 meters upwards, the number of satellite pellet holes will progressively increase around the main
wound.
19. Warm antibody in hemolytic anemia – IgG type of antibodies
20. Zona glomerulosa secrete – mineralocorticoid (aldosterone)
21. Erythropoietin is secreted from – peritubular capillary (interstitial fibroblasts in the kidney)
22. Under the Qisas and Diyat Act, injuries present on the neck are classified as:
A. Jaifa Hurts
B. Other hurts falling U/S 337 L II.
C. Shajjah
D. Jurh.
E. Damiyah
KEY: E
23. Swollen bleeding gums in???
A. Scurvy
B. Vitamin d deficiency
24. BP 120/80 mean arterial pressure (MAP) is – 93mmHg
» MAP = Diastolic pressure + 1/3 pulse pressure
» At 120/80mmHg MAP = 80mmHg + 1/3 of pulse pressure (40mmHg) = 93mmHg
25. Complication of successful treatment of thrombophlebitis
A. rupture
B. Infection
C. thrombus formation
KEY: C
26. Incubation period of HCV
A. 20 days
B. 50 days
C. 30 days
D. 10 days
KEY: B
27. Derivative of 4th pharyngeal arch – external laryngeal nerve
28. Muscle used in walking – Tabialis anterior
NEB/PMDC STEP-1 PAST PAPERS
DR.ASIF ALI KHAN
YANGTZE UNIVERSITY
29. Enlargement of the prostate cells usually arise from
A. periurethral area
B. peripheral area
C. transitional zone
30. A patient with only HBsAg positive will be called carrier if
A. HBsAg positive for more than 6 months
31. If a person’s BP is 120/90, the mean arterial pressure will be
A. 30
B. 20
C. 120
D. 90
KEY: D
32. In deafness, garment is inserted in the ear in
A. conductive type of deafness
B. Sensorineural type
C. Presbycuses
33. ECG shows no P wave, where is the pace maker
A. Ventricle
B. AV node
C. Atrium
D. Purknje fibers
E. Bundle of his
34. Counselling is a technique which aims at :
A. Making people less emotional.
B. Achieving a greater depth of understanding and clarification of the problem
C. Comparing the patient’s experiences with one’s own.
D. Giving sincere advice and solutions to the patients problems
E. Breaking bad news in a professional manner.
KEY: B
35. Patient may present with lucid interval in
A. Subdural hematoma
B. Extradural haematoma
C. Sub arachnoid hemorrhage
KEY: B
36. A highly infective but less pathogenic state is found in
A. Sporadic outbreak
B. Epidemic outbreak
C. Carrier
NEB/PMDC STEP-1 PAST PAPERS
DR.ASIF ALI KHAN
YANGTZE UNIVERSITY
37. Which of the following increase plasma level of phenytoin
A. Penicillin
B. Carbamazepine
C. Cimetidine
KEY: B
» Phenytoin metabolism is decreased by: Cimetidine, isoniazid, chloramphenicol, Dicumarol, Sulfonamide
» Phenytoin metabolism is increased by Carbamazepine
38. Which of the following states have least metastatic spread?
A. Squamous cell carcinoma
B. Basal cell carcinoma
C. Melanoma
39. Case of kidney transplant, Patient on immunosuppressive therapy, he has severe headache and some
other signs the organism was isolated they were branched and were stained with Indian Ink.
A. Pneumocystis carniii
B. Pneumocystis jervoci
C. Cryptococcus neoformans
D. Candida
40. Causative agent of urinary bladder Cancer
A. Schistosoma Japonicum
B. Schistosoma Mansoni
C. Schistosoma Haematobium
KEY: C
41. Patient had luminescent stools and vomit is due to
A. Arsenic poisoning
B. Copper poisoning
C. Phosphorus poisoning
KEY: C
42. Activated charcoal is used in following poisoning
A. Metals
B. Alkalis
C. Acid bases
D. Lithium
KEY: A
NEB/PMDC STEP-1 PAST PAPERS
DR.ASIF ALI KHAN
YANGTZE UNIVERSITY
43. Which of the product is recycled in urea cycle after urea is made?
A. Citrulline
B. Ornithine
C. Arginosuccinate
KEY: C
44. Arsenate poisoning blocks which step in TCA cycle
A. Isocitrate dehydrogenase
B. Alpha keto glutarate dehydrogenase
C. Succinyl dehydrogenase
KEY: B, Arsenate poisoning inhibit pyruvate dehydrogenase, alpha keto glutarate dehydrogenase
45. Epo-ophron and para-ophron are remnants of
A. Mullerian duct
B. Mesonephros
C. Metanephros
KEY: B
46. Which of the following medication for GI problems is contraindicated in pregnancy?
A. Calcium carbonate
B. Famotidine
C. Lanzoprazole
D. Misoprostol
KEY: D, Contraindicated in pregnancy because it may stimulate uterine contractions.
47. An elderly woman with recent hx of myocardial infarction is seeking medication to help treat occasional
heartburn. She is currently taking several meds, including aspirin, Clopidogrel, simvastatin, metoprolol,
and Lisinopril. Which of the following choices should be avoided in this patient?
A. Calcium citrate
B. Omeprazole
C. Ranitidine
D. Calcium carbonate
KEY: B, May possibly decrease the efficacy of clopidogrel because it inhibits the conversion of clopidogrel to
its active form
NEB/PMDC STEP-1 PAST PAPERS
DR.ASIF ALI KHAN
YANGTZE UNIVERSITY
19 SEPTEMBER 2015 UHS
MCQS OF NEB PMDC STEP-l
Physiology
1. Increase Cardiac output — cardiac reservoir
2. For renal function endogenous substance is — creatinine
3. For increasing concentration of local anesthesia — epinephrine
4. watery diarrhea gram negative increase — CAMP
5. Immune regulator are suppress — T cells
6. CSF protein — 350gm
7. MI ECG with one year history — Q wave
8. High K lead to release of — Aldosterone
9. Albumin — negative charge
10. Absent A wave — atrial fibrillation
11. 28 litters out of 42 store inside — cells of body
12. After 4th days chest pain increase with inspiration is — Fibrinoid pericarditis
Biochemistry
1. Best for heart is — Hdl:LDL is 3:1
2. Enzyme of HMP shunt for hemolysis is — glutamate oxidase
3. Main regulator enzyme for Krebs cycle is – alpha ketoglutrate
4. Diabetes mellitus— glycosylate hb
5. Serotonin — 10 min
6. Enzyme competitive inhibitors – increase km
7. During fasting blood glucose level is main by – glycogenolysis
8. Darkness growth —Vit A
9. Cholelithiasis — vitamin E
10. Which among the following is a cardioprotective fatty acid – Omega-3 fatty acids
» Oleic acid is a monounsaturated fatty acid – cardioprotective, reduce LDL.
» Omega-3 fatty acids like linolenic acid and docosahexaenoic acid are cardioprotective.
NEB/PMDC STEP-1 PAST PAPERS
DR.ASIF ALI KHAN
YANGTZE UNIVERSITY
Pharmacology
1. Drug along with nitrates to overcome oxygen demand is—metoprolol
2. Drug for aspirin induce asthma— zafirleukast
3. Drug for MRSA — Vancomycin
4. Drug for mild Amebiasis is — metronidazole
5. Anti-tubercular drug in pregnancy which lead to deafness is — streptomycin
» Streptomycin is contraindicated in pregnant women because of fetal ototoxicity.
6. Drug of choice for diabetic nephropathy is — captopril
7. Digoxin toxicity ECG – increase PR interval
8. Drug which is not metabolize in liver— diazepam
9. Case scenario and ask side effect of Ethambutol – Optic neuritis
10. Side effect of insulin is – hypoglycemia
11. Most potent Fluoroquinolones against Mycobacterium leprae is – Moxifloxacin
12. Drug which cause discoloration of teeth in the fetus – Tetracycline
13. Drug which inhibits insulin secretion and causes hyperglycemia – Phenytoin
14. Drug of choice in acute pulmonary edema due to left ventricular failure – Furosemide
» Frusemide is a loop diuretic. It is also a pulmonary venodilator. In left ventricular failure, frusemide is
given intravenously. It causes pulmonary venodilation and relieves pulmonary congestion. Also by
increasing urine formation, it decreases blood volume and venous return thus decreasing preload of the
heart.
Pathology
1. Necrosis in granuloma of TB — Caseous necrosis
2. Long term Hypertension lead to— left ventricular hypertrophy
3. Diagnostic investigation for thalassemia is – Hb electrophoresis
4. Movements disorders plus dementia — Huntington disease
5. Ankyrin spectrin deficiency — hereditary spherocytosis
6. Collapsing pulse with diastolic murmur is aortic regurgitation
7. Case scenario of a worker who had been in the battery manufacturing unit for the last 23 years, reported
to you with complaints of lost appetite and abdominal colic of 2 weeks duration. You would prefer to
investigate him for:
A. Cholecystitis
B. Lead poisoning
C. Appendicitis
D. Ameobiasis
E. Stomach cancer
KEY: B
NEB/PMDC STEP-1 PAST PAPERS
DR.ASIF ALI KHAN
YANGTZE UNIVERSITY
8. Epithelium of endocervix is mucus secreting — simple columnar epithelium
9. Pyloric gland cause bicarbonate secretion along with — mucus
10. Wound —tetanus
11. Vertical transmission — HIV/AIDS
12. Crohn disease— non caseating granuloma
13. Case scenario of patient with Gastroectomy, causes deficiency of —B12
14. ALT and AST arise with IgM anti HAV—fulminant hepatitis
15. Pigmentsion with low Na and ACTH is — Addison’s disease
16. Antidote for methanol is — Ethanol
17. Pinpoint purpura with low platelets high bleeding time and clotting time is Thrombotic thrombocytopenia
18. Electroencephalogram (EEG) petit mal seizures with – 3-Hz spike-and-wave discharges
19. Patient with centripetal obesity, acne and hairsutism – Cushing’s syndrome
20. Endotoxin—TNF
Forensic medicine
1. Patient negligence is contributory
2. Most common cause of death in case of dental anesthesia is – Chocking
3. Exhumation done under order by
A. Police officer
B. Superintendent police
C. First class magistrate
KEY: C
4. Earliest sign of death is
A. loss of skin elasticity
B. Corneal clouding
C. Cooling of body
D. Postmortem lividity
5. Postmortem caloricity seen in poisoning from
A. Arsenic
B. Strychnine
C. Cyanide
D. Organophosphorus
KEY: B
NEB/PMDC STEP-1 PAST PAPERS
DR.ASIF ALI KHAN
YANGTZE UNIVERSITY
6. Blunt abdominal trauma commonest site of GI ruptures is
A. Stomach
B. Duodenum
C. Jejunum
D. Transverse colon
KEY: D
7. Characters of smooth bored firearm are:
A. It is generally a long barreled firearm
B. The inside of barrel is marked by grooves
C. It fires spinning missiles
D. It usually fires single missile
KEY: A
8. Medico legal system practiced in Pakistan:
A. Coroner's system
B. Continental system
C. Modified continental system
D. Judicial / police inquest
KEY: D
Anatomy
1. Only lateral ventricular enlargement – Intraventricular stenosis
2. Stab wound in axilla with lose of supination and flexion at elbow joint is – musculocutanoeus nerve
3. Fracture of shaft of humerus is — radial nerve
4. Fibular neck fracture —common peroneal nerve
5. Titling or positive Trendelenberg sign — gluteus maximum and minimus
6. Hit on lateral side of head artery damage is – frontal branch of meningeal artery (also called anterior branch
7. Appendicular artery is — posterior cecal artery
8. Deep venous thrombus – one of pulmonary arteries
9. Babinski dorsiflexion of greater toe plus fainting of other toe
10. Thrombus—greater saphenous vein
11. Lateral meniscus – hearing
12. Facial nerve palsy
13. Lymph from medial inferior side drained into – parasternal lymph node
14. structure behind the urinary bladder of male is – seminal vesicle
15. Lateral part of posterior wall of inguinal canal is formed by – transversalis
NEB/PMDC STEP-1 PAST PAPERS
DR.ASIF ALI KHAN
YANGTZE UNIVERSITY
16. Loss of index finger flexion and loss of sensation on palmer aspect – Median nerve
17. Difficulty rising from standing position damage to – gluteus Maximus muscle
18. Right suprarenal vein drains in to – IVC
19. Laterally rotated thumb by which nerve – radial nerve
20. Thyroid is enclosed in which fascia – Pre tracheal
Microbiology
1. In case of TB CSF — low glucose with lymphocytosis
2. Bloody diarrhoea with causative organism in mucosa – C. difficile
3. Yellow sputum with fever and on x ray infiltration abd neutrophilia is — S.aureus
4. Herpes simplex virus transmitted by – droplet
5. Currant jelly sputum – klebsiella
6. Pear shape eggs and causing diarrhoea – Giardia
7. Farmer with migratory lesion on foot — Cutaneous Larva migrans
8. Definitive diagnosis of TB is made through — sputum AFB
Community medicine
1. A study was conducted to investigate the effect of HIV infection on mortality among people in Kenya with
TB. Individuals with TB were recruited from hospitals and their HIV status determined. They were then
followed-up over ten years to compare mortality rates in the HIV positive group and HIV negative group.
A. Case-control study
B. Cohort study
C. Randomized controlled trial
D. Cross-sectional study
KEY: B
2. The non-clinical Vitamin A deficiency is more common as compared to the clinical, threatening the health
of as many as one third of the world’s children. The best proxy indicator of this is:
A. Infant mortality rate
B. Maternal mortality rate
C. Literacy rate
D. Proportion of the rural population
KEY: A
NEB/PMDC STEP-1 PAST PAPERS
DR.ASIF ALI KHAN
YANGTZE UNIVERSITY
3. Which of the following is caused due to extreme lack of proteins?
A. Malaria
B. Typhoid
C. Kwashiorkor
D. None of these
KEY: C
4. One gram of protein contains
A. 4.3 kilocalories of energy
B. 4.2 kilocalories of energy
C. 5.2 kilocalories of energy
D. 5.4 kilocalories of energy
KEY: B
5. Community mean deviation
6. To limit the movements of patient – Quarantine
16 March 2017 (NUMS)
MCQS OF NEB PMDC STEP-l
Anatomy
1. A 70-year-old man has difficulty rising from a seated position and straightening his back, but he has no
difficulty flexing his hip and knee. Which one of the following muscles is most likely to be weak?
A. Hamstrings
B. Ileo-psoas
C. Obturator internus
D. Gluteus maximus
E. Gluteus minimus
KEY: D
2. What exits from the Stylomastoid foramen
A. Chorda tympanic nerve
B. Accessory nerve
C. Facial nerve
D. Glossopharyngeal nerve
E. Hypoglossal nerve
KEY: C
NEB/PMDC STEP-1 PAST PAPERS
DR.ASIF ALI KHAN
YANGTZE UNIVERSITY
3. Temporal lobe – Middle Cranial Fossa
4. Vagus nerve injury – least likely effect Sigmoid colon
5. Middle Nasal concha – Ethmoid bone
6. Right ear near shoulder – Sternocleidomastoid
7. Wernicke’s area – Middle cerebral artery
8. Main support of Uterus – Urogenital diaphragm??
9. Vein used in graft — Greater Saphenous vein
10. If nystagmus is a prominent symptoms of a cerebellar lesion, the lesion within:
A. The dentate nucleus
B. The flocculonodular lobe
C. The lateral cerebellum
D. The cerebrocerebeller cortex
E. The superior cerebellar peduncle
KEY: B
11. The oesophageal opening in the diaphragm is at
A. T6
B. T8
C. T10
D. T12
E. L4
KEY: C
12. The isthmus of thyroid gland – lies anterior to the 2nd
, 3rd
, and 4th
Tracheal rings
A. Lies directly anterior to the thyroid cartilage
B. Lies directly anterior to the cricoid cartilage
C. Lies directly anterior to the 2nd, 3rd, and 4th Tracheal rings
D. Is the same as the pyramidal lobe
E. Contains the superior pair of parathyroid glands
KEY: C
13. The structure passing through the aortic opening of diaphragm
A. Accessory nerve
B. Esophagus
C. Superior vena cava
D. Azygous Vein
E. L-R laryngeal
KEY: D, Azygous Vein
NEB/PMDC STEP-1 PAST PAPERS
DR.ASIF ALI KHAN
YANGTZE UNIVERSITY
14. Tennis player – Extensor Carpi Radialis
15. Volkmann canal— oblique
16. Myocardial infarction — Left anterior descending artery (LAD)
» Most commonly involved coronary artery in myocardial infarction (MI) is left anterior descending artery.
17. Laterally displaced Tibia over Femur – Medial Collateral Ligament
18. Osteosarcoma – Knee joint
19. Suspensory ligament of axilla is derived from
A. Clavipectoral fascia
B. Deltoid
C. Axillary sheath
D. Platysma
E. Pectoralis major
KEY: A
20. The fetal period extends from:
A. 12th week to term
B. 3rd week to 8th week
C. 4th week to term
D. 5th
week to term
E. 9th week to term
KEY: E
21. A 15 years old boy is brought to emergency after a fall from his bicycle x-ray reveals fracture of neck of
fibula. On physical examination the patient a foot drop. Which of the following nerves is most likely
injured
A. Tibial
B. Common fibular (peroneal) nerve
C. Superficial fibular (peroneal) nerve
D. Saphenous
E. Deep fibular (peroneal) nerve
KEY: B
22. A newborn presented with dribbling of urine from the umbilicus, he was diagnosed of having urachal
fistula which is a remnant of
A. Allantois
B. Urogenital sinus
C. Urorectal septum
D. Cloaca
E. Mesonephric duct
KEY: A
NEB/PMDC STEP-1 PAST PAPERS
DR.ASIF ALI KHAN
YANGTZE UNIVERSITY
23. A development of thyroid gland is marked by a depression called foramen cecum in the floor of the
primitive pharynx. The foramen caecum gives rise to a diverticulum which later elongates and descends
anterior to the hyoid bone and thyroid cartilage to assume the adult position. What is this diverticulum
called?
A. Hypoglossal duct
B. Laryngeal diverticulum
C. Respiratory diverticulum
D. Thyroglossal duct
E. Thyroglossal diverticulum
KEY: D
24. A 60-year-old male executive who had a history of a chronic duodenal ulcer was admitted to the ER
exhibiting signs of a severe internal hemorrhage. He was quickly diagnosed with perforation of the
posterior wall of the first part of the duodenum and erosion of an artery behind it by the gastric expellant.
The artery is most likely the:
A. Common hepatic
B. Gastroduodenal
C. Left gastric
D. Proper hepatic
E. Superior mesenteric
KEY: C, Gastroduodenal
25. A 43 year old women came with large abscess in the middle of right post triangle of the neck. The
physician incised and drained the abscess. Five days later patient noticed that she could not extend her
right hand above her head to brush her hair. Which of the following structure is damaged
A. Damage to scaleneus Medius
B. Spread of infection to shoulder joint
C. Spinal accessory nerve injury
D. Thoracodorsal nerve injury
E. Suprascapular nerve injury
KEY: C, Spinal accessory nerve injury
26. A women was diagnosed as having a cervical carcinoma. For exfoliative cytology cervical cells were taken
from the junction of endocervix with ectocervix. Here the epithelium changes from simple columnar to:
A. Stratified columnar
B. Stratified cuboidal
C. Stratified squamous keratinizing
D. Stratified squamous non-keratinizing?
E. Simple squamous epithelium
KEY: D
NEB/PMDC STEP-1 PAST PAPERS
DR.ASIF ALI KHAN
YANGTZE UNIVERSITY
Biochemistry
1. A 10 years old was evaluated for burning sensation in his feet and clusters of small red purple spots on his
skin. Lab studies reveal protein in his urine. Enzyme analysis revealed a deficiency of alpha galactosidase
and enzyme replacement therapy was recommended. The most likely diagnose is
A. Fabry disease
B. Farber disease
C. Gaucher disease
D. Krabbe disease
E. Niemann-Pick disease
KEY: A. Fabry disease
2. A 2 years child with severe fasting hypoglycemia, hyperlacticedemia and hyperuricemia. Biopsy reveal
hepatocytes with increased amount of glycogen – Von Griek disease (G6P deficiency)
3. Enzyme which is deficient or absent in lens, causing cataract in diabetes – Sorbitol dehydrogenase
deficiency
4. Sorbitol accumulation
5. 37 years old male with signs and symptoms of hyperglycemia, diagnose is Diabetes Mellitus type-2 due to
insulin resistance, lab show hypertriglyceridemia, hyperlipidemia – very low density lipoprotein (VLDL)
6. Which of the following lipoprotein particles are most likely responsible for the appearance of the patient's
plasma?
A. Chylomicrons
B. HDL
C. IDL
D. LDL
E. VLDL
KEY: A, Chylomicrons
7. A term infant is born and does well with breast feeding. Two days later his mother calls frantically
because the baby is bleeding from umbilicus as nostrils. The following vitamin is most probably deficient
in this infants:
A. Vitamin A
B. Vitamin D2
C. Vitamin D3
D. Vitamin E
E. Vitamin K
KEY: E
NEB/PMDC STEP-1 PAST PAPERS
DR.ASIF ALI KHAN
YANGTZE UNIVERSITY
8. Corn is continuously low in water soluble vitamin. In 1936, the corn meal was fortified with the vitamins,
and the consequently large numbers of the insane asylum inmates in South Carolina (where corn is the
staple diet were miraculously cured of mental illness. The vitamin that cured their pellagra and their
mental illness was
A. Folic acid
B. Niacin
C. Pyridoxine
D. Riboflavin
E. Thiamine
KEY: B
9. Lactic acidosis – Thiamine
10. Semi-essential Amino Acid – Arginine
11. Pregnant with Cystic Acne – Vitamin A
12. Case scenario of Albinism is due to deficiency of the enzyme:
A. Phenylalanine
B. Tyrosinase
C. P-Hydroxyphenylpyruvic acid oxidase
D. Tyrosine dehydrogenase
KEY: B
13. RNA does not contain
A. Thymine
B. Adenine
C. Guanine
D. Cytosine
KEY: A, Thymine is present in DNA but not in RNA. Both DNA and RNA are composed of a mix of four bases.
However, only three of the four bases are common to both nucleic acids - adenine, guanine, and cytosine. In
DNA, the fourth base is thymine; in RNA, thymine is replaced by uracil.
14. In a new born presenting with refusal to feeds and irritability, a deficiency of Cystathionine –β- synthase
has been diagnosed, which of the following compounds is expected to be elevated in blood?
A. Serine
B. Glutamate
C. Homocysteine
D. Valine
KEY: C
NEB/PMDC STEP-1 PAST PAPERS
DR.ASIF ALI KHAN
YANGTZE UNIVERSITY
15. A 3 months old girl is developing cataracts. Other than having social smile or being able to track objects
visually, all other aspects of the girl examination are normal. Test on baby’s urine are positive for reducing
sugar but negative for glucose. Which enzyme is most likely deficient?
A. Aldolase A
B. Aldolase B
C. Fructokinase
D. Galactokinase
E. Galactose-1 P uridyltransferase
KEY: D, The girl is deficient in galactokinase and is unable to appropriately phosphorylate galactose.
Galactose accumulates in the blood (and urine). In the lens of the eye, galactose is reduced by aldose
reductase to galactitol, a sugar alcohol, which causes osmotic effects that result in cataract formation.
Deficiency of galactose 1-phosphate uridylyltransferase also results in cataracts but is characterized by liver
damage and neurologic effects. Fructokinase deficiency is a benign condition. Aldolase B deficiency is
severe, with effects on several tissues. Cataracts are not typically seen.
16. Meat restriction for 5-6 years + Anemia – Vitamin B12 deficiency
17. A 76 years old woman who is bed bound in a nursing home begins to develop swelling of her left leg. She
is evaluated with venous Doppler ultrasound and is found a deep vein thrombosis. She is immediately
started on heparin to further prevent the clot from enlargement. Heparin is an example of a
A. Sphingolipid
B. Cerebroside
C. Ganglioside
D. Glycoseaminoglycan
E. Prostaglandin
18. Urea cycle and citric acid cycle are linked by
A. Glutamate
B. Fumarate
C. Oxaloacetate
D. Ketoglutarate
E. Argininosucciate
19. Hyperammonemia lead to brain damage due to depletion of amphibo intermediate
A. Oxaloacetate
B. Fumarate
C. Malate
D. Citrate
E. Ketoglutarate
NEB/PMDC STEP-1 PAST PAPERS
DR.ASIF ALI KHAN
YANGTZE UNIVERSITY
20. A 55 year old male significantly reduced reabsorption of bile salts resulting from the action of bile salt-
binding resins. This expected to lead to a significant decrease in which of the following steroid compound
A. Aldosterone
B. Cholesterol
C. Cortisole
D. Pregnenolone
E. Progesterone
21. A teenager, concerned about his weight, attempts to maintain a fat-free diet for a period of several
weeks. If his ability to synthesize various lipids were examined, he would be found to be most deficient in
his ability to synthesize:
A. cholesterol
B. glycolipids
C. phospholipids
D. prostaglandins
E. triacylglycerol
KEY: D, Prostaglandins
22. Patient with H/O recurring attacks of pancreatitis, eruptive xanthomas and increased plasma triglycerides
level (Hypertriglyceridemia) associated with chylomicrons
A. Apolipoprotein E deficiency
B. Lipoprotein lipase deficiency
C. …
D. …
E. …
KEY: B
23. Two enzymes have similar V max, but enzyme 1 has lower Km as compared to enzyme 2, this means:
A. Less substrate is required by enzyme 1 to attain v max/2
B. Enzyme 1 has more active sites
C. Enzyme 2 has lesser number of regulatory sites
D. Enzyme 2 has lesser number binding sites
E. Enzyme 2 is slowed down by substrate
KEY: A
NEB/PMDC STEP-1 PAST PAPERS
DR.ASIF ALI KHAN
YANGTZE UNIVERSITY
Pharmacology
1. A 60 years old man had a myocardial infarction, which of the following should be used to prevent life-
threatening arrhythmias that can occur post-myocardial infarction in this patient
A. Digoxin
B. Flecainide
C. Procainamide
D. Quinidine
E. Metoprolol
KEY: E
Key: B - B-Blockers, such as metoprolol prevent cardiac arrhythmias that occur subsequent to a myocardial
infarction. None of the other drugs has been shown to be particularly effective in preventing post infract
arrhythmias
2. Chills, fever, and muscle aches are common reactions to which one of the following antiviral drugs?
A. Acyclovir
B. Gancyclovir
C. Oseltamivir
D. Interferon
E. Ribavirin
KEY: D
3. A 57-year-old man is being treated for an atrial arrhythmia. He complains of headache, dizziness, and
tinnitus. Which one of the following antiarrhythmic drugs is the most likely cause?
A. Amiodarone
B. Procainamide
C. Propranolol
D. Quinidine
E. Verapamil
KEY: D - The clustered symptoms of headache, dizziness, and tinnitus are characteristic of cinchonism,
4. We have a patient with essential hypertension and unusually high circulating catecholamine level. Our
goal is to block α- and β-adrenergic receptors using just one drug. Which of the following is capable to do
that
A. Labetalol
B. Metoprolol
C. Nadolol
D. Pindolol
E. Timolol
KEY: A - Labetalol (Alpha + Beta blocker)
NEB/PMDC STEP-1 PAST PAPERS
DR.ASIF ALI KHAN
YANGTZE UNIVERSITY
5. Which of the following agent(s) are associated with gingival hyperplasia?
A. Beta blockers
B. ACE inhibitors
C. Phenytoin
D. Statins
E. Carbamazepine
KEY: C, Phenytoin
6. A 16 years old boy presents to the ambulatory care clinic with persistent dry cough. He had cleaned out a
barn attic that was full of bats about 2 week ago and has had the cough for 5 days. The physician suspects
histoplasmosis, which would be the best treatment for this patient.
A. Albendazole
B. Fluconazole
C. Ciprofloxacin
D. Ribavirin
E. Penicillin-G
KEY: B - Fluconazole
Histoplasmosis is a fungal infection caused by the dimorphic fungus Histoplasma capsulatum. Although it is
found in the soil of the entire Ohio River basin, spores appear to be more concentrated in bird and bat
droppings. This patient likely stirred up a large number of spores while cleaning an area frequented by bats,
enough spores to cause an infection. A common treatment for less serious infections of histoplasmosis is
ketoconazole or fluconazole, Amphotericin B for more serious systemic infections.
MCQ copied from Lippincott's Illustrated Q&A Review of Pharmacology: page 206, chapter.6, MCQ #40
7. A 32 year old male presents to an outpatient clinic with a 5-day history of productive cough, purulent
sputum, and shortness of breath. He is diagnosed with community Acquired pneumonia (CAP). It is noted
that this patient has a severe Ampicillin allergy (anaphylaxis). Which of the following would be an
acceptable treatment for this patient
A. Levofloxacin
B. Ciprofloxacin
C. Penicillin-V
D. Nitrofurantoin
KEY: A – Streptococcus pneumoniae is a common cause of CAP, and the respiratory fluoroquinolones
levofloxacin and Moxifloxacin provide good coverage. Ciprofloxacin does not cover S. pneumoniae well and
is a poor choice for treatment of CAP. Penicillin would be a poor choice due to allergy. Nitrofurantoin has no
clinical utility for respiratory tract infections.
NEB/PMDC STEP-1 PAST PAPERS
DR.ASIF ALI KHAN
YANGTZE UNIVERSITY
8. A 40 years old woman on chemotherapy for ovarian carcinoma, developed hemorrhagic Cystitis. Which of
the following anticancer drugs may be responsible for it?
A. Methotrexate
B. Cyclophosphamide
C. 5-Fluorouracil
D. Cytarabine
E. Vinblastine
KEY: B
9. Drug of choice for Cisplatin induced emesis is
A. Metoclopramide
B. Domperidone
C. Ondansetron
D. Octreotide
KEY: C
10. Which of the following is euglycemic drug
A. Chlorpropmide
B. Metformin
C. Glyburide
D. Insulin
E. Rapaglinide
KEY: B
11. Carbidopa is used in the treatment of Parkinson’s disease because it
A. Is a precursor of L-dopa
B. Dopaminergic receptor agonist
C. Prevents peripheral biotransformation of L-dopa
D. Prevents break down of dopamine
E. Prevents in decreased concentration of L-dopa in the nagrostratium
KEY: C
12. Leukotriene inhibitor – Zafirleukast
13. Cinchonism – Quinidine
NEB/PMDC STEP-1 PAST PAPERS
DR.ASIF ALI KHAN
YANGTZE UNIVERSITY
14. The macrolide antibiotics act by binding with
A. 30-S ribosomal subunit
B. 50-S ribosomal subunit
C. 60-S ribosomal subunit
D. 70-S ribosomal subunit
E. 90-S ribosomal subunit
KEY: B
15. Antihypnozoite – Primaquine
16. A 28 year old diabetic pregnant should be treated with
A. Sulphonyl urea
B. Biguanides
C. Insulin
D. Setagliptin
17. Benzodiazepine – Clonazepam
18. Insulin – Hypoglycemia
19. T4 conversion to T3 – Propylthiouracil (PTU)
20. Isoniazid – Pyridoxine
21. Taxi driver signs symptoms of allergy – Fexofenadine
22. Carbidopa – Inhibit L-dopa in periphery
23. Which of the following steroids can be administered for maturity of surfactant in preterm labour.
A. Beclomethasone
B. Betamethasone
C. Prednisolone
D. Hydrocortisone
KEY: B
24. Which of the following agents is useful in treatment of malignant hyperthermia
A. Baclofen
B. Diazepam
C. Dantrolene
D. Tizanidine
E. Tubocurarine
KEY: C
25. Which of the following insulin preparation has the longest duration of action
A. Insulin Detemir
B. Insulin aspart
C. NPH insulin
D. Glulisine
KEY: A
NEB/PMDC STEP-1 PAST PAPERS
DR.ASIF ALI KHAN
YANGTZE UNIVERSITY
26. A 12 year old girl with a childhood history of asthma complained of cough, dyspnea, and wheezing after
visiting a riding stable. Her symptoms became so severe that her parents brought her to the emergency
room. Which of the following is the most appropriate drug to rapidly reverse her bronchoconstriction
A. Inhaled fluticasone
B. Inhaled Beclomethasone
C. Intravenous propranolol
D. Inhaled albuterol
E. Oral theophylline
KEY: D –Inhaled albuterol - inhalation of a rapid-acting B2 agonist, such as albuterol, usually provides
immediate bronchodilation.
27. Pinpoint pupil – naloxone
28. Antifungal drug used for systemic fungal infection
A. Griseofulvin
B. Clotrimazole
C. Amphotericin B
D. Econazole
E. Terbinafine
KEY: C
29. Case scenario of Pulmonary Edema, ask about drug of choice – Furosemide
30. Case scenario of hypertensive on drugs, patient develop dry cough – Enalapril
31. A 22-year old man, who frequently backpacks, complains of diarrhea and fatigue. Examination of stool
specimens shows bi-nucleate organisms with four flagellae. Which one of the following drugs would be
effective in treating this patient’s infestation
A. Metronidazole.
B. Quinidine.
C. Pentamidine.
D. Sulfadoxine.
E. Stibogluconate
KEY: A
32. Case scenario of a child using fluoroquinolones – Arthropathy
33. Glucocorticoid – osteoporosis
34. A 45 year old female suffering from acute gouty attack of left big toa is in need of a drug which should not
disturb her GIT
A. Allopurinol
B. Colchicine
C. Indomethacin
D. Acetaminophen
KEY: A
NEB/PMDC STEP-1 PAST PAPERS
DR.ASIF ALI KHAN
YANGTZE UNIVERSITY
35. Which of the following sulfonamide is used in the treatment of ulcerative colitis
A. Sulfadiazine
B. Sulfasalazine
C. Sulfadimidine
D. Sulfadoxine
E. Mefenid
KEY: B
Physiology
1. In athletes, physical fitness is more closely correlated with
A. Maximal pulse rate than with resting pulse rate
B. Maximal oxygen uptake (VO2max) than with resting oxygen uptake
C. Maximal minute ventilation than with maximal cardiac output
D. Size of muscles
E. Resting sympathetic tone than with maximal cardiac output
KEY:
2. If muscle strength is increased with restrictive training, which of the following conditions is most likely to
occurs
A. Decrease in the number of myofibrils
B. Increase in mitochonchondrial enzymes
C. Decrease in the components of phosphagen system
D. Decrease in stored triglycerides
E. Hyperkalemia
KEY:
3. The potassium in the renal tubule is
A. Only filtered and secreted
B. Only filtered and reabsorbed
C. Filtrated, reabsorbed, and secreted
D. Only secreted
E. Only filtered
KEY: C
4. During in the passage of an intravenous catheter, numerous endothelial cells are dislodged from lining of
popliteal vein. What substance allows platelet adhesion to the exposed collagen
A. Factor VIII
B. Factor IX
C. Fibronectin
D. Tissue factor
E. Von Willebrand factor
KEY:
NEB/PMDC STEP-1 PAST PAPERS
DR.ASIF ALI KHAN
YANGTZE UNIVERSITY
5. The hormone which has contraceptive role is:
A. Luteinizing hormone
B. Prolactin
C. Testosterone
D. Estrogen
E. Human chorionic gonadotropin
KEY:
6. A substance that is filtered, but not secreted or reabsorbed (substance X), is infused into a volunteer until
a steady state plasma level of 0.1 mg/mL is achieved. The subject then empties his bladder and waits one
hour, at which time he urinates again. The volume of urine in the second specimen is 60 mL and the
concentration of substance X is 10 mg/mL. What is the glomerular filtration rate (GFR) in this individual?
A. 30 mL/min
B. 60 mL/min
C. 100 mL/min
D. 300 mL/min
E. 600 mL/min
KEY: C
7. A 45 years obese man feeling hungry and thirsty most of the time, which increase frequency of urination
is recently diagnosed to have type 2 Diabetes mellitus. Considering the problem at glucose transporters
the following tissue is likely to be affected the most
A. Brain and liver
B. Liver and intestine
C. Muscle and adipose
D. Red blood cell and brain
E. Small intestine and brain
KEY: D
8. Growth Hormone secretion increase – Exercise
9. Splitting of S2 during inspiration occurs due to
A. Delayed closure of pulmonic valve
B. Early closure of pulmonic valve
C. Early closure of tricuspid valve
D. Early closure of mitral valve
KEY: A
10. 1st degree Heart block – syncope
11. Most sensitive cardiac marker – Troponin T
12. Presbyopia – Convex lenses
NEB/PMDC STEP-1 PAST PAPERS
DR.ASIF ALI KHAN
YANGTZE UNIVERSITY
13. In a patient with a urine flow rate of 1 mL/minute, the tubular fluid with the lowest osmolarity would be
found in the
A. Beginning of the proximal tubules
B. End of the cortical collecting tubule or duct
C. End of the papillary collecting tubule or duct
D. macula densa
E. tip of the loop of Henle
KEY: D, Tubular fluid first becomes hypotonic toward the end of the thick ascending limb of the loop of
Henle and will therefore be hypotonic by the macula densa (which is the border between the thick
ascending limb and the distal convoluted tubule).
14. A woodworker operating a bandsaw accidently injures his wrist, severing his radial artery and producing
severe hemorrhage. As he loses blood, his body tries to compensate for the developing hypotension by
increasing sympathetic outflow. The postganglionic signals carrying the impulses to constrict his arterioles
are transmitted along which of the following fiber types?
A. A & delta fibers
B. B fibers
C. C fibers
D. Ia fibers
E. Ib fibers
KEY: C, Type C - Post Ganglionic Autonomic fib-= Type IV (Unmyelinated)--Slow Pain & Temp
15. Muscle spindle:
A. Control body posture
B. Maintains the length of skeletal muscle bundle
C. Regulates muscle activity
D. Is the sensory stretch receptor
E. Relays motor impulse to the skeletal muscle
KEY: B
16. Secretion of oxytocin is increased by
A. milk ejection
B. suckling of the cervix
C. Increased prolactin levels
D. Increased extracellular fluid (ECF) volume
E. Increased serum osmolarity
KEY: B, Suckling and dilation of the cervix are the physiologic stimuli for oxytocin secretion. Milk ejection is
the result of oxytocin action, not the cause of its secretion. Prolactin secretion is also stimulated by suckling,
but prolactin does not directly cause oxytocin secretion. Increased extracellular fluid (ECF) volume and
hyperosmolarity are the stimuli for the secretion of the other posterior pituitary hormone, antidiuretic
hormone (ADH).
NEB/PMDC STEP-1 PAST PAPERS
DR.ASIF ALI KHAN
YANGTZE UNIVERSITY
17. Maintained lactation – Prolactin
18. Increase GFR…..efferent constriction
19. A 72-year-old woman with insomnia participates in a sleep study. As part of the study protocol, she has
EEG leads attached, then goes to sleep. At one point during the evening, 12-16 Hz sleep spindles and K-
complexes are observed. Which of the following stages of sleep is associated with this pattern?
A. REM
B. Stage 1
C. Stage 2
D. Stage 3
E. Stage 4
KEY: C, Explanation
1. Stage 2 has more theta waves than stage 1 and is associated with sleep spindles (short bursts of 12-16
Hz activity) and K-complexes (high amplitude slow waves with superposed sleep spindles) on the
electroencephalogram
2. Transient large amplitude potentials in the occipital areas (ponto-geniculo-occipital [PGO] spikes) are
associated with REM sleep (choice A).
3. Stage 1 (choice B), or drowsiness, is characterized by the attenuation of alpha rhythm (8-13Hz) and the
appearance of 4-7-Hz theta waves.
4. Stages 3 (choice D) and 4 (choice E), or slow wave sleep, are characterized by high amplitude slow
waves, especially in the delta (< 4 Hz) frequency range.
Pathology
1. A 35 years female who got her cervical smear done, which reveals High Grade Squamous Intraepithelial
Lesions (HSIL) which one of the following HPV types will be implicated in this lesion
A. 6 and 11
B. 5 and 8
C. 16 and 18
D. 19 and 22
E. 15 and 17
KEY: C, HPV 16 and 18
2. Japanese have a higher rate of stomach cancer and a low rate for colon carcinoma than the U.S. However
third generation descendants of Japanese immigrants to U.S have rates of stomach and colon cancer
like that of U.S. This particular characteristic supports effects of:
A. Environment
B. Genetics
C. Mutation
D. Accidents
E. Misinterpretation
KEY: A, Environment
NEB/PMDC STEP-1 PAST PAPERS
DR.ASIF ALI KHAN
YANGTZE UNIVERSITY
3. A 5 years old African male was chronically ill had swollen jaw and spleen and liver enlargement. The cells
on biopsy were monotonous small non-cleaved lymphocytes. Most likely chromosomal translocation to
be found in this disease is
A. t(4;22)
B. t(9;22)
C. t(12;22)
D. t(8;14)
E. t(x;18)
KEY: D - Burkett lymphoma case – t;(8;14)
1. ALL = 12;21
2. AML= 17;15
3. CML = 9;22
4. Burkitt lymphoma =8;14
5. Mantle cell 11;14
6. Follicular cell = 14;18
4. Deposition of calcium in nonviable or dying tissue in the presence of normal serum calcium levels is called
A. Metastatic calcification
B. Dystrophic calcification
C. Intracellular calcification
D. Calcium excess
E. None of the above
KEY: B
5. Most common karyotype seen in Down’s syndrome is:
A. Trisomy 13
B. Trisomy 21
C. Trisomy 16
D. Monosomy 21
E. Trisomy 12
KEY: B
6. Insertion or deletion of one or two base pairs altering the reading frame of the DNA strand is known as
Which of the following would result in a
A. Point mutation
B. Frame shift mutation
C. Trinucleotide repeat mutation
D. Missence mutation
E. Single nucleotide polymorphism
KEY: B - Frame shift mutations lead to lease effect when they lead to an insertion or deletion of 3 bases, as
due to our codon being triplet this doesn’t shift the reading frame. However, if the codon thus lost or added
is a very important one in the gene, there could be serious consequences.
NEB/PMDC STEP-1 PAST PAPERS
DR.ASIF ALI KHAN
YANGTZE UNIVERSITY
7. A 30 years old male patient presented in outdoor with complain of oral ulceration. The consultant
suspects a fungal infection. His medical record also showed that he is suffering from acquired immune
deficiency syndrome (AIDS). What is the most likely cause of his oral lesions?
A. Cryptococcosis
B. coccidiomycosis
C. Candidiasis
D. Histoplasmosis
E. Tinea
KEY: C
8. A specific form of necrosis, also visible under light microscope, which occur after deposition of immune
complexes in the walls of arteries is
A. Coagulative necrosis
B. Caseous necrosis
C. Fibrinoid necrosis
D. Fat necrosis
E. Liquefactive necrosis
KEY: C
9. Cytotoxic T-cells help in
A. Destroying tissue invaded by viruses
B. Producing antibodies
C. Activation of mast cells
D. Regulating T-cell activity
E. Suppressing the T-cell activity
KEY: A
20. A person with endemic goiter has
A. High level of Plasma TSH
B. High level of Plasma thyroxine
C. High level of Plasma triiodothyronine
D. Low production of thyroglobulin
E. Low level of growth hormone
KEY: A, High level of Plasma TSH (Decreased T3, T4 and Increased TSH)
10. A 42 years old obese women presented to the emergency room with complain of worsening nausea,
vomiting and epigastric pain. The blood chemistry revealed a high Amylase level. Most likely diagnosis is:
A. Acute appendicitis
B. Acute Pancreatitis
C. Gastritis
D. Renal colic
E. Viral hepatitis
KEY: B,
NEB/PMDC STEP-1 PAST PAPERS
DR.ASIF ALI KHAN
YANGTZE UNIVERSITY
11. Sterilization by autoclaving is best confirmed by
A. Processing at temp of 120 o
C
B. Ten minutes incubation
C. 10 lb/in2 pressure
D. 90% air removal
E. Bowie Dick test
KEY: A
12. Incidence of Bile duct is increased in individuals with infestation by
A. E. histolytica
B. Schistosoma mansoni
C. Fasiola hepatica
D. Echinococcus
E. Clonorchis sinensis
KEY: E, Clonorchis sinensis (Cholangiocarcinoma)
13. The test that screens the extrinsic pathway is
A. Prothrombin time (PT)
B. Activated partial thromboplastin time (aPTT)
C. Thrombin time
D. Urea solubility test
E. Clot lysis time
KEY: A
14. A 65-year-old female with renal failure presents for hemodialysis. She is found to be anemic and is
given a dose of erythropoietin along with her usual vitamin and mineral supplements.
Erythropoietin stimulates which of the following intermediates in hematopoiesis?
A. Basophilic erythroblasts
B. Colony forming units-erythroid
C. Multipotential stem cells
D. Proerythroblasts
E. Reticulocytes
KEY: B, The colony forming unit-erythroid (CFU-E) is a unipotential stem cell that develops from a burst
forming unit-erythroid (BFU-E), which develops eventually from the multipotential stem cell. The BFU-E is
somewhat responsive to erythropoietin, but the CFU-E is completely dependent on erythropoietin.
Erythropoietin is normally released from the kidney in response to hypoxic or anemic conditions. Its half-life
is about 3-6 hours. Clinically it takes 5 days to see reticulocyte formation in the peripheral blood following
erythropoietin administration
NEB/PMDC STEP-1 PAST PAPERS
DR.ASIF ALI KHAN
YANGTZE UNIVERSITY
15. A 52 years old female having chronic viral hepatitis present with pain right hypochondrium. Ultrasound
abdomen shows a space occupying lesion in the right lobe. The most likely diagnosis is
A. Hepatic adenoma
B. Hepatocelullar carcinoma
C. Angiosarcoma
D. Cyst of echinococcus granulossus
E. Amoebiasis
KEY: B, Hepatocellular carcinoma
16. A women smokes one pack/day, she drinks at least 5 cups of coffee/day. Her mother died of breast
cancer. Which is the greatest risk factor for breast cancer in this patient
A. Birth control pills
B. Cigarette smoking
C. Family history
D. Caffeine intake
E. Nulliparity
KEY: C
17. A 27 years old man presents with a mass at the base of skull extending down in the oral cavity, which on
biopsy was diagnosed as being a yolk sac tumour. Which of the following tumour marker is most likely to
be increased in this patient’s serum as a result of being secreted from the cells of this tumour?
A. Acid phosphatase
B. Alpha-fetoprotein (AFP)
C. Alkaline phosphatase
D. Beta-human chorionic gonodotriphin (Beta HCG)
E. Calcitonin
KEY: B, Alpha-fetoprotein (AFP)
» AFP is Measured in routine screening around 15-18 weeks (second trimester)
» AFP and amniocentesis can help detect: neural tube defects and trisomies
» AFP is synthesized in the fetal liver, GI tract, and yolk sac
» Alpha-fetoprotein (AFP) is tumor marker for Hepatocellular carcinoma (HCC), Hepatoblastoma, Yolk sac
tumor of the ovary (endodermal sinus tumor), Mixed germ cell tumor
» Transient elevation during pregnancy
» AFP increased in:
 abdominal wall defects: anacephaly, meningomyelocele, encephalocele
 neural tube defects: gastroschisis, omphalocele
 twin gestations: if the measures twice the median value then evaluate by ultrasound and
amniocentesis
» AFP decreased in: associated with:
 Trisomy 21 (Downs Syndrome)
 Trisomy 18 (Edwards Syndrome)
 Trisomy 13 (Patau Syndrome)
NEB/PMDC STEP-1 PAST PAPERS
DR.ASIF ALI KHAN
YANGTZE UNIVERSITY
18. Aortic dissection occurs in a tall 31 years old male with long fingers. His ophthalmic examination reveals
ocular lens dislocation. These features are found in
A. Down’s syndrome
B. Glycogen storage disease
C. Neurofibromatosis
D. Marfan syndrome
E. Tay sachs disease
KEY: D
19. A 4 year old boy presents with fever, an abdominal mass which has grown rapidly over the last four
months accompanied by weight loss. Ultrasound confirms a solid space occupying lesion in the kidney.
The diagnosis is:
A. Renal cell carcinoma
B. Transitional cell carcinoma of kidney
C. Squamous cell carcinoma of renal pelvis
D. Wilms tumor
E. Hydronephrosis
KEY: D
20. Which of the following organism is responsible of causing infective endocarditis after skin infection
A. S. Aureus
B. Strep. Viridians
C. Haemophilus
D. Eikenella
E. Kingella
KEY: A
21. A 35 years old truck driver presented in STI clinic with complain of painless of ulceration of on external
genitalia. On examination there is chancre formation with inguinal and femoral lymphadenopathy.
A. Neisseria gonorrhoea
B. Human papilloma virus
C. Chlamydia trachomatis
D. Mycobacterium genitalium
E. Treponema pallidum
KEY: E
NEB/PMDC STEP-1 PAST PAPERS
DR.ASIF ALI KHAN
YANGTZE UNIVERSITY
22. A 30 years old patient presented in emergency with multiple abscesses on different parts of his body. He
also gave the history of fever with evening rise and productive cough. Blood picture showed lymphocytic
predominance. Acid fast stain is also positive. X-ray showed an endobrochial mass. What is most likely
diagnosis
A. Pneumonia
B. Cellulitis
C. Nocardiasis
D. Kaposi sarcoma
E. Sarcoidosis
KEY: C
23. A group of students developed vomiting, abdominal pain and diarrhea six hours after having lunch
consisting of Chinese rice at restaurant. The most likely organism involved is:
A. Clostridium botulinum
B. Vabrio cholera
C. Bacillus cereus
D. Enterotoxicgenic escherchia coli
E. Clostridium perfringens
KEY: C
24. Malignancy – Telomerase
25. Peripheral cyanosis – Primary Reynaud Phenomenon
26. Cells against TB – CD4 T lymphocyte
27. Endothelium platelet aggregation – Von Willebrand Factor
28. Pregnant lady with hepatitis , how to confirm diagnosis — SGOT
29. A 30 years old active sex worker presented in the outdoor of tehsil headquarters hospital complaining of
pain, itching and prulent vaginal discharge. Female medical officer took the sample of pus for lab
examination also. Which organism is most likely cause of these symptoms
A. C. Trachomatis
B. H. Papilomavirus-2
C. H. Papilomavirus-7
D. Trichomonas Vaginalis
E. U. Urealyticum
KEY: D
Systemic fungal
30. Beta Thalassemia – Hypochromic and Microcytic anemia
31. Increased An-ion gap is seen in – Metabolic Acidosis
32. Infant with vomiting – Pyloric Stenosis
33. Ovary mass with Caseous necrosis – Tuberculosis
NEB/PMDC STEP-1 PAST PAPERS
DR.ASIF ALI KHAN
YANGTZE UNIVERSITY
34. A 70-year-oltl Negroid man from Makkran coast was stabbed on the upper arm, he received emergency
treatment and his wound healed. He developed a raised hypertrophic scar with boundaries beyond the
original wound, and it did not regress. Which of the following terms best describes this condition:
A. Cicatrix
B. Keloid
C. Callus
D. Granulation tissue
E. Wound
KEY: B
35. Myasthenia Graves – Abs against Ach receptors
36. Vasodilation – Septic shock
37. A transudate is edema fluid
A. Rich in proteins
B. Rich in neutrophils
C. Rich in albumin
D. Low in proteins
E. Rich in fibrinogen
KEY: D
38. A 50 years old non-smoker lady with alpha-1 antitrypsin deficiency developed severe lung disease and
requires a double lung transplant. What is the most likely lung lesion she is having?
A. Adenocarcinoma
B. Bullous emphysema
C. Centriacinar emphysema
D. Interstitial emphysema
E. panacinar emphysema
KEY: E
39. Normal flora of intestine – E. coli
40. UV radiations – Oxygen free radicals
41. Specific with Cardiac Muscle – CK-MB
42. Steroid – increase neutrophil
43. Esophageal varices…cirrhosis
44. Allergy – IgE
45. transplant rejection – cytotoxic t cell
NEB/PMDC STEP-1 PAST PAPERS
DR.ASIF ALI KHAN
YANGTZE UNIVERSITY
Community Medicine
1. Low birth baby is one who has weight at birth less than
A. 1.5 kg
B. 2 kg
C. 2.5 kg
D. 3 kg
E. 3.5 kg
» KEY: C, 2.5 kg
2. A mother bought her six months old child to a BHU. She is worried about the growth of her child. The best
single measure for assessing the physical growth in this age
A. Weight for age
B. Height for age
C. Chest circumference
D. Mild upper arm circumference
E. Head circumference
KEY: A, weight for age
3. BMI is calculated with the help of
A. Age and weight
B. Height and age
C. Height and weight
D. Mid upper area circumference and height
E. Mid upper area circumference and weight
KEY: C, height and weight
4. Most sensitive indicator of health of status of population is
A. Crude death rate
B. Infant mortality rate
C. Maternal mortality rate
D. Child mortality rate
E. Dependency rate
KEY: B, Infant mortality rate
5. In the data 15, 2, 9, 10, 18, 14 and 72 the median is
A. 10
B. 14
C. 72
D. 15
E. 18
KEY: B, 14
NEB/PMDC STEP-1 PAST PAPERS
DR.ASIF ALI KHAN
YANGTZE UNIVERSITY
6. Case fatality rate – 25%
7. Concept of primary Health Care and Health for All by 2000 was evolved in 1978 at Alma Ata is it
A. China
B. Russia
C. America
D. Egypt
E. Kazakhstan
KEY: E, Kazakhstan - The Declaration of Alma-Ata was adopted at the International Conference on Primary
Health Care (PHC), Almaty (formerly Alma-Ata), Kazakhstan (formerly Kazakh Soviet Socialist Republic), 6–12
September 1978. It expressed the need for urgent action by all governments, all health and development
workers, and the world community to protect and promote the health of all people. It was the first
international declaration underlining the importance of primary health care.
8. Weaning should be started usually at
A. Birth
B. 4-6 months
C. 1 year
D. 1 month
E. At any time
KEY: B, 4-6 months
9. Pakistan is attached with WHO regional HQ of:
A. South East Asia region
B. Africa region
C. Western Pacific region
D. Eastern Mediterranean region
E. None of the above
KEY: A, South East Asia region
10. COCP – Ethylene Estradiol
11. Neonatal tetanus is an important preventable cause of neonatal mortality that account for 7% of neonatal
deaths worldwide. This could be prevented by:
A. Vaccination of women in reproductive age group
B. Vaccination of newborn’s
C. Vaccination of pregnant ladies
D. Vaccination of females of child bearing age
E. Vaccination of non-pregnant females
KEY: D, Vaccination of females of child bearing age
12. Vaccination for those going abroad Saudi to perform hajj
A. Meningitis
B. Tetanus
NEB/PMDC STEP-1 PAST PAPERS
DR.ASIF ALI KHAN
YANGTZE UNIVERSITY
C. Pneumonia
D. Cerebral Malaria
E. Influenza
KEY: A, Meningitis vaccination
13. A 46 year old male worker of a textile factory presented with 02 months history of cough, fever and
hemoptysis. The exposure to cotton dust in the factory lead to a this condition known as
A. Bagassosis
B. Byssinosis
C. Anthracosis
D. Silicosis
E. Asbestosis
KEY: B, Byssinosis
14. Limitation of freedom movements of apparently healthy individuals for the period of not more than the
longest incubation period of particular disease is
A. Isolation
B. Quarantine
C. Surveillance
D. Segregation
E. All of the above
KEY: B, Quarantine - Who has been exposed to a case of communicable disease for maximum incubation
period, is termed as Quarantine
15. Dental caries is due to lack of
A. Chlorine
B. Fluorine
C. Iodine
D. Iron
E. Calcium
KEY: B, Fluorine
16. Bone breaking disease also known as
A. Typhus fever
B. Dengue fever
C. Hay fever
D. Congo hemorrhagic fever
E. Yellow fever
KEY: B, Dengue fever
NEB/PMDC STEP-1 PAST PAPERS
DR.ASIF ALI KHAN
YANGTZE UNIVERSITY
17. A 26 years old female patient, at 3rd day of delivering a baby developed cyanosis, shortness of breath and
orthopnea. What is the most likely underlying pathology causing these signs and symptoms -- Amniotic Fluid
Embolism
18. 28 year old healthy mother of 2 with fear side effects of IUCD and COCP what should advice after proper
counseling – COCP
19. Hospital waste is best disposed of by
A. Composting
B. Dumping
C. Incineration
D. Burial
E. None of the above
KEY: C, Incineration
20. If annual growth of population is 2.0% than it will be double in
A. 35 years
B. 28 years
C. 23 years
D. 47 years
E. 50 years
KEY: A, 35 years
21. If the population of a country grows at a rate of approximately 5 percent per year, the number of years
required for the population to double is closest to
A. 5 years
B. 10 years
C. 15 years
D. 25 years
E. 35 years
KEY: C, 15 years
22. Shortly after a barbeque, Medical students of a medical college came back to their rooms and most of
them (62 out of 74 students) experienced acute vomiting and diarrhea. This epidemic maybe classified as
A. Point source
B. Propagative
C. Common source
D. Serial direct
E. None of the above
KEY: A Point source
NEB/PMDC STEP-1 PAST PAPERS
DR.ASIF ALI KHAN
YANGTZE UNIVERSITY
23. An operon is best described as
A. A constitutively expressed gene system
B. An unregulated gene system
C. A co-ordinately regulated gene system
D. A gene that produces monocistronic messenger RNA
E. A combination of gene expressed independent of each other but under influence of
KEY:
24. Geriatrics deals with the people
A. Neonates
B. Aldosence
C. Teenagers
D. Old age
E. Orphan
KEY: D, Old age
25. After a disaster, TRIAGE is an approach for
A. Provision of food and shelter to the people
B. Selection of injured who can be benefited the most from urgent medical treatment
C. Rehabilitation of the affected people
D. Removal of dead bodies
E. Immunisation of people
KEY: A, - The approach of rapidly classifying the injured on the basis of severity of their injuries and
likelihood of their survival with prompt medical intervention after disaster is called as Triage
26. Sporadic epidemic
Forensic Medicine
1. Munaqillah – Fracture with dislocation
2. Phossy jaw (mandibular necrosis) is associated with which of the following occupation is seen in which
poisoning?
A. Felt hat makers
B. Leather manufacturers
C. Match makers
D. Shipyard workers
E. Synthetic drivers
KEY: C
NEB/PMDC STEP-1 PAST PAPERS
DR.ASIF ALI KHAN
YANGTZE UNIVERSITY
3. Hatter shake is observed in chronic poisoning with?
A. zinc
B. Mercury
C. Lead
D. copper
E. Arsenic
KEY: B
4. Phosphorus poisoning most commonly seen in workers in the – Match industry
5. Shajjah-i-Munqaliba means:
A. Causing Shajjah with exposing bone of the victim but without causing fracture
B. Causing Shajjah by fracturing the bone of the victims but without dislocation
C. Causing Shajjah by both fracturing and dislocation of bone of the victims
D. Causing Shajjah by causing fracture of the skull and wound touches the membranes of the brain
E. Causing Shajjah by causing fracture of the skull and the wound ruptures the membrane of the brain
KEY: C, Explanation
1. Shajjah-i-Khafifa Causing Shajjah without exposing bone of the victim
2. Shajjah-i-Mudihah: Causing Shajjah with exposing bone of the victim but without causing fracture
3. Shajjah-i-Hashimah: Causing Shajjah by fracturing the bone of the victim but without dislocating it
4. Shajjah-i-Munaqqilah: Causing Shajjah by both fracturing and dislocating the bone of the victim
5. Shajjah-i-Ammah: Causing Shajjah by causing fracture of the skull and the wound touches the
membranes of the brain
6. Shajjah-i-Damighah: Causing Shajjah by causing fracture of the skull and the wound ruptures the
membranes of the brain
6. The direction of fire can be determined most reliably from:
A. The nature of the tattooing around entrance wound
B. The bullet track in the tissues
C. Grease mark
D. Collar of abrasion
E. Preterm of damage on clothing
KEY:
7. Following statement does not depict characteristic of a bullet wound
A. Exit wound is usually irregular in shape
B. Damage can be produced at a distance away from the original bullet track
C. With low velocity gunshot, the main wound occurs due to cavitation phenomenon
D. Entry produced a clean-cut hole in skull
E. Entry produce a stellate shape wound on head
KEY:
NEB/PMDC STEP-1 PAST PAPERS
DR.ASIF ALI KHAN
YANGTZE UNIVERSITY
8. In Pakistan, a medical officer is authorized to perform medicolegal post-mortem examination under
authority of following law:
A. Section 302 of Pakistan penal code 1869
B. Section 174 of code of criminal procedure 1898, Pakistan
C. Section 307 of Pakistan penal code
D. Section 274 of code of criminal procedure 1898, Pakistan
E. Section 374 of code of criminal procedure 1898, Pakistan
KEY:
9. The snake bite of family viperidae is commonly identified by one of the following effect:
A. Nephrotoxic
B. Neurotoxic
C. Mytoxic
D. Heamatotoxic
E. Vasculotoxic
KEY: D
10. Whiplash is which form of injury
A. Hyper flexion
B. Hyperextension
C. Lateral flexion
D. Atlanto-axial dislocation
E. ..
KEY: B
11. Brush burn refers to
A. Electric burn
B. Lightening burn
C. Sliding abrasion
D. Pressure abrasion
E. ..
KEY: C
12. Which one of the following organic acids is made from methanol
A. Acetic Acid
B. Botanic acid
C. Formic acid
D. Propanoic acid
E. ..
KEY: C
13. A man survives after giving dying declaration?
A. He has to give oral oath for evidence or something
B. His declaration is valid for one year
C. Declaration is null and void.
D. No value
E. ..
NEB/PMDC STEP-1 PAST PAPERS
DR.ASIF ALI KHAN
YANGTZE UNIVERSITY
14. The legal right of consent is conferred to patient under the principle of
A. Benevolence
B. Autonomy
C. Paternalism
D. Justice
E. Religion
15. First treatment for snake bite is
A. Antisnake venom
B. Application of tourniquet
C. Clean the wound with soap & water
D. Local incision & suction
E. All
KEY: B, Application of tourniquet
16. A 46 year old male worker of a textile factory presented with 02 months history of cough, fever and
hemoptysis. The exposure to cotton dust in the factory lead to a this condition known as
A. Bagassosis
B. Byssinosis
C. Anthracosis
D. Silicosis
E. Asbestosis
KEY: B
TYPES OF OCCUPATIONAL LUNG DISEASES
DISEASE CAUSATIVE AGENT
1. Silicosis » Inhalation of free silica or silicon dioxide
2. Asbestosis » Inhalation of asbestos
3. Byssinosis » Cotton fiber dust
4. Bagassosis » sugarcane dust or fiber
5. Farmer’s lung » moldy hay or grain dust (fungal spore)
6. Siderosis » Iron dust
7. Coal worker’s
pneumoconiosis
(Anthracosis)
» Coal worker
8. Berylliosis » Beryllium
NEB/PMDC STEP-1 PAST PAPERS
DR.ASIF ALI KHAN
YANGTZE UNIVERSITY
17. The demographics cycle has five stages, the late expanding stage of the cycle indicates
A. High birth rate and high death rate
B. Decreasing birth rate and decreasing death rate
C. Decreasing death rate and stationary birth rate
D. Low birth rate and low death rate
E. High birth rate and stationary death rate
KEY:
18. A high school has experienced a series of incidents of sexual harassment in the hallways. One of the
health education teacher assigns students to create posters in class encouraging positive peer pressure to
prevent this behavior
A. Group approach
B. Individual approach
C. Mass approach
D. Personal approach
E. Massive approach
KEY:
19. The average number of children of a women will have if she were to pass through reproductive years
bearing children at the same rate as the women in each age group is
A. General fertility rate
B. General marital fertility rate
C. Total fertility rate
D. Gross reproductive rate
E. Total marital fertility rate
KEY:
20. A researchers collects data to find the prevalence of smoking among school children in Lahore, he is using
A. Sampling
B. Test of significance
C. Inferential statistics
D. Descriptive statistics
E. Complete enumeration
KEY:
21. Orthotolidine test is carried out to
A. Estimate the odour in water
B. Estimate fluorine content in water
C. Estimate chlorine content in water
D. Estimate nitrate level of water
E. Estimate nitrite level of water
KEY:
NEB/PMDC STEP-1 PAST PAPERS
DR.ASIF ALI KHAN
YANGTZE UNIVERSITY
22. Denominator in Crude death Rate is
A. Population at risk
B. Mild year population
C. Total No of births
D. Total No of death
E. None of the above
15 APRIL 2018 NUMS
MCQS OF NEB PMDC STEP-1
Community medicine
1. Case scenario of Cement industry worker reported to you with complaints of cough, dyspnea on exertion
and chest pain. His X-ray chest showed 'snow storm' appearance. The diagnosis would be:
A. Asbestosis
B. Siderosis
C. Silicosis
D. Aspergillosis
E. Byssinossis
KEY: C
2. Case scenario of a worker, Ground glass appearance on X ray is seen in
A. Asbestosis
B. Silicosis
C. Anthracosis
D. Berylliosis
E. Byssinosis
KEY: A
3. Night sweats , productive cough — TB
4. Shock after IUCD— uterine perforation
5. Drug stopped mortality— prevalence will be increased
6. Hair tilt— spina biffida
7. Giving penicillin— secondary care
8. Psychotherapy/rehabilitation— tertiary
9. Non immunized child— BCG , OPV, DPT ,measles , Vit A
10. Infection avoided— hand washing
11. Tb diagnosed and OCP— rifampicin
NEB/PMDC STEP-1 PAST PAPERS
DR.ASIF ALI KHAN
YANGTZE UNIVERSITY
12. Needle go to — sharp container
13. Rabies— Ig, vaccine , wound open
14. Calories count— 105
15. Folic acid dose— 0.5/0.4?
16. Niejeria to pak— yellow fever
17. Number of basic health units in Pak — 5400
18. Mumps – Parainfluenza virus
Anatomy
1. The nerve which passes through the quadrangular space of the posterior shoulder
A. Axillary nerve
B. Ulnar nerve
C. Median nerve
D. Musculocutaneous nerve
E. Vagus nerve
KEY: A
» Axillary nerve passes through the quadrangular space and then winds around the surgical neck of the
humerus, it supplies teres minor and deltoid muscle
» The quadrangular space is bounded medially by the tendon of the long head of the triceps, laterally by
the humerus, superiorly by teres minor, and inferiorly by teres major. The posterior circumflex humeral
artery and the axillary nerve traverse this space. The axillary nerve innervates 2 muscles: deltoid and
teres minor.
2. Cavernous sinus — facial vein
3. The facial muscle that produces horizontal wrinkles on the forehead is
A. Procerus
B. Corrugator supercilii
C. Frontal belly of occipitofrontalis
D. Orbicularis oculi
KEY: C , The frontal belly of occipitofrontalis elevates the skin of the forehead (panniculus carnosus)
producing horizontal wrinkles on the forehead, thus helping the eyes to open wider and also to produce an
expression of surprise.
4. Sole of foot – post tibial artery
5. Outer quadrant breast lymph – pectoral
6. Thyroid derived from – endoderm
7. Parathyroid – cheif cells
NEB/PMDC STEP-1 PAST PAPERS
DR.ASIF ALI KHAN
YANGTZE UNIVERSITY
8. Scavenger cells – microglia
9. PNS myelination – Schwan cells
10. Frozen shoulder muscle – sub scapularis
11. Medial rotation – Erbs palsy
12. Motor area in cerebrum – primary motor
13. Organ of corti – vestibulocochlear
14. Permanent mydriasis— oculomotor nerve
15. Horner syndrome — sympathetic nerve
16. Tongue deviated on right side — right hypoglossal nerve damaged
17. Patellar reflex — L3 L4
18. Pain after appendectomy — iliohypogastric nerve
19. Medial thigh problem after disc herniation— L 4
20. Saturday night palsy — radial nerve
21. Duodenal ulcer— gastro duodenal artery
22. Moving touch along sides — ruffini
23. Half face weakness— same sided facial nerve
24. Nerve between c5 and c6 — c6
Pharmacology
1. interferon — flue like symptoms
2. Cholicine MOA — muscle spindle
3. Heart failure diuretic — spironolactone
4. A 60-year-old man is admitted to the hospital with acute heart failure and pulmonary edema. Which of
the following drugs would be most useful in treating the pulmonary edema?
A. Digoxin
B. Lisinopril
C. Dobutamine
D. Furosemide
E. Spironolactone
KEY: D
5. Metallic taste— metronidazole
» Metronidazole: it is used for treatment of anaerobic infections and certain ... The side effects include
nausea, headache, a metallic taste and loss of appetite.
NEB/PMDC STEP-1 PAST PAPERS
DR.ASIF ALI KHAN
YANGTZE UNIVERSITY
6. A 30 year old female developed antibiotics-induced colitis due to C. difficile; she was given metronidazole
to treat it but it proved ineffective. Now she can respond to oral
A. Amoxicillin
B. Erythromycin
C. Chloramphenicol
D. Doxycycline
E. Vancomycin
KEY: E
7. One of the following oral hypoglycemic agent help the diabetic patient by promoting the release of
endogenous insulin
A. Acarbose
B. Metformin
C. Glipizide
D. Pioglitazone
E. Miglitol
KEY: C
8. Hyperthyroidism – PTU
9. Patient with history of Prolactinoma treated by – Bromocriptine
10. Asthma by which drug— aspirin
11. Bradycardia due to vagal— atropine
12. Short procedure anesthesia— propofol
13. Sublingual drug— nitroglycerine
14. Angina Rx— Nifedipine
15. Asthma prophylaxis — cromolyn sodium
16. Case fatality — 10%
17. Antifungal for mouth — Nystatine
18. Oral antifungal – Griseofulvin
19. Antiepileptic drug interaction— sodium valproate
20. Anti-Peptic ulcer — PPI
21. Opioids — naloxone
22. Drug give with atropine — diphenoxylate
23. Add with amoxicillin — clavulanic acid
24. In pregnancy — amoxicillin
25. Diabetic ketoacidosis (DKA) treatment — regular insulin
26. Phyrngitis — Doxycycline
27. Tetracycline to give in renal insufficiency — Doxycycline or oxytetracycline?
NEB/PMDC STEP-1 PAST PAPERS
DR.ASIF ALI KHAN
YANGTZE UNIVERSITY
28. Pregnancy combination— ticarcillin and tobramycin
29. SVT – adenosine
30. Potassium channel blocker – Amiodarone
31. Hyperkalemic hypertensive patient – thiazide diuretic
32. A 36-year old woman who is pregnant is diagnosed with hypertension. Which of the following agents
would be the most suitable to treat her hypertension?
A. Atenolol
B. alpha-methyl dopa
C. Nifedipine
D. Clonidine
E. Lisinopril
KEY: B
33. A 67-year-old man is hospitalized recovering from a left wall myocardial infarction. He begins to show signs
of fluid retention. His doctors want to start a drug regimen for congestive heart failure, including either an
ACE inhibitor or an angiotensin receptor blocker (ARB). ACE inhibitors and ARBs treat hypertension in a
similar fashion and have similar side effects. Which of the following is a side effect of ACE inhibitors only –
dry cough
34. Tremors is side effect of—salbutamol/Salmeterol
35. Which of the following anti-seizures drug is most likely to elevate the serum concentration of other drugs
administration concomitantly
A. Carbamazepine
B. Diazepam
C. Phenobarbitone
D. Phenytoin
E. Valproic acid
KEY: E
36. Abrupt withdrawal of anti-seizure drugs can result in increases in seizure frequency and severity.
Withdrawal is most easily accomplished if the patient is treated with
A. Carbamazepine
B. Clonazepam
C. Ethosuximide
D. Phenobarbitone
E. Phenytoin
KEY: A
37. Case scenario of Parkinson’s disease, Antiviral used in the treatment of tremors
A. Ribavirin
B. Acyclovir
C. Amantadine
D. Oseltamivir
KEY: C
NEB/PMDC STEP-1 PAST PAPERS
DR.ASIF ALI KHAN
YANGTZE UNIVERSITY
Physiology
1. Cardiac markers — 3 MCQS
2. Transudate — low protein in cells
3. L/S ratio— lung maturity
4. Running, cardiac blood flow increased by local arteriolar metabolites??
5. Fast conduction pathway—bundle of his
6. Short acting blood control— barro receptor
7. Phosphate absorption —PCT
8. Cardiac output is – SV into HR
9. Cardiogenic shock— decreased cardiac output
10. Saw tooth waves on ECG is seen in
A. Atrial flutter
B. Arterial fibrillation
C. Digoxin toxicity
D. Hyperkalemia
E. Ventricular fibrillation
11. Lazy man — 30 mins walk / day or thrice a week?
Pathology
1. A 52 years old female having chronic viral hepatitis present with pain right hypochondrium. Ultrasound
abdomen shows a space occupying lesion in the right lobe. The most likely diagnosis is
A. Hepatic adenoma
B. Hepatocelullar carcinoma
C. Angiosarcoma
D. Cyst of echinococcus granulossus
E. Amoebiasis
KEY: B, Hepatocellular carcinoma
2. Liver abscess — Echinococcus?
3. Bloody stools — iron deficiency anemia
4. IgG + HbeAg — chronic infectivity
5. Gall stones obstruction— bile duct
6. Germ cell tumor—dysgerminoma
7. Pregnant female present with Acne, which is Teratogenic Rx for Acne — vitamin A
NEB/PMDC STEP-1 PAST PAPERS
DR.ASIF ALI KHAN
YANGTZE UNIVERSITY
8. A term infant is born and does well with breast feeding. Two days later his mother calls frantically because
the baby is bleeding from umbilicus as nostrils. The following vitamin is most probably deficient in this
infants:
A. Vitamin A
B. Vitamin D2
C. Vitamin D3
D. Vitamin E
E. Vitamin K
KEY: E
9. Black water fever— falciparum
10. H-pylori— adenocarcinoma
11. Abdominal aorta — atherosclerosis
12. Blood transfusion — O Negative blood
13. Exophthalmos — graves’ disease
14. Conns syndrome — hypokalemia
15. Hypoglycemia— insulinoma
16. Liver dx — ALT will increase
17. Pernicious anemia— intrinsic factor deficiency
18. Microcytic anemia —iron
19. Megaloblastic anemia —b12
20. Investigation after Hb in macrocytic anemia—Hb electrophoresis
21. Auer rods is seen in — AML
22. Commonest leukemia in kids — ALL
23. Hepatitis B — HBsAg
24. Hepatitis infectivity — Hbe
25. To check in Thalassemia — HbA2
26. Splenomegaly—sickle cell anemia
27. For monitoring Warfarin drug check — Prothrombin Time (PT)
28. Swollen abdomen, thin legs n arms — hepatic cirrhosis
29. First line defense — skin
30. Tb test HSV type — four
NEB/PMDC STEP-1 PAST PAPERS
DR.ASIF ALI KHAN
YANGTZE UNIVERSITY
31. Following an IV dose of penicillin, a middle-aged man develops allergy such as skin rash and breathing
difficulty followed by hypotension and shock. The hypersensitivity reaction is most likely mediated by
which of the following?
A. C3b
B. IgE
C. IgM
D. Prostaglandins
E. CD 4+ lymphocytes
KEY: B
32. Anaphylactic shock — IgE
33. Placenta cross — IgG
34. Which of the following is the leading cause of severe diarrhoea among young children?
A. RSV
B. Adenovirus
C. Rota virus
D. EBV
E. Echovirus
KEY: C
Forensic medicine
1. Patient death, Doctor was drunk — criminal
2. Didn’t do x-rays — civil negligence
3. Didn’t tell anything to patient — blanket consent
4. Arsenic is used for — homicidal
5. Multiple throat cuts—suicide
6. Stab wound patient — inform police
7. Blast injury— blast lung
8. Wedding firing—ricochet
9. Murree, cherry red— CO poisoning
10. Bugs under skin—coccaine poisoning
11. Chronic poisoning check — long bones
12. Farmers came with poison — OP poisoning
13. Wrestlers eye — contusion
14. Bluish black bruise —2-4 days
15. More bleeding in— incised wounds
NEB/PMDC STEP-1 PAST PAPERS
DR.ASIF ALI KHAN
YANGTZE UNIVERSITY
16. Case: A 6 years old girl ate her mother’s anxiety drugs, she may be best treated by giving her an
antagonist like:
A. Flumazenil.
B. Naloxone.
C. Fomepizole.
D. Nalorphine.
E. Naltrexone.
KEY: A
17. Vanilla flavor custard— food additive
18. Birds foot injury— driver
19. Consent from injured — written
20. Gunshot on right side—ipsilateral lower motor neuron injury
Biochemistry
1. The diet of a child suffering from Maple syrup urine disease (an amino acid disorder), should be low, in
which out of the following amino acids content?
A. Branched chain amino acids
B. Phenylalanine Alanine
C. Methionine
D. Tryptophan
KEY: A
2. Lactate — cori cycle
3. Phenyl ketone urea, what to avoid — phenylalanine
4. Maximum substance made by—phenylalanine
5. Alcoholic patient— thiamine deficiency
6. Alcoholic with neurological problems—vit B 12
7. Cheilosis —riboflavin
8. Dementia — niacin
9. Hemopoitic vit— vitamin C
10. Vitamin in fats— vitamin E
11. In severe starvation energy comes from — amino acids
12. Patient on vegetable diet — b12 deficiency
13. Urea cycle enzyme deficiency — glutamine will increase
14. Bloating with milk— lactase deficiency
15. Exercise will reduce— LdL ? Vldl ?
NEB/PMDC STEP-1 PAST PAPERS
DR.ASIF ALI KHAN
YANGTZE UNIVERSITY
16. Common sugar made her obese.. due to glucose or fructose? ]Number of BHUs (Basic health units) in
Pakistan
– According to Economic Survey of Pakistan 2016, currently there are 5464 basic health units in the
country. The country's public health care system comprises of 1167 hospitals, 5695 dispensaries, 675
rural health centers(RHCs), 733 mother and child health centers and allied medical professionals)
– 2017-2018 -- By the year 2017, the number of public sector hospitals has increased to 1211, 5508 basic
health units (BHUs), 676 rural health centers (RHCs) and 5,697 dispensaries.
PMDC Neb Step-1 Exam held on 25 November 2018
1. Basic health units (BHU) in Pakistan — 5000 - 6000
2. weakness of thenar muscles & sensation lost lateral aspect — Median nerve
3. Medial epicondyle fracture — ulnar nerve injury
4. In fracture of medial epicondyle of humerus, the patient suffers claw-hand deformity and sensory loss in
medial 1/3' of palm and medial 11/2 fingers. The damaged nerve is:
A. Musculocutaneous nerve.
B. Ulnar nerve
C. Median nerve
D. Medial coetaneous nerve of forearm
E. Radial nerve.
KEY: B
5. A 23-year-old lady presents with abnormal spoon shaped nails (Koilonychia), pallor, weakness and
palpitations and glossitis—Iron deficiency anemia
6. A 2 years old child presented with weakness and bowing of legs on x-ray — rickets
7. Laceration look a like incisions seen — on head/scalp
8. Patient with amenorrhea and nipples spread widely — Turner
9. Post UTI infection
10. 5th intercostal space which valve — Mitral
11. Thyroid – thyroglossal duct
12. BMI — 24.5
13. Alma-Ata declaration — physician oath
14. Isoniazid — pyridoxine deficiency
NEB/PMDC STEP-1 PAST PAPERS
DR.ASIF ALI KHAN
YANGTZE UNIVERSITY
15. A 32-year-old body builder has decided to go on a diet consisting of egg whites to ensure only proteins for
muscle growth. After a few weeks he experiences decreased energy and is found to be hypoglycemic. A
nutritionist tells the patient that he most likely has the deficiency of vitamin biotin. Which of the
following enzymes is unable to catalyze its step in synthesizing glucose from pyruvate.
A. Pyruvate carboxylase
B. Phospho enol pyruvate carboxykinase
C. glucose-6-phosphatase
D. Fructose 1, 6 bisphosphatase
E. phosphoglycerate kinase
KEY: A, Pyruvate carboxylase
16. 36-48 hr. — Falciparum malaria
17. Fatty acid synthesis — acetyl coA carboxylase
18. Biceps shoulder can't touch — musculocutanoeus
19. A child travelling in car uphill having nausea and vomiting — Scopolamine
20. Hydrochlorothiazide side effect — Gout
21. Koplik spot — measles
22. Antimalarial for falciparum malaria — Artemether
23. Enalapril — Dry cough
24. Acute pancreatitis diagnose — serum amylase
25. Sulbuterol — Tremor/seizure
26. Benzodiazepines— Flumazenil
27. Doxycycline— Renal failure
28. Green light tag— Ambulatory
29. Tetracycline— Teeth discoloration
NEB/PMDC STEP-1 PAST PAPERS
DR.ASIF ALI KHAN
YANGTZE UNIVERSITY
30. A tissue graft between two people who are between genetically identical individuals is termed a:
A. Isograft
B. Heterograft
C. Xenograft
D. Autograft
E. Allograft
KEY: A, Explanation:
» An “isograft“is performed between genetically identical individuals (i.e. monozygotic twins).
» A “heterograft”is not a word that is used in transplantation immunology.
» A “xenograft” is a transplant that is performed across species.
» An “autograft” is a transplant from one location in the body to another.
» An allograft is a transplant between two members of the same species who are not
genetically identical.
31. Third pharyngeal pouch— thymus
32. Transitional epithelium—bladder
33. A male patient is taking cimetidine. Which adverse effect is more likely to occur with cimetidine than with
other histamine2 receptor antagonists?
A. Seizures
B. Gynecomastia
C. Hypoxia
D. Hypertension
E. Heart block
KEY: B,
34. Which of the following drugs is most likely to cause systemic lupus-like syndrome?
A. Isoniazid
B. Methotrexate
C. Procainamide
D. Sulfasalazine
KEY: C, Procainamide
35. Finger adduction and abduction is controlled by which of the following nerves?
A. Radial
B. Ulnar
C. Musculocutaneous
D. Median
E. Axillary
KEY: B
NEB/PMDC STEP-1 PAST PAPERS
DR.ASIF ALI KHAN
YANGTZE UNIVERSITY
36. A 35-year old man is brought to a Tehsil Headquarter hospital in southern Punjab following a road traffic
accident. He requires urgent blood transfusion and is transfused unscreened whole blood. His medical
record retrieved later reveals that he is a chronic carrier of Hepatitis B virus. Two weeks later the patient
dies of fulminant hepatic failure. What transfusion related incidence do you suspect most in this case?
A. ABO incompatible blood transfused
B. Rh incompatible blood transfused
C. Transmission of HDV
D. Transmission of HCV
E. Transmission of HGV
KEY: C
37. Pregnant lady — iron deficiency anemia
38. Pellagra— niacin
39. Sign of drowning— Ansa cutis
40. Leishmaniasis – Balochistan sandfly
41. Biguanide (Metformin) side effect— lactic acidosis
42. Fluorine – dental
43. Pregnant lady fell in washroom, severe bleeding — tertiary health unit
44. Bone marrow sample should be from iliac crest or sternum
45. In autopsy blood can be taken from— Femoral vein
46. Anthelminthic— Praziquantel
47. Mechanism of penicillin— transpeptidase cross link
48. SIADH causing hyponatremia in— Lung carcinoma
49. GFR increase by – decrease Afferent arterial resistance
50. Gynecomastia — Spironolactone
51. Hyperuricemia — Thiazide diuretic (2 questions)
52. Case of increase ICP — Mannitol
53. DM ketoacidosis— Regular insulin
54. Anti-thyroid — Agranulocytosis
55. Microcytic with splenomegaly—Thalassemia major
56. ALT — Viral hepatitis
57. Fatty liver changes— alcoholic hepatitis
58. Mallory bodies —Alcoholic hepatitis
59. After ATTB develops fever malaise and hemoptysis—Pyrazinamide
60. Alcoholic case scenario — Thiamine deficiency
61. Iron studies in blood — Ferritin
NEB/PMDC STEP-1 PAST PAPERS
DR.ASIF ALI KHAN
YANGTZE UNIVERSITY
62. gluten hypersensitivity — Anti-IGA transglutamic AB
63. Pseudomembranous colitis— Vancomycin
64. Case scenario of heart failure patient, drug for pulmonary edema — Furosemide
65. Which of the following agents is useful in treatment of
A. Baclofen
B. Diazepam
C. Cyclobenzaprine
D. Dantrolene
E. Halothane
KEY: D
66. A 50 years old non-smoker lady with alpha-1 antitrypsin deficiency developed severe lung disease and
requires a double lung transplant. What is the most likely lung lesion she is having?
A. Adenocarcinome
B. Bullous emphysema
C. Centriacinar emphysema
D. Interstitial emphysema
E. panacinar emphysema
KEY:
67. Tuft of hair neural tube defect— spina bifida
68. Neural tube defect having membrane covering outside — meningo
69. Facial nerve doesn’t supply Masseter muscle
70. Lateral ventricle with third ventricle with foramen monro
71. Patellar reflex — sciatic nerve
» The muscles involved in the Achilles tendon reflex are supplied by the tibial nerve (a branch of the sciatic
nerve), which is derived from the S1 and S2 nerve roots. The tibial nerve forms in the popliteal fossa,
and leaves it running inferiorly on the tibialis posterior muscle. It supplies the posterior muscles of the
leg and knee joint, and terminates by dividing into the medial and lateral plantar nerves.
72. The prophylactic treatment of malaria:
A. Mefloquine
B. Primaquine
KEY: B
73. Drug which belongs to class-IA anti-arrhythmic
A. Lidocaine
B. Propranolol
C. Procainamide
D. Amiodarone
KEY: C
NEB/PMDC STEP-1 PAST PAPERS
DR.ASIF ALI KHAN
YANGTZE UNIVERSITY
74. Erythroblastosis fetalis can be prevented if the mother is injected, at parturition, with an antibody called:
A. Blocking antibody
B. Rh0 (D) immunoglobulin (RhoGAM)
C. Antilymphocyte serum
D. Antithymocyte serum
E. Univalent antiserum
KEY: B
75. The surgical neck of the humerus is related to the:
A. Radial nerve.
B. Axillary nerve.
C. Ulnar nerve.
D. Median nerve.
KEY: B
76. Which one of the following nerve palsy may result in winging of the scapula?
A. Long thoracic nerve palsy
B. Suprascapular nerve palsy
C. Spinal accessory nerve palsy
D. Radial nerve palsy
KEY: A
77. What is the serious adverse effect of metformin?
A. Obesity
B. Lactic acidosis
C. Hypoglycemia
D. Hypercalcemia
KEY: B
78. A 54 year old female with an abdominal mass undergoes exploration laparoscopy. Both ovaries are
enlarged hence resected. Pathology report is Krukenberg tumor, indicating which of the following:
A. Ectopic pregnancy
B. Endometriosis
C. Hyperestrogenic state
D. Immunosuppression
E. Metastatic carcinoma
KEY: E
NEB/PMDC STEP-1 PAST PAPERS
DR.ASIF ALI KHAN
YANGTZE UNIVERSITY
79. Chills, fever, and muscle aches are common reactions to which one of the following antiviral drugs?
A. Acyclovir
B. Gancyclovir
C. Oseltamivir
D. Interferon
E. Ribavirin
KEY: D
80. Which of the following sulfonamide is used in the treatment of ulcerative colitis
A. Sulfadiazine
B. Sulfasalazine
C. Salfadimidine
D. Sulfadoxine
E. Metronidazole
KEY: B
81. Orthotolidine test is used to determine
A. Nitrates in water
B. Nitrites chlorine in water
C. Free and combined chlorine in water
D. Ammonia content in water
E. None
KEY: C
82. A mother brought her infant to the pediatrician for delayed milestones, hepatosplenomegaly and pallor.
Another sibling had similar problems for which repeated blood transfusions was advised. Confirmation of
the diagnosis can be made by:
A. Full blood profile
B. Iron studies
C. Hb electrophoresis
D. Bone marrow study
E. Biopsy
KEY: C
83. Antiemetic like effect— propo
84. Dissociative sedation— ketamine
85. A person died in police custody: magistrate
86. fasting plasma glucose _> 126
87. HbA1c _>6.5 to 7.5
88. Adenocarcinoma— Barrett’s esophagus
NEB/PMDC STEP-1 PAST PAPERS
DR.ASIF ALI KHAN
YANGTZE UNIVERSITY
89. Which of the following steroids can be administered for maturity of surfactant in preterm labour.
A. Beclomethasone
B. Betamethasone
C. Prednisolone
D. Hydrocortisone
KEY: B
90. A patient needs antibiotic treatment for culture-positive infective streptococcal endocarditis. His medical
history includes a severe anaphylactic reaction to penicillin G during the last year. The best approach
would be treatment with
A. Amoxicillin-clavulanate
B. Aztreonam
C. Penicillin G
D. Ticarcillin
E. Vancomycin
KEY: B
91. mRNA codon—AUG
92. Black guy with skin problem— ?fatty acid defect
93. Vitamin causing hypercreatinine— vit D?
94. Pulmonary HTN— Frusemide
95. Patient with HF present in emergency with dyspnea, rales on auscultation— Furosemide
96. Sublingual Anti anginal— Glyceryl trinitrate
97. Anti-peptic ulcer PPI—Omeprazole
98. Femur neck fracture, hx of long term drugs taking— Prednisolone
99. Asthmatic presents with tremors — Salbutamol
100. Long term use, seizures—Theophylline
101. Cromolyn Sodium
A. Is given by oral route
B. Produce bronchodilation in asthmatic patient
C. Act by preventing potassium efflux from the mast cells
D. Is useful for Prophylaxis of asthma
KEY: D
102. Loperamide — Opioid Derivative
NEB/PMDC STEP-1 PAST PAPERS
DR.ASIF ALI KHAN
YANGTZE UNIVERSITY
103. The most serious adverse reaction that may occur with the use of Thionamide antithyroid drugs is
A. Agranulocytosis
B. Joint pains
C. Paresthesias
D. Alopecia
E. Skin rashes
KEY: A
104. GIT motility — Domperidone
105. Drug which is used in the treatment of Bipolar Disorder
A. Fluoxetine
B. Benzodiazepine
C. Lithium
D. Clozapine
KEY: C
106. Extrapyramidal symptoms (EPS) have been associated with which of the following drugs?
A. Metoclopramide
B. Alprazolam
C. Aprepitant
D. Loperamide
KEY: A, Metoclopramide
107. Benzodiazepines Antidote— Flumazenil
108. fat emboli— long bone fracture
109. After criminal abortion— Hemorrhagic shock
110. Babinski positive— UMN and corticospinal tract damage
111. A teenager concerned about his weight, attempts to maintain a fat free diet for a period of several week.
If his ability to make lipids were examined, he would be found to be the most deficient in his ability to
make
A. Cholesterol
B. Glyco lipids
C. Phospholipids
D. Prostaglandins
E. Triacyl Glycerol
KEY: E
NEB/PMDC STEP-1 PAST PAPERS
DR.ASIF ALI KHAN
YANGTZE UNIVERSITY
112. A young girl with history of severe abdominal pain was taken to local hospital at 5 am in severe distress.
Blood was drawn the plasma appeared milky with TAG more than 2000 mg/dl (N = 4-150 mg/dl). The
patient was placed on diet limited on fat but supplements with medium chain TAG. Which of the
following protein is deficient in this patient.
A. Apo A-1
B. Apo B-48
C. Apo C-II
D. Cholestryl Ester transfer protein
E. Microsomal triglycerides transfer protein
KEY: C
113. A couple of American-African descent, after an otherwise uneven pregnancy, gives birth to an
exceptionally fair skinned baby boy with also white hair. Further examination reveals red pupils. A
postnatal screen likely to confirm the deficiency of the following enzyme
A. Glutathione reductase
B. Glutathione peroxidase
C. Tyrosinase
D. Methionine synthase
E. Cystathionine beta synthase
114. A 24 year old women presents with diarrhea, dysphagia, jaundice and white transverse line on fingernails
(Mee’s lines). The patient is diagnosed with arsenic poisoning which inhibit the following enzyme of TCA
cycle
A. Aconitase
B. Alpha ketoglutarate dehydrogenase complex
C. Citrate synthase
D. Isocitrate dehydrogenase
E. Succinate dehydrogenase
KEY: B, Arsenic is toxic at many levels due to its broad specificity of enzyme inhibition. The mode of action
of arsenic-containing compounds, primarily arsenic trioxide, is via it reacting with biological ligands that
possess available sulfur groups. Arsenic interferes with the activity of the TCA cycle by inhibiting the α-
ketoglutarate dehydrogenase complex, which requires the activity of the sulfhydryl group associated with
the dihydrolipoamide moiety of the enzyme complex.
115. A 40 years old male presented with chest pain that radiates to his left jaws and shoulder, he has
diagnosed with myocardial infarction and is prescribed a statin medication. Statin are competitive
inhibitors of HMG-CoA-Reducatse. Which convert HMG-CoA to the following
A. Cholesterol
B. Famesyl pyrophosphate
C. Geranyl pyrophosphate
D. Isopententyl pyrophosphate
E. Mevalomate
NEB/PMDC STEP-1 PAST PAPERS
DR.ASIF ALI KHAN
YANGTZE UNIVERSITY
116. Three days after a full-term normal delivery, a neonate vagina to hyperventilate, develops hypothermia
and cerebral edema and becomes comatose, urinalysis reveals high levels of glutamine and orotic acid. The
BUN is below normal. Which enzyme is most likely to be deficient in this child?
A. Cytoplasmic glutaminase
B. Cytoplasmic carbamoyl phosphate synthetase
C. Cytoplasmic orotidylate decarboxylase
D. Mitochondrial carbamoyl phosphate synthetase
E. Mitochondrial orinthine transcarbamoyolase
117. Screening of disease is a part of
A. Primary prevention
B. Secondary prevention
C. Tertiary prevention
D. All of the above
E. None of the above
KEY: B
118. Commonest cause of blindness in preschool children’s is
A. Maternal rubella
B. Vitamin A deficiency
C. Accidental injury
D. Congenital cataract
E. Trachoma neonatorum
KEY: B
119. Clavulanic acid is a beta-lactam antibiotic that
A. That easily penetrates gram negative microorganisms
B. Is specific for gram positive microorganisms
C. Is a potent inhibitor of cell transpeptidase?
D. Inactivates bacterial B-lactamases
E. Has a spectrum of activity similar to penicillin G
KEY: D
120. Secondary intension of wound healing differs from primary intention in which of the following aspects
A. Neutropenia accumulation
B. Mitoses
C. Fibrous reaction
D. Wound contraction
E. Clot formation
KEY: D
121. The following is inhibitor of prokaryotic transcription
A. Ciprofloxacin
B. Etoposide
C. Erythromycin
D. Rifampicin
E. Methotrexate
KEY: D
NEB/PMDC STEP-1 PAST PAPERS
DR.ASIF ALI KHAN
YANGTZE UNIVERSITY
122. The following subunit of bacterial RNA polymerase is responsible promoter recognition
A. Alpha
B. Beta
C. Beta prime
D. Sigma
E. Alpha prime
KEY
123. A specific form of necrosis, also visible under light microscope, which occur after deposition of immune
complexes in the walls of the arteries is
A. Coagulative necrosis
B. Caseous necrosis
C. Fibrinoid necrosis
D. Fat necrosis
E. Liquefactive necrosis
124. A 70 years old man presented in medical emergency with c/o severe chest pain with profuse sweating
and shortness of breath. He was diagnosed as a case of acute myocardial infarction. Which isoenzyme of
creatine kinase (CK) raised in this case
A. CK 1
B. CK 2
C. CK 3
D. CK MB
E. Both b and d
KEY: D
125. A known patient of haemophilia a being treated by factor VIII concentrates has stopped responding to
this treatment and his condition is worsening. the most likely explanation to this phenomenon is
A. Neutralizing antibodies against factor VIII
B. Low dose of factor VIII
C. Expired factor VIII
D. Wrong diagnosis
E. Infection
126. Over the counter medications containing bismuth are quite popular for managing diarrhea and several
other common GI maladies. However, the bismuth salt they contain is the subsalicylate. Given the
presence of salicylate, these should not be administered to children
A. Under 14 years old
B. With diarrhea lasting more than 18h
C. Having diarrhea with flatus
D. Child with flu, common cold, chickenpox
E. With otitis media
KEY
NEB/PMDC STEP-1 PAST PAPERS
DR.ASIF ALI KHAN
YANGTZE UNIVERSITY
127. Following system is in vogue in Pakistan
A. Coroner system
B. Medical examiner system
C. Continental system
D. Modified continental system
E. Proctor fiscal system
KEY: D
128. Cytological examination of FNA breast of 45 years old female showed groups of cells which were
dyscohesive and were showing dyspolarity, individually scattered cells with open chromatin and nuclear
membrane irregularity were also found. Most likely cytological category of lesion is:
A. Atypical probably benign lesion (C-3)
B. Benign breast lesion (C-2)
C. Malignant lesion (C-5)
D. Suspicious malignant lesion (C-4)
E. Smear inadequate (C-1)
KEY:
129. The drug most effective against malarial parasites in the liver but not effective against parasites within
erythrocytes is
A. Primaquine
B. Pyremithamine
C. Quinidine
D. Chloroquine
E. Chloroguanide
KEY: A
130. Four hours after abdominal surgery a 65 years old male was found to have some equivocal ECG changes
and markedly raised serum CK (>1500 U/L). which of the following cardiac markers would be the most
specific investigations to rule out myocardial infarction in this patient:
A. CK-MB activity measurement
B. CK-MB mass measurement
C. CK-MM activity measurement
D. LDH levels
E. Estimation of cardiac troponin T
KEY: D
131. Flow like syndrome is seen with:
A. Rifampin
B. Isoniazid
C. Streptomycin
D. Pyrazinamide
E. Ethambutol
KEY: B
NEB/PMDC STEP-1 PAST PAPERS
DR.ASIF ALI KHAN
YANGTZE UNIVERSITY
132. A 55 year old woman presents in OPD with inability to adduct and medially rotate her arm after right
mastectomy. Examination showed loss of function of latissmiss dorsi muscle. The nerve supplying in the
muscle derived from:
A. Lateral cord of brachial plexus
B. Medial cord of brachial plexus
C. Posterior cord of brachial plexus
D. Upper trunk of brachial plexus
E. Lumbar plexus
133. Septic shocks results from severe inflammatory response to bacterial infection. It has a high mortality
rate and is associated with changes in the level of nitric oxide. Which statement concerning septic shock
is mostly likely accurate
A. Activation of endothelial nitric oxide synthase cases an increase in NO
B. High mortality is the result of the long half-life of nitric oxide
C. Lysine, the nitrogen source of nitric oxide synthesis, is deaminated bacteria
D. Over production of nitric oxide by Ca – independent enzyme is the cause of the hypotension
E. High mortality is the result of short half-life of NO
KEY: A
134. Separation of skull suture in young persons by blunt trauma is classified as
A. Comminuted fracture
B. Diastatic fracture
C. Hinge fracture
D. Ring fracture
E. Signature fracture
KEY: A
135. A 26 year old female is using injectable Medroxyprogesterone as a method of contraception. Which of
the following adverse effects is a concern if she wishes to use this therapy long-term?
A. Hyperkalemia
B. Male pattern baldness
C. Osteoporosis
D. Weight gain
E. Pulmonary fibrosis
KEY: C
136. Myasthenia gravis is an autoimmune disease in which antibodies damage or destroy:
A. Acetylcholine molecules in the synaptic cleft
B. Acetylcholine receptors on the muscle membrane
C. Acetylcholine vesicles in the neuromuscular junction
D. Acetylcholinesterase molecules in the presynaptic terminal
E. Acetylcholinesterase molecules in the synaptic cleft
KEY: B
NEB/PMDC STEP-1 PAST PAPERS
DR.ASIF ALI KHAN
YANGTZE UNIVERSITY
137. The pap smear of patient showed enlarged cells having orarangophilic cytoplasm and central vascular
nucleus which was surrounded by perinuclear hallow. These cells are most likely indicative of
A. CIN-1
B. Invasive squamous cell carcinoma
C. Invasive adenocarcinoma of cervix
D. Invasive adenocarcinoma of endometrium
E. Koilocystic cells
138. A six-year-old child, whose medical history includes a rather difficult birth, has a permanently tilted head
posture, with the right ear near the right shoulder and the face turned upward and to the left. Which of
the following muscles was very likely damaged during birth?
A. Anterior scalene
B. Omohyoid
C. Sternocleidomastoid
D. Trapezius
E. Platysma
KEY: C
139. Which of the following antiemetics functioning through inhibition of neurokinin-1 (NK-1) receptor
A. Aprepitant
B. Domperidone
C. Granisteron
D. Ondansetron
140. Which of the following drugs is leukotriene-modifying drug indicated in the management of bronchial
asthma
A. Triamcinolone acetonide
B. Budesonide
C. Zafirleukast
D. Flunisolide
E. Fluticasone
KEY: C
141. There is inverse relationship between hardness of drinking water and
A. Diabetes mellitus
B. Cardiovascular disease
C. malignancy
D. Obesity
E. All of the above
KEY: B
NEB/PMDC STEP-1 PAST PAPERS
DR.ASIF ALI KHAN
YANGTZE UNIVERSITY
142. Vasodilation caused by
A. Parasympathetic stimulation
B. Sympathetic blockade
C. Localized hypoxia
D. Anaphylactic shock
E. Stimulation of chemoreceptors
143. A 56 year old female develop massive pulmonary embolism. The most likely chances that embolus must
be from
A. Deep calf veins
B. Femoral veins
C. Iliac veins
D. Pelvic veins
E. Portal veins
KEY: B
144. Surgical removal of adrenal tumour of a 47-year old man, is planned. It is anticipated that the procedure
will take approximately 16h. The patient has a history of hypertension controlled with a beta-blocker.
Which of the following agents, used intraoperatively, will provide efficacious blood pressure control for
the duration of the procedure
A. Acebutolol
B. Esmolol
C. Metoprolol
D. Nadolol
E. Pindolol
KEY: B
145. Two enzyme have similar V max, but enzyme 1 has lower Km as compared to enzyme 2, this means
A. Less substrate is required by enzyme 1 to attain V max/2
B. Enzyme 1 has more active sites
C. Enzyme 2 has lesser number of binding sites
D. Enzyme 2 has lesser number of regulatory sites
E. Enzyme 2 his slowed down by substrate
146. A 25 years old male suffering from high grade fever, rigors and severe headache. Examination of the
peripheral blood film reveals presence of rings in the red cells and crescent shaped gametes. What is the
most likely pathogen
A. Plasmodium vivax
B. Plasmodium malariae
C. Plasmodium ovale
D. Plasmodium falciparum
E. Plasmodium knowlesii
NEB/PMDC STEP-1 PAST PAPERS
DR.ASIF ALI KHAN
YANGTZE UNIVERSITY
147. A 50 years old patient suffering from chronic cough, weight loss and low grade fever is admitted with
probable diagnosis of tuberculosis. Immunity to tuberculous infection is primarily mediated by
A. DC4+ve T cells
B. CD8 + T cells
C. Macrophages
D. Neutrophils
E. B Cells
148. The most likely cause of intermittent diarrhea of 3weeks duration in a 26 year old male with space
occupying lesion in the right hypochondrium most likely is due to
A. Giardia Lamblia
B. Entamoeba histolytica
C. Balantidium coli
D. Entamoeba coli
E. Cryptosporidium parvum
149. Exposure to 4 atmosphere of oxygen (PO2 = 3040 mmhg) can cause d duration within an hour or so due
to acute oxygen poisoning. The lethal effect of acute oxygen poisoning can be attributed to the
dysfunction of which of the following agents
A. Brain
B. Heart
C. Kidney
D. Liver
E. Lungs
150. An aviator is flaying at 30,000 feet, where the barometric pressure is 226 mm hg. He is breathing 100%
oxygen. His alveolar PCO2 is 40mm hg, and alveolar water vapor pressure is 47 47 mm hg. What is the
alveolar PO2 of the aviator
A. 43 mm Hg
B. 75 mm Hg
C. 99 mm Hg
D. 139 mm Hg
E. 100 mm Hg
NEB/PMDC STEP-1 PAST PAPERS
DR.ASIF ALI KHAN
YANGTZE UNIVERSITY
23 JUNE 2019 NUMS
MCQS OF NEB PMDC STEP-1
1. Histopathological examination of lymph nodes of 18 years old diagnosed patient of Hodgkin’s lymphoma,
involving supraclavicular and lower cervical lymph nodes, showed a lacunar variant of Reed-sternburg
cells. In which sub type of Hodgkin’s lymphoma this variant is found.
A. Nodular sclerosing H.L
B. Mixed cellularity H.L
C. Lymphocytic predominance H.L
D. Lymphocytic rich H.L
KEY: A
2. An 83 years old female has a biopsy of an ulcerated nipple lesion that is interpreted as Paget’s disease. A
biopsy of the breast tissue with an ulcerated lesion on the nipple diagnoses to be Paget’s disease will
most likely show the following disease:
A. Acute mastitis
B. Ductal carcinoma in situ
C. Intraductal papilloma
D. Invasive lobular carcinoma
KEY: B
3. X-ray of a patient show skull thickening with narrowing of foramina and the proximal humerus- typical
sun burst or sun ray appearance. There is bowing of femur and tibia as well and raised alkaline
phosphates. Bone biopsy reveals mosaic pattern of bone spicules with prominent osteoid seams. Which
neoplasm will it be?
A. Astrocytoma.
B. Osteosarcoma.
C. Meningioma.
D. Hodgkin's lymphoma
KEY: B
4. A person with endemic goiter has
A. High level of Plasma TSH
B. High level of Plasma thyroxine
C. High level of Plasma triiodothyronine
D. Low production of thyroglobulin
E. Low level of growth hormone
KEY: A, High level of Plasma TSH (Decreased T3, T4 and Increased TSH)
NEB/PMDC STEP-1 PAST PAPERS
DR.ASIF ALI KHAN
YANGTZE UNIVERSITY
5. A 60-year old businessman returns to Pakistan after a 2-month long trip to the Middle East. A week later
he presents with gross hematuria. Urine examination shows pear shaped ova with large terminal spine.
Which of the following is most likely responsible for this patient’s symptoms?
A. Schistosoma haematobium
B. Schistosoma Japonicum
C. Schistosoma mansoni
D. Wuchereria Bancrofti
KEY: A
6. A 42 years old obese women presented to the emergency room with complain of worsening nausea,
vomiting and epigastric pain. The blood chemistry revealed a high Amylase level. Most likely diagnosis is:
A. Acute appendicitis
B. Acute Pancreatitis
C. Gastritis
D. Renal colic
E. Viral hepatitis
KEY: B,
7. After prolonged active hepatitis C, a 50 years old patient developed ascites and esophageal varices. These
complication are secondary to:
A. Cholangitis
B. Hepatocellular carcinoma
C. Pancreatitis
D. Cirrhosis
KEY: D
8. A women smokes one pack/day, she drinks at least 5 cups of coffee/day. Her mother died of breast
cancer. Which is the greatest risk factor for breast cancer in this patient
A. Birth control pills
B. Cigarette smoking
C. Family history
D. Caffeine intake
KEY: C
9. A 33 year old female presented with pain abdomen for 3-6 months, low grade fever and lethargy. On
ultrasound examination right ovary was four to be enlarged and was removed for examination. On
microscopy inflammation with epithelioid cells, Langerhans’s giant cells and caseous necrosis was seen.
The most likely diagnosis is:
A. Mumps
B. Syphilis
C. Tuberculosis
D. Sarcoidosis
KEY: C
NEB/PMDC STEP-1 PAST PAPERS
DR.ASIF ALI KHAN
YANGTZE UNIVERSITY
10. Deposition of calcium in nonviable or dying tissue in the presence of normal serum calcium levels is called
A. Metastatic calcification
B. Dystrophic calcification
C. Intracellular calcification
D. Calcium excess
E. None of the above
KEY: B
11. A 15-year-old girl is brought to the physician by her parents, who are concerned because she has
developed multiple nodules on her skin. On physical examination, there are 20 scattered, 0.3- to 1-cm,
firm nodules on the patient's trunk and extremities. There are 12 light brown macules averaging 2 to 5 cm
in diameter on the skin of the trunk. Slit-lamp examination shows pigmented nodules in the iris. These
findings are most likely to be associated with which of the following types of neoplasm?
A. Dermatofibroma
B. Leiomyoma
C. Neurofibroma
D. Lipoma
KEY: C, The patient has neurofibromatosis type 1 (NF-1), characterized by the development of multiple
neurofibromas and pigmented skin lesions. Neurofibromas are most numerous in the dermis but also may
occur in visceral organs. Patients with NF-1 also may develop a type of sarcomatous neoplasm known as a
malignant peripheral nerve sheath tumor. Dermatofibromas are subcutaneous masses that typically are
small and solitary. Leiomyomas occur most frequently in the uterus. Lipomas can occur almost anywhere in
the body, but do so sporadically. Hemangiomas may occur sporadically on the skin; they typically are red-
blue masses.
12. A 70-year-old Negroid man from Makkran coast was stabbed on the upper arm, he received emergency
treatment and his wound healed. He developed a raised hypertrophic scar with boundaries beyond the
original wound, and it did not regress. Which of the following terms best describes this condition:
A. Cicatrix
B. Keloid
C. Callus
D. Granulation tissue
KEY: B
13. Which of the following arrhythmia is most commonly associated with syncope
A. Sinus arrhythmia
B. First degree heart block
C. Second degree heart block
D. Third degree heart block
KEY: B
NEB/PMDC STEP-1 PAST PAPERS
DR.ASIF ALI KHAN
YANGTZE UNIVERSITY
14. Which of the following corticosteroids is most appropriate to administer to a women in preterm labor to
accelerate fetal lung maturation.
A. Beclomethasone
B. Betamethasone
C. Prednisolone
D. Hydrocortisone
KEY: B
15. A 55 year old women stop menstruating approximately 3 months ago. Worried that she is maybe pregnant,
she decided to have a pregnancy test. The test came back negative. Which of the following series of tests
results would confirm that the women is post-menopausal
A. Decreased LH, decreased FSH, increased estrogen
B. Decreased LH, increased FSH, decreased estrogen
C. Increased LH, decreased FSH, decreased estrogen
D. Increased LH, increased FSH, decreased estrogen
KEY: D, FSH and LH can no longer perform their usual functions to regulate estrogen, progesterone and
testosterone. These inevitable changes in hormones and natural decline of estrogen levels causes
menopause
16. PTH
A. Acts directly on bone cells to increase calcium deposition
B. Acts indirectly on intestinal cells to increase calcium absorption
C. Causes decrease in cAMP concentration within renal proximal tubular cells
D. Is secreted in response of an increase in plasma calcium concentration
KEY:
17. The part of male reproductive tract which carries only semen within prostate gland is the
A. Prostatic urethra
B. Seminal vesicles
C. Ductus deferens
D. Ejaculatory duct
KEY: D, Ejaculatory duct-the ejaculatory duct is a duct which courses through the prostate gland and
contains only semen. Remember, semen is the combination of sperm from the ductus deferens, seminal
fluid from the seminal vesicle, and secretions of the prostate gland. The ejaculatory duct is formed by the
union of the duct of the seminal vesicle and the ampulla of the ductus deferens, and it is the site where
sperm and seminal fluid mix. The prostatic urethra is also contained in the prostate gland, and it carries
semen, but it also carries urine out of the bladder. The membranous urethra is the continuation of the
prostatic urethra outside of the prostate gland, and it carries both semen and urine as well. The seminal
vesicle is a structure on the posterior surface of the bladder that produces seminal fluid. The ductus
deferens is a passageway that carries sperm from the epididymis to the ejaculatory duct.
NEB/PMDC STEP-1 PAST PAPERS
DR.ASIF ALI KHAN
YANGTZE UNIVERSITY
18. The structure passing through the aortic opening of diaphragm
A. Accessory nerve
B. Esophagus
C. Superior vena cava
D. Azygous Vein
KEY: D, Azygous Vein
19. Cytotoxic T-cells help in
A. Destroying tissue invaded by viruses
B. Activation of mast cells
C. Regulating T-cell activity
D. Suppressing the T-cell activity
KEY: A
20. A 43 year old women came with large abscess in the middle of right post triangle of the neck. The
physician incised and drained the abscess. Five days later patient noticed that she could not extend her
right hand above her head to brush her hair. Which of the following structure is damaged
A. Long thoracic nerve
B. Suprascapular nerve
C. Accessory nerve
D. Dorsal scapular nerve
KEY: C, Accessory nerve (spinal accessory nerve)
21. Knee jerk nerve involvement — sciatic nerve ✅
» The muscles involved in the Achilles tendon reflex are supplied by the tibial nerve (a branch of the sciatic
nerve), which is derived from the S1 and S2 nerve roots. The tibial nerve forms in the popliteal fossa,
and leaves it running inferiorly on the tibialis posterior muscle. It supplies the posterior muscles of the
leg and knee joint, and terminates by dividing into the medial and lateral plantar nerves.
22. A Case scenario of and asked about association of CNS axon - Multiple Sclerosis is a disease characterized
by preferential damage to myelin that becomes detached from axon in the central nervous system.
Which associated cells are involved?
A. Oligodendrocytes.
B. Microglia.
C. Protoplasmic astrocytes.
D. Satellite cells.
KEY: A
23. Premature closure of the sagittal suture
A. Anencephaly
B. Scaphocephaly
C. Brachycephaly
D. …..
KEY: B
NEB/PMDC STEP-1 PAST PAPERS
DR.ASIF ALI KHAN
YANGTZE UNIVERSITY
24. A patient who has suffered severe chest trauma in an automobile accident is found to have fluid in the
right pleural space. A thoracentesis reveals the presence of chylous fluid in the pleural space, suggesting
a rupture of the thoracic duct. In which regions of the thorax is the thoracic duct found.
A. Anterior and middle mediastinum
B. Anterior and superior mediastinum
C. Middle and superior mediastinum
D. Posterior and superior mediastinum
KEY: D, Posterior and superior mediastinum
25. During a full workup on a 2-month old with a history of intermittent gastrointestinal pain and vomiting,
physician discovered that the cause was lack of emptying of the stomach. They immediately suspected
that the use was spasmodic contraction of which of the following parts of the stomach
A. Cardiac notch
B. Fundus
C. Lessar curvature
D. Pylorus
KEY: D
26. Neonatal tetanus is an important preventable cause of neonatal mortality that account for 7% of
neonatal deaths worldwide. This could be prevented by:
A. Vaccination of women in reproductive age group
B. Vaccination of newborn’s
C. Vaccination of pregnant ladies
D. Vaccination of females of child bearing age
E. Vaccination of non-pregnant females
KEY: A, NOT CONFIRMED
27. A mother bought her six months old child to a BHU. She is worried about the growth of her child. The best
single measure for assessing the physical growth in this age
A. Weight for age
B. Height for age
C. Chest circumference
D. Mild upper arm circumference
KEY: A, Weight for age
28. In study carried out in the hospital ward, every 10th admitted patient was included in the sample, which
sample procedure is this
A. Random sampling
B. Stratified sampling
C. Quota sampling
D. Systemic sampling
KEY: D
NEB/PMDC STEP-1 PAST PAPERS
DR.ASIF ALI KHAN
YANGTZE UNIVERSITY
29. Secular trend ✅? Cyclic trend?
» Secular trends are those trends which describe the change in occurrence of a disease over prolonged
periods, usually years. It depends on the multiple factors such as degree of immunity, socioeconomic
status and nutritional levels in the population.
30. Hospital waste is best disposed off by
A. Composting
B. Dumping
C. Incineration
D. Burial
KEY: C, Incineration
31. Equity in health service means
A. Provision of health services on equal basis
B. Provision of health services according to the demands
C. Provision of health services according the needs
D. Provision of health services according to the resources
KEY: C
32. A term infant is born and does well with breast feeding. Two days later his mother calls frantically
because the baby is bleeding from umbilicus as nostrils. The following vitamin is most probably deficient
in this infants:
A. Vitamin A
B. Vitamin D3
C. Vitamin E
D. Vitamin K
KEY: E
33. Pap-smear is graded as cervical intra-epithelial neoplasia, Grade II (CIN II). Which of the following viruses
is related to this neoplastic growth?
A. Epstein Barr virus.
B. Hepatitis B virus.
C. Human papilloma virus.
D. Human herpes virus 8
KEY: C
34. Which of the following is a common compound shared by the TCA cycle and the Urea cycle?
A. α- Keto glutarate
B. Succinyl co A
C. Oxalo acetate
D. Fumarate
KEY: D
NEB/PMDC STEP-1 PAST PAPERS
DR.ASIF ALI KHAN
YANGTZE UNIVERSITY
35. Aortic dissection occurs in a tall 31 years old male with long fingers. His ophthalmic examination reveals
ocular lens dislocation. These features are found in
A. Down’s syndrome
B. Glycogen storage disease
C. Neurofibromatosis
D. Marfan syndrome
KEY: D
36. A 40-year old woman complains of deceased energy, significant weight gain and cold intolerance. She is
seen by her family physician, who has diagnosed her to be having hypothyroidism (Low level of thyroid
hormone). Which of the following is a precursor for thyroid hormone?
A. DOPA
B. Glutamine
C. Tyrosine
D. Tryptophan
KEY: C
37. A 55-year-old man suffers from cirrhosis of liver. Toxins such as ammonia are not properly metabolized
by the liver and can damage brain. Which of the following compounds is expected to be in highest
concentration in brain as a result of detoxification of ammonia?
A. Alpha keto glutarate
B. Glutamate
C. Glutamine
D. GABA
E. Asparagine
KEY: C
38. The predominant effect of Glucocorticoids on the circulating white blood cells is
A. Increase in T lymphocytes
B. Increase in monocytes
C. Increase in basophils
D. Increase neutrophils
KEY: D
39. A 68 year old male has COPD with moderate airway obstruction, Depsite using salmeterol twice daily as
prescribed, his reports continued symptoms of shortness of breath with mild exertion. Which one of the
following agents would be an appropriate addition to his current therapy?
A. Systemic corticosteroids
B. Montelukast
C. Tiotropium
D. Oral theophylline
Key: C – Tiotropium
NEB/PMDC STEP-1 PAST PAPERS
DR.ASIF ALI KHAN
YANGTZE UNIVERSITY
40. A 22-year old man, who frequently backpacks, complains of diarrhea and fatigue. Examination of stool
specimens shows bi-nucleate organisms with four flagellae. Which one of the following drugs would be
effective in treating this patient’s infestation
A. Metronidazole.
B. Quinidine.
C. Pentamidine.
D. Sulfadoxine.
KEY: A
41. A 32 year old male presents to an outpatient clinic with a 5-day history of productive cough, purulent
sputum, and shortness of breath. He is diagnosed with community Acquired pneumonia (CAP). It is noted
that this patient has a severe Ampicillin allergy (anaphylaxis). Which of the following would be an
acceptable treatment for this patient
A. Levofloxacin
B. Ciprofloxacin
C. Penicillin-V
D. Nitrofurantoin
KEY: A – Streptococcus pneumoniae is a common cause of CAP, and the respiratory fluoroquinolones
levofloxacin and Moxifloxacin provide good coverage. Ciprofloxacin does not cover S. pneumoniae well and
is a poor choice for treatment of CAP. Penicillin would be a poor choice due to allergy. Nitrofurantoin has
no clinical utility for respiratory tract infections.
42. A 56 years old female present with vomiting, diarrhea, headache, vertigo, tinnitus, her temperature is
98.8o
F, BP 106/60mmHg, PR is 56, P/E skin rashes and photosensitivity etc — Quinine ✅
» This patient presents with symptoms of cinchonism, a syndrome caused by the ingestion of derivatives
of cinchona bark such as quinine and quinidine. The key features of cinchonism include tinnitus (and
often hearing loss), blurred vision, headache, vomiting, and diarrhea. Confusion, delirium and psychosis
may occur. Quinidine toxicity will also cause bradycardia and hypotension, and a lichenoid
photosensitivity reaction is possible.
43. A 60-year-old man is admitted to the hospital with acute heart failure and pulmonary edema. Which of
the following drugs would be most useful in treating the pulmonary edema?
A. Digoxin
B. Dobutamine
C. Furosemide
D. Spironolactone
KEY: C
NEB/PMDC STEP-1 PAST PAPERS
DR.ASIF ALI KHAN
YANGTZE UNIVERSITY
44. A 42-year-old woman presents to the emergency department with fevers and headaches for the past 3
days. The fevers are off and on but are usually at least 102.5°F. Her headaches usually occur at the same
time as the fevers. Splenomegaly is noted on examination. Peripheral blood smear confirms the presence
of Plasmodium vivax. What is the most appropriate treatment —Primaquine (Tertian malaria)
» Primaquine is added to the regimen for treatment of Plasmodium vivax and Plasmodium ovale to
eradicate the dormant stages in the liver. Without Primaquine, once the blood stream infection is
cleared, it is likely that recurrence will still occur. However, Primaquine only will not cure malaria, so
both Primaquine and quinine is usually prescribed.
45. A 66-year-old woman with a history of Cushing’s disease treated with oral glucocorticoids presents to her
primary care physician for follow-up. She was recently hospitalized for a hip fracture following a fall, the
most common side effect of prolong use of glucocorticoid. — Osteoporosis ✅
» Osteoporosis is the most common adverse effect because of the ability of glucocorticoids to suppress
intestinal Ca2+
absorption, inhibit bone formation, and decrease sex hormone synthesis. Alternate-day
dosing does not prevent osteoporosis. Patients are advised to take calcium and vitamin D supplements.
Drugs that are effective in treating osteoporosis may also be beneficial.
46. Drug combination of Sulbactam + with
A. Ciprofloxacin
B. Amoxicillin
C. Erythromycin
D. Piperacillin
KEY: B, Sulbactam + Amoxicillin
Available antibiotics drug combination in Pakistan
1. Ampicillin + Cloxacillin Ampiclox (GSK)
2. Amoxicillin + Sulbactam Inj. Bactamox plus (Bosch)
3. Amoxicillin + Clavulanic acid Augmentin (GSK)
4. Ampicillin + Sulbactam Inj. Unasyn (Pfizer/GSK)
5. Sulbactam + Cephalosporin group
(Ceftriaxone, Ceftazidime,
cefoperazone, Cefepime)
6. Sulbactam + cefoperazone Inj. Cebac (Bosch) / Inj. xorbact (highnoon)
7. Piperacillin + Tazobactam Inj. Tazocin EF 4.5mg (Pfizer)
8. Ticarcillin + Clavulanic acid Inj. Timentin 3.1g (GSK)
47. Chills, fever, and muscle aches are common reactions to which one of the following antiviral drugs
A. Acyclovir
B. Gancyclovir
C. Oseltamivir
D. Interferon
KEY: D
NEB/PMDC STEP-1 PAST PAPERS
DR.ASIF ALI KHAN
YANGTZE UNIVERSITY
48. Calcium magnesium inhibit the absorption of which drug in intestine
A. Tetracycline
B. Ciprofloxacin
C. Benzyl penicillin
49. Which of the following sulfonamide is used in the treatment of ulcerative colitis
A. Sulfadiazine
B. Sulfasalazine
C. Salfadimidine
D. Sulfadoxine
KEY: B
50. We have a patient with essential hypertension and unusually high circulating catecholamine level. Our
goal is to block α- and β-adrenergic receptors using just one drug. Which of the following is capable to do
that
A. Labetalol
B. Metoprolol
C. Nadolol
D. Pindolol
KEY: A - Labetalol (Alpha + Beta blocker)
51. A 12 year old girl with a childhood history of asthma complained of cough, dyspnea, and wheezing after
visiting a riding stable. Her symptoms became so severe that her parents brought her to the emergency
room. Which of the following is the most appropriate drug to rapidly reverse her bronchoconstriction
A. Inhaled fluticasone
B. Inhaled Beclomethasone
C. Inhaled albuterol
D. Oral theophylline
Key: C, Inhaled albuterol - inhalation of a rapid-acting B2 agonist, such as albuterol, usually provides
immediate bronchodilation.
52. Whiplash is which form of injury
A. Hyper flexion
B. Hyperextension
C. Lateral flexion
D. Atlanto-axial dislocation
KEY: B
NEB/PMDC STEP-1 PAST PAPERS
DR.ASIF ALI KHAN
YANGTZE UNIVERSITY
53. During medico-legal autopsy, bullet is picked with
A. Toothed forceps
B. Hand
C. Rubbed ended forceps
D. Scissors
KEY: C
54. Bleeding from nose and ears is commonly seen at autopsy in
A. Hanging
B. Lynching
C. Strangulations by ligature
D. Smothering
KEY: C
55. Under the Pakistan Panel code, the minimum age of criminal responsibility is
A. 18 Years
B. 13 Years
C. 7 Years
D. 5 Years
KEY: C, 7 Years, The age of full criminal responsibility is 18 years
56. A 10 year old develops a bruise on his arm after a fall, which changes its color to yellow to brown a few
days later due to accumulation of:
A. Lipofuscin
B. Bilirubin
C. melanin
D. Hemosiderin
KEY: D
57. Dr. sana is operating a patient with myocardial infarction, she will prefer following structure for the
bypass graft
A. Great saphenous vein
B. Small saphenous vein
C. Profundafemoris vein
D. Popliteal vein
KEY: D
NEB/PMDC STEP-1 PAST PAPERS
DR.ASIF ALI KHAN
YANGTZE UNIVERSITY
58. A 26 years old female patient, at 3rd day of delivering a baby developed cyanosis, shortness of breath
and orthopnea. What is the most likely underlying pathology causing these signs and symptoms
A. Fat embolism
B. Amniotic Fluid Embolism
C. Air embolism
D. Myocardial infarct
KEY: B
59. A girl receiving treatment for asthma developed muscle tremors, which drug may be responsible
A. Salbutamol
B. Beclomethasone
C. Cromolyn sodium
D. Montelukast
KEY: A
60. Pulmonary surfactant is formed by:
A. Goblet cells
B. Type I alveolar cells
C. Type II alveolar cells
D. Alveolar macrophages
KEY: C
61. Which of the following is the risk of digoxin toxicity
A. Hypernatremia
B. Hypokalemia
C. Hyperkalemia
D. Hyponatremia
KEY: B
62. When the respiratory drive for increased pulmonary ventilation becomes greater than normal, a special
set of respiratory neurons, that are inactive during normal quiet breathing, then become active
contributing to the respiratory drive. These neurons are located in which of the following structures?
A. Apneustic center
B. Nucleus of the tractus solitaries
C. Pneumotaxic center
D. Ventral respiratory group
KEY: D
NEB/PMDC STEP-1 PAST PAPERS
DR.ASIF ALI KHAN
YANGTZE UNIVERSITY
63. The oesophageal opening in the diaphragm is at
A. T6
B. T8
C. T10
D. T12
KEY: C, Esophageal opening at T10 - It is located in the right crus, one of the two tendinous structures that
connect the diaphragm to the spine. Fibers of the right crus cross one another below the hiatus. It is
located approximately at level of the tenth thoracic vertebra (T10).
64. The isthmus of the thyroid gland
A. Lies directly anterior to the thyroid cartilage
B. Lies directly anterior to the cricoid cartilage
C. Lies directly anterior to the 2nd, 3rd, and 4th Tracheal rings
D. Contains the superior pair of parathyroid glands
KEY: C
65. What exits from the Stylomastoid foramen
A. Chorda tympanic nerve
B. Accessory nerve
C. Facial nerve
D. Glossopharyngeal nerve
KEY: C
66. A 22-year-old woman who is a professional tennis player presents to the physician with pain on lateral
aspect of her elbow radiating down her forearm. Repetitive use of which of the following muscle most
likely lead to this patient’s condition?
A. Biceps brachii
B. Extensor carpi radialis
C. Extensor carpi ulnaris
D. Flexor carpi ulnaris
KEY: B, Due to repetitive use of Extensor like flexor carpi Radialis (due to Backhand shots), he developed
lateral Epicondylitis. *However note that both (medial, lateral) overuse injuries can be idiopathic.
67. The fetal period extends from:
A. 12th
week to term
B. 3rd
week to 8th
week
C. 4th
week to term
D. 9th
week to term
KEY: D
NEB/PMDC STEP-1 PAST PAPERS
DR.ASIF ALI KHAN
YANGTZE UNIVERSITY
68. The secretion of growth hormone is increased by
A. Hyperglycemia
B. Exercise
C. Somatostatin
D. Hypothermia
KEY: B
69. If a patient with metabolic alkalosis
A. HCO3 of arterial blood is decreased
B. Kidney retained bicarbonate
C. Urine becomes alkaline
D. H+
is secreted from blood to rental tubules
E. H+
is excreted in urine
KEY: A
70. Anticholinergic drugs may be used in pre anesthetic medication to:
A. Slow down the gastric emptying
B. Dilate the pupils
C. Prevent bradycardia
D. Cause bronchodilation
E. Reduces peristalsis
KEY: A
71. Which of the following is 1st generation antihistamine
A. Promethazine
B. Loratidine
C. Fexofenadine
D. Levocetrizine
KEY: A
72. A 68-year-old women is diagnosed as a case of ovarian cancer. She begins using Cisplatin but becomes
nauseous and suffers from severe vomiting. Which of the following medication would be most effective
to counteract the emesis in this patient:
A. Droperidal
B. Dimenhydramine
C. Prochlorperazine
D. Ondensetron
KEY: D, DOC FOR Chemotherapy induce vomiting is Ondansetron
NEB/PMDC STEP-1 PAST PAPERS
DR.ASIF ALI KHAN
YANGTZE UNIVERSITY
73. A 45-year-old man was just started on therapy for hypertension and developed a persistent, dry cough.
Which is most likely responsible for this side effect
A. Enalapril
B. Losartan
C. Nifedipine
D. Prazosin
KEY: A
74. A 40 years old female diagnosed with short bowel syndrome in which, fat cannot be properly absorbed
so long-chain fatty acid are mobilised from adipose tissue to generate energy, the following is the
substrate from fatty acid oxidation
A. Long chain fatty acid
B. Fatty acyl carnitine
C. Fatty acyl coA
D. Beta-hydroxyacyl CoA
E. Acetyl CoA
KEY:
75. Which of the following organism is responsible of causing infective endocarditis after skin infection
A. S. Aureus
B. Strep. Viridians
C. Haemophilus
D. E.coli
KEY: A
76. A registered medical practitioner attempted pregnancy of a 35 35 years old women because she did not
want another child. However the woman died due to complication of the procedure. In Pakistan, the
medical practitioner would be guilty of the offense of:
A. Medical negligence
B. Qatl-e-Khata
C. Qatl-e-Shibh-Amd
D. Qatl-e-Bisabab
E. Isqat-e-Hamal
24. A teenager, concerned about his weight, attempts to maintain a fat-free diet for a period of several
weeks. If his ability to synthesize various lipids were examined, he would be found to be most deficient in
his ability to synthesize:
A. cholesterol
B. phospholipids
C. prostaglandins
D. triacylglycerol
KEY: C, Prostaglandins
NEB/PMDC STEP-1 PAST PAPERS
DR.ASIF ALI KHAN
YANGTZE UNIVERSITY
77. Which of the following gram negative bacilli mostly colonize healthy GI tract as non-pathogenic
organisms
A. Campylobacter enterocolitis
B. Shigella
C. Salmonella
D. Escherichia colli
KEY: D
78. The amino acid that undergoes oxidative deamination at a highest rate is
A. Arginine
B. Urea
C. Asparagine
D. Glutamine
KEY: D
79. In hyperammonemia which one of the following is also elevated in blood
A. Glutamine
B. Arginine
C. Urea
D. Asparagine
KEY: A
80. A lady gives birth to a baby with small abnormally shaped ears, mandibular hypoplasia, cleft Palate and
heart defects. She gives the history of treatment for cystic acne throughout the pregnancy. Which
chemical gents is responsible for the above mentioned abnormalities in the baby?
A. Lead
B. Valproic acid
C. Lithium
D. Vitamin A
KEY: D
81. Addition of an epinephrine to a local anesthetic results in
A. Early onset of local anesthesia
B. Rapid metabolism of local anesthesia
C. Prolonged duration of local anesthesia
D. Increased toxicity of local anesthesia
KEY: C
NEB/PMDC STEP-1 PAST PAPERS
DR.ASIF ALI KHAN
YANGTZE UNIVERSITY
82. You are caring for a patient who has suffered a stroke and has issue comprehending any spoken
language. Branches of what artery do you know have been compromised?
A. Posterior cerebral artery
B. Middle cerebral artery
C. Anterior cerebral artery
D. Anterior choroidal artery
KEY: B
83. A 15 years old boy is brought to emergency after a fall from his bicycle x-ray reveals fracture of neck of
fibula. On physical examination the patient a foot drop. Which of the following nerves is most likely
injured
A. Tibial
B. Common fibular (peroneal) nerve
C. Superficial fibular (peroneal) nerve
D. Deep fibular (peroneal) nerve
KEY: B
84. The middle nasal concha is part of what bone?
A. Ethmoid bone
B. Maxilla
C. Palatine bone
D. Sphenoid bone
KEY: A
85. A 40 years old female developed a tumour, which was labeled as grade-2 on grading. This grading of his
lesion is most likely based on:
A. Cell size & mitoses
B. Cell shape & mitoses
C. Degree of differentiation and mitosis
D. Nuclear Pleomorphism
KEY: C, Staging is by size of tumor but grading is by differentiation
86. Which of the following values are the best Indexes of the preload on Heart?
A. Blood volume
B. Pulmonary capillary wedge pressure
C. Left ventricular end-diastolic volume
D. Left ventricular end-diastolic pressure
KEY: C
NEB/PMDC STEP-1 PAST PAPERS
DR.ASIF ALI KHAN
YANGTZE UNIVERSITY
87. While playing football a person falls on the ground because of foot abduction of the tibia on femur. Dr.
Sheharyar is of the opinion that ligamentum has been injured. The most likely structure is:
A. Lateral collateral ligament
B. Capsule of Knee joint
C. Medial collateral ligament
D. Ligament patellae
KEY: A
88. Antianginal drug administered by inhalation:
A. Isosorbide dinitrate
B. Amyl nitrite
C. Glyceryl trinitrate
D. …
KEY: B, Amyl nitrate – Anti-anginal effects of nitrites and nitrates are usually due to decrease preload leading
to decreased ventricular wall tension (decreased oxygen demand)
» Isosorbide dinitrate: sub-lingual or chewed
» Glyceryl trinitrate: sub-lingual
89. A 45-year-old chronic alcoholic male has been brought to medical emergency in a semi-conscious state.
Blood lactate level is high. Which of the following vitamins can be used as a part of the treatment?
A. Vitamin C
B. Vitamin B12
C. Thiamine
D. Folic acid
KEY: C
90. Pantothenic acid is important for which of the following steps or pathways?
A. Pyruvate carboxylase
B. Fatty acid synthesis
C. Gluconeogenesis
D. Glycolysis
KEY: B
91. Activation of the parasympathetic nervous system resulting
A. Vasoconstriction
B. Broncho-dilation
C. Bradycardia
D. Relaxation of GI tract
KEY: C
NEB/PMDC STEP-1 PAST PAPERS
DR.ASIF ALI KHAN
YANGTZE UNIVERSITY
92. Which of the following vitamins is an antioxidant action
A. Vitamin B6
B. Vitamin E
C. Vitamin D
D. Cyanocobalamin
KEY: B
93. Osteonic canals communicating with the medullary cavity obliquely transversely are known as
A. Volkmann’s canals
B. Haversian canals
C. Canaculli
D. Interstitial lamellae
KEY: A
94. Case scenario in patient with increased bleeding time, what could be the cause of bleeding?
A. Prothrombin deficiency
B. Hemophilia – A
C. Vitamin – K deficiency
D. Von Willebrand’s disease
KEY: D
95. Case scenario of Patient with bone pains having normal Ca, increase alkaline phosphatase. Most likely
suffering from:
A. Paget’s disease
B. Hyper PTH
C. Hyper Vit D
D. Osteomalacia
KEY: A
96. Calories intake during pregnancy and lactation
A. 200-400
B. 350-600
C. 650-100
D. >1000
KEY: B?
97. A transudative fluid in edema
A. Rich in proteins
B. Rich in neutrophils
C. Rich in albumin
D. Low in proteins
KEY: D
NEB/PMDC STEP-1 PAST PAPERS
DR.ASIF ALI KHAN
YANGTZE UNIVERSITY
98. A 70-year-old man has difficulty rising from a seated position and straightening his back, but he has no
difficulty flexing his hip and knee. Which one of the following muscles is most likely to be weak?
A. Hamstrings
B. Obturator internus
C. Gluteus maximus
D. Gluteus minimus
KEY: C
99. In Severe starvation energy comes from
A. Amino acid
B. …
C. …
D. …
KEY: A
100. The nerve between C5, C6 is
A. C6
B. C5
C. C4
D. C2
KEY: A
101. Tall-tented T waves and widened QRS are seen in:
A. Hyponatremia
B. Hyperglycemia
C. Hyperkalemia
D. Hyperphosphatemia
KEY: C
102. Regarding the CNS
A. The tentorium cerebelli divided the right and left of the cerebellum
B. The temporal lobe the middle cranial fossa
C. …
D. …
103. With regard to fetal circulation:
A. Fetal umbilical vein has higher PO2 than maternal vein
B. Fetal umbilical vein has higher PO2 than fetal umbilical artery
C. Foetal haemoglobin has a lower affinity for O2 than which increases delivery to foetal tissues.
D. The stroke volume of the left ventricle is the same as the stroke volume of the right ventricle.
KEY: B, Umbilical Vein PO2 30mmHg > Umbilical Artery PO2 20mmHg
NEB/PMDC STEP-1 PAST PAPERS
DR.ASIF ALI KHAN
YANGTZE UNIVERSITY
104. Loop of Henle is called as
A. The countercurrent multiplier
B. The countercurrent exchanger
C. …..
D. …..
KEY: A
105. Which one of the following most likely causes Increase in GFR
A. Constriction of afferent arteriole
B. Constriction Of efferent arteriole
C. Decreased angiotensin
D. Dilatation of efferent arteriole
KEY: B. GFR increases with increase in glomerular pressure. Afferent constriction decreases blood flow to
glomeruli, thus decreasing glomerular pressure and GFR. Efferent constriction decreases blood flow from
glomeruli, thus increasing glomerular pressure and GFR.
106. A taxi driver comes to the clinic with complaints of runny nose, effused eyes, fever and malaise. He
wishes to continue his work while taking treatment for these symptoms. Which one of the following
anti-histamines would be most suitable for him?
A. Chlorphenamine
B. Diphenhydramine
C. Fexofenadine
D. Meclizine
KEY: C
107. A 42-year-old woman is evaluated for a 3-week history of vaginal discharge. The discharge is described
as yellowish and malodorous and is accompanied by burning, itchy and dyspareunia. She is sexually
active with a new male partner. Her last Pap smear was 2 years ago and was normal.
A. Trichomonas Vaginalis
» Trichomoniasis is caused by the organism called Trichomonas vaginalis and characterized by copious,
malodorous, pale yellow or gray frothy discharge with vulvar itching, burning, and postcoital bleeding; it
is effectively treated with a single dose of metronidazole. Treat the patient's sexual partner as well.
108. Case scenario of asthmatic patient and history of seasonal symptom’s
A. IgG
B. IgM
C. IgE
D. IgD
KEY: C
NEB/PMDC STEP-1 PAST PAPERS
DR.ASIF ALI KHAN
YANGTZE UNIVERSITY
109. In case of typhoid fever of 06 days. Which investigation is of first choice?
A. Widal test
B. Blood culture
C. Typhidot test
D. Urine culture
KEY: B
110. Case scenario: Within one hour of the myocardial infarction, which of the following enzymes serum will
be raised?
A. CK-MB
B. LDH
C. Alkaline Phosphatase
D. Troponin – T
KEY: A
111. Case scenario about Increased ICP — Cushing reflex ✅
» The triad of cushing’s is Increase ICP, hypertension, Bradycardia
» The Cushing reflex is a combination of hypertension and bradycardia.
» Raised intra-cranial pressure (ICP) leads to reduced cerebral perfusion and cerebral ischemic. This causes
a hypothalamic response leading to sympathetic activation, causing an increased cardiac output and
with peripheral vasoconstriction. This leads to systemic hypertension, which helps to increase the
cerebral perfusion pressure.
» The increased systemic blood pressure stimulates the carotid baroreceptors, causing a reflex
parasympathetic response causing the bradycardia.
» The Cushing reflex is usually seen in the terminal stages of an acute head injury. It is a sign of impending
brainstem herniation due to raised ICP.
112. Death certificate is written by
A. Authorized medical officer
B. Magistrate
C. …
D. …
113. A 55 year old obese male patient of type-ll diabetes mellitus was given metformin, but he became at
high risk of
A. Hypoglycemia
B. Excessive weight gain
C. Hepatotoxicity
D. Lactic acidosis
KEY: D
NEB/PMDC STEP-1 PAST PAPERS
DR.ASIF ALI KHAN
YANGTZE UNIVERSITY
114. S2 heart sound is produced due to:
A. Closure of aortic and pulmonary valves
B. Opening of aortic and pulmonary valves
C. Opening of tricuspid valve
D. Closure of tricuspid valve
KEY: A
115. A 30 year old female developed antibiotic-induced colitis due to C. difficile she was given Metronidazole
to treat it but it proved ineffective. Now she can respond to which oral drug
A. Erythromycin
B. Doxycycline
C. Vancomycin
D. amoxicillin
KEY: C
116. Case scenario with sarcoidosis microscopically by
A. Granoma with Asteroid
B. Non-caseating granuloma
C. Caseating granuloma
D. Macrophages and giant cells
KEY: B
117. Drug for AV nodal delay is
A. Propranolol
B. Adenosine
C. Flacinnide
D. Digoxin
KEY: B
118. Case scenario of Patient with polycythemia increase BP. The kidney secretes an enzyme-hormone which
raises blood pressure.
A. aldosterone
B. renin
C. angiotensinogen
D. angiotensin II
KEY: B
NEB/PMDC STEP-1 PAST PAPERS
DR.ASIF ALI KHAN
YANGTZE UNIVERSITY
119. A 15 year old boy with Type-I Diabetes Mellitus has been admitted in emergency with Severe
Ketoacidosis which needs rapid control with some appropriate antidiabetic drugs like
A. Crystalline regular insulin
B. NPH insulin
C. Insulin Lispro
D. Iultralente insulin
KEY: A
120. In a chronic carrier of hepatitis B virus (HBV), which positive test is most indicative of high infectivity of
viral replication?
A. HbsAg
B. HbcAg
C. HbeAg
D. Anti-HbeAg
KEY: C, HBeAg is detected in the serum of persons with high virus titers and indicates high infectivity.
121. Community medicine is the system of delivery of
A. Basic health care
B. Selective health care
C. Essential health care
D. Comprehensive health care
KEY: D
122. Case scenario of a chronic smoker and there is change in squamous epithelium
A. Metaplasia
B. Dysplasia
C. Hyperplasia
D. Neoplasia
KEY: A, change of columnar epithelium of the bronchi into mature squamous epithelium is called
metaplasia
123. Aspirin inhibits platelet aggregation by inhibiting:
A. Thromboxane A2 (TXA2)
B. Prostaglandins
C. Endothelial cells
D. Leukotrienes
KEY: A
NEB/PMDC STEP-1 PAST PAPERS
DR.ASIF ALI KHAN
YANGTZE UNIVERSITY
124. Mechanism of action of Penicillin is
A. transpeptidase cross link
B. protein synthesis inhibitors
C. nucleic acid synthesis inhibitors
D. act on DNA gyras
KEY: A
125. Splitting of S2 during inspiration occurs due to
A. Delayed closure of pulmonic valve
B. Early closure of pulmonic valve
C. Early closure of tricuspid valve
D. Early closure of mitral valve
KEY: A
126. Case scenario , infection of GI tract , bacteria or virus will first line defense against that microbial organism
A. Chemotactic
B. Skin
C. Complement
D. Opsonin
KEY: B
127. Cytotoxic cells are
A. B cells
B. Memory cells
C. T cells
D. Mast cells
KEY: C, T cells
128. Hormone which is responsible for the maintenance of corpus luteum
A. Testosterone
B. FSH and LH
C. HCG
D. Prolactin
KEY: C
129. Exogenous steroids in adrenal insufficiency—decrease ACTH
130. cavernous sinus thrombosis and Emissary vein related mcqs
131. Accidental injury – JG cells
132. Type 2 alveolar cell - Lung maturation ✅
133. Old and young age divide by — Median age✅
134. BMI wala age 55 , height 172cm , weight 88kg — 30✅
NEB/PMDC STEP-1 PAST PAPERS
DR.ASIF ALI KHAN
YANGTZE UNIVERSITY
135. Boxer hit a person on front of face , bleeding and smelly discharge which bone is fractured
136. Organ kept in during autopsy in — formaldehyde ✅
137. Cytotoxic T cells
138. Patient death in in illegal delivery—criminal negligence?
139. A person trying to lose weight by no-fat diet but consuming same amount of calories as like before. Which
of the following is contributing towards failure to lose weight — Carbohydrates
140. After milk intake abdominal bloating — Lactase deficiency ✅
141. Case scenario Behavior changes, Skin rashes, diarrhea - tryptophan ✅
142. Patient with H/O recurring attacks of pancreatitis, eruptive xanthomas and increased plasma triglycerides
level (Hypertriglyceridemia) associated with chylomicrons – Lipoprotein lipase deficiency
143. Lipoprotein particles are most likely responsible for the milky appearance of the patient's plasma —
chylomicron ✅
144. Case scenario no breast — Turner syndrome ✅
145. Severe starvation—Amino acid ✅
146. Case scenario Previous history of fetal loss — Anti-phospholipid syndrome✅
147. A 39-year-old woman gives birth to a term infant with a right transverse palmar crease, low-set ears,
oblique palpebral fissures, and a heart murmur and mental retardation. Which of the following
chromosomal abnormalities is most likely to be present in this child – Down syndrome 21
148. Glycogen synthase + by – insulin✅
149. Maximum substance made by —phenylalanine ✅
150. Postmortem (exhumation) — no limit ✅
151. Oral glucose test - impaired ✅
152. Road traffic Accident frequency — Enforcement of traffic law
153. Something about poisoning, guys pupil were constricted and pulling his earlobe dilated them for a short
while — alcohol
154. Diagnosis of DM HbA1c — 7-13%✅
155. Bradycardia results from increased vagal tone, drug should be given - atropine ✅
156. a case was of anti-ulcer drug which act slowly but once daily given -omeprazole✅
157. Chelation use for — heavy metal ✅
158. A person drink poison and vomiting due to— salt ✅
159. Thyroid swelling - pre tracheal ✅
160. Prostatic urethra related?
161. Scenario on Sea water (ship ) — direct to kidney damage✅
162. Oral thrush treatment — Nystatine✅
163. Insulin side effect— hypoglycemia ✅
164. More bleeding — incised wound✅
165. Blast injury —lung stab wound
166. In case of stab injury from other person or poisoning — inform police ✅
167. Multiple throat cut — suicide attempt ✅
168. Bird foot injury —driver car accident ✅
NEB/PMDC STEP-1 PAST PAPERS
DR.ASIF ALI KHAN
YANGTZE UNIVERSITY
169. Antisnake venom administration route— IV✅
170. Patient meal after poisoning , died — fatty meal ✅
171. Gunshot—8yard✅
172. WHO recommended salt —2g✅
173. Log phase
174. Eggshell appearance— silicosis✅
175. Prevent congenital rubella prevention – vaccinate female
176. Child with lose of subcutaneous fats and muscle wasting e hx of some intake of protein and other calories
— under nutrition? Marasmus
177. Night vision - cod liver oil✅
178. Vitamin A deficiency causes- Xerophthalmia ✅
179. Gestational DM - regular insulin ✅
180. Ephedrine- Bronchiole
181. COPD patient Moxifloxacin use for—14 days✅
182. Protective layer in stomach by which drug - sucralfate (gastric protective) ✅
183. A patient present with prolong use in seizure states, he has gum hyperplasia, hairsutism, Osteomalacia etc.
the following drug is responsible — Phenytoin ✅
184. Fluoroquinolones—arthropathy ✅
185.
186. Strong prokinatic — metoclopramide✅
187. Haloperidol induce tremors (drug induce Parkinsonism) — procyclidine✅
188. cromolyn sodium administration— inhalation ✅
189. Isoniazid—peripheral neuropathy ✅
190. Hyperuricemia is the side-effect of —pyrazinamide✅
191. Migraine pathophysiology – vasospasm ✅
192. Ewing sarcoma— onion skin✅
193. WBC 2000, RBC2milion , platelet 35k - aplastic anemia ✅
194. AB crosses placenta – IgG ✅
195. Hydropes fetalis ✅
196. Patient present with abdominal pain , amylase 100 time more normal — Acute pancreatitis
197. Most common tumor of parotid gland - mucoepdermoid carcinoma✅
198. Mycoplasma associated with anemia?? ✅
199. Compensated heart failure and Decompensated heart failure related mcqs
200. Liver injury – SGPT✅
201. Heinz bodies – G6PD deficiency ✅
» Heinz bodies appear in G6PD deficiency and oxidative stress
202. CKMB – MI 1MCQS✅
203. Paget disease — increase ALP ✅
204. Case scenario APTT prolonged — Factor Vlll deficiency ✅
205. Primary Raynaud’s phenomena
NEB/PMDC STEP-1 PAST PAPERS
DR.ASIF ALI KHAN
YANGTZE UNIVERSITY
206. Pencil cells, poikylicytosis- further Ix — hb electrophoresis?
207. Detective hepatitis – hepatitis D
208. 50 year old present with central chest pain, profuse sweeting, ST elevation on ECG next step – Troponin T
209. Epidemic goiter – potassium iodine✅
210. 10 year old traveling to Murree hills with family suddenly develop nausea and vomiting — scopolamine✅
211. Notochord derivative — nucleus pulposus ✅
212. Epistaxis location — septum
213. Body builder muscle atrophy —axillary??
214. Scaleus muscle — subclavian artery
215. Child foreign body obstruction —right main bronchus ✅
216. Boca’s area — MCA✅
217. Patient has H/O orbital cellulitis and result was cavernous sinus thrombosis and spread of infection to brain
is — ophthalmic vein
218. Left Bundle Branch is supplied by — Right coronary artery ✅
219. Loss of arm abduction — axillary nerve✅
220. Outstretch injury – ulnar Nerve ✅
221. During excercise blood supply decrease—cerebral ✅
222. Beta HCG - maintenance of corpus leutium ✅
223. Intra vascular hemolysis — heptoglobin
224. A malformation of the hand or foot resulting in fusion of two or more digits – syndactyly✅
225. Repolarization of membrane caused by — K ✅
226. 100ml hb in RBC - HB index✅
227. 3 days child dyspnea and nasal Filariasis – type 2 pneumocytes not present✅
228. Nessel bodies - RER in neuron ✅
229. Blood transfusion to infant – O negative
230. Resting tremors — occurs in resting state
231. Naegleria fowleri is transmitted by - swimming pool water (fresh water)✅
232. Diphtheria transmit through - via droplet ✅
233. Parasite rate - infant parasite rate✅
234. Vitamin D – Osteomalacia ✅
235. Muscle wasting and lower leg edema — Kwashiorkor ✅
236. Treatment of acne – retinol✅
237. hypoalbuminemia e effusion khawshioker disease✅
238. Vitamin b12 — methylmalonic acid
239. A lady is worried about breast cancer her mother also has breast cancer in order to prevent her method
should be —Self breast examination
240. Girl playing in bushes— contact dermatitis
241. Maternal infection lead to peri carditis - Rubella (German measles)
242. Further elongation of fatty acid take place in — peroxisome ✅
243. Pulmonary larevea — Ascaris✅
244. Case scenario on Measles — paramyxo virus ✅
NEB/PMDC STEP-1 PAST PAPERS
DR.ASIF ALI KHAN
YANGTZE UNIVERSITY
245. Retroperitoneal pain back — supra renal gland✅
246. Mild moderate severe - ordinal ✅
247. Polio vaccine when not to give — Inner dark, outer light ✅
248. The cases occur irregularly and haphazardly — Sporadic ✅
249. Antenatal care – investigation for fatal anomalies ✅
250. Contraception of choice for married with 1 child – IUCD
251. Basic health unit (BHU) in Pakistan 5400✅
252. Case scenario previous delivery of women child died due to meningocele now she is pregnant Rx – folic
acid ✅
253. One mcqs was Brownsquard syndrome and Hemisection of spinal cord
254. Liver vitamin k dependent factor ✅
255. lecithin–sphingomyelin ratio used to check – Lung maturity ✅
256. Rh -ve mother with Rh + child and child is anemic so blood transfusions – O Negative ??
257. Episodic hypertension – chromaffin cells ✅
» Episodic hypertension is a feature of Pheochromocytoma and Pheochromocytoma is a tumor that arises
from chromaffin cells.
258. A person hit by medial malleolus which action damage — Planter-flexion✅
259. Mallary bodies — alcoholic hepatitis ✅
260. Iron deficiency anemia – ferritin
261. Thoracic duct lies in which mediastinal compartment— posterior compartment✅
262. Atherosclerosis which artery damage — LAD
263. Hematogenous spread – ovarian CA? Sarcoma?
264. Pancrease CA spread – Common Bile Duct
265. Jaundice , pruritis, marker will be increase – ALP ✅
266. chronic HBV replication – HbeAg
267. patient with hx of chills and fever now hb uria – G6PD deficiency ✅
268. Hyperproteineuria ki reason—A. Bowman capsule? B. Endothelial c. injury Podocyte D. injury GBM
269. Paget disease ALP increase, nipple discharge – invasive ductal CA?
270. Allergic symptoms— mast cells activation✅
271. Increase anion gap – metabolic acidosis✅
NEB/PMDC STEP-1 PAST PAPERS
DR.ASIF ALI KHAN
YANGTZE UNIVERSITY
KINDLEY IF CORRECT ANY KEY IF MISTAKENLEY PUT WRONG
I WILL SHARE PAST PMDC STEP-2 AND STEP 3 OSCE QUESTION WITH
COMPLETE EXPLAINATION INSHA-ALLAH SOON
THIS NOTES IS AVAILABLE IN LAHORE AT STUDENTS PHOTOCOPY KHWAJA
ARCADE WAHDAT ROAD MAYBE IT WILL HELP IN YOUR STEPS EXAM, this
NOTES IS FREE OF CAST FOR MY ALL FRIENDS
REMEMBER ME AND MY FAMILY IN YOUR PRAYERS
BEST OF LUCK FOR THE STEPS EXAME
DR. ASIF ALI KHAN

More Related Content

PDF
Davidson mc qs
PPT
02 cardiac pathology
PPTX
CARDIAC PATHOLOGY (MCQ QUESTIONS & ANSWERS)
PDF
Cardiovascular Pathology Case-based_Gross and Microscopic
PDF
Pathology Slides For Exam (Kiev Medical University)
PPTX
Pathology of Cardiomyopathies
PPTX
Pathology & micro biology intro
DOCX
Ischemic heart diseae lecture
Davidson mc qs
02 cardiac pathology
CARDIAC PATHOLOGY (MCQ QUESTIONS & ANSWERS)
Cardiovascular Pathology Case-based_Gross and Microscopic
Pathology Slides For Exam (Kiev Medical University)
Pathology of Cardiomyopathies
Pathology & micro biology intro
Ischemic heart diseae lecture

What's hot (20)

DOCX
Pathology mcq
DOC
Pathology Practice Examination
DOCX
MCQs respiratory system
DOCX
Mcqs infectious diseases 08
PPTX
Pathology of CNS Tumors - Quiz
DOCX
Mcqs cvs 2
DOCX
Mcq contact diseseaes
PPTX
Dermatology MCQ and AAFP.pptx
PDF
MCQ July with answers - Dr Ameen Alawadhi
DOC
Physiology
PDF
MCQs-for-Dermatology-1-1.pdfffffffffffff
DOC
Internal Medicine Sample Questions
PPTX
Pathology of CNS Degenerations - In class Quiz
DOCX
Medicine BCQs (Respiratory system BCQs)
PPTX
Hepatitis B virus
DOCX
Written questions on General Pathology for 4th year MBBS students.
PDF
Krok -2 Previous Year Paper (2020)
DOC
ETAS_MCQ_05 dermatopathology
PPT
Microbiological diagnosis infective endocarditis
PPTX
meningitis
Pathology mcq
Pathology Practice Examination
MCQs respiratory system
Mcqs infectious diseases 08
Pathology of CNS Tumors - Quiz
Mcqs cvs 2
Mcq contact diseseaes
Dermatology MCQ and AAFP.pptx
MCQ July with answers - Dr Ameen Alawadhi
Physiology
MCQs-for-Dermatology-1-1.pdfffffffffffff
Internal Medicine Sample Questions
Pathology of CNS Degenerations - In class Quiz
Medicine BCQs (Respiratory system BCQs)
Hepatitis B virus
Written questions on General Pathology for 4th year MBBS students.
Krok -2 Previous Year Paper (2020)
ETAS_MCQ_05 dermatopathology
Microbiological diagnosis infective endocarditis
meningitis
Ad

Similar to Step 1 PMDC paper compiled by Dr. Asif ali (20)

PDF
Jipmer solved 2000
PDF
Jipmer solved 2000 1
PDF
Jipmer 2000
PDF
Nclex test review
PDF
NIMSET 2013 Questions
DOCX
Nepal nursing council licencing exam mcqs sample
PDF
Exam Qs for NURS 2536 Pathophysiology Pharmacology and Knowledge Integration ...
DOC
Multiple Choice Questions (MCQS) Basic and Clinical Science Course.doc
PDF
FMGE-RECALL-July2023-PrepLadder.pdf to ace the revision session
DOC
AIIMS 2001 quetions
DOC
523 ophthalmology MCQs
PPTX
Nursing Competitive Exam Model Questions - 2018
PDF
Mcq for laboratory medicine
PPTX
QUESTION SESSION -msn ashok baniwal.pptx
PDF
Krok 1 - 2014 Question Paper (General medicine)
DOCX
Bpkihs 2012
PDF
Usmle step 2 ck september 2020 recalls (dr notes.com)
DOCX
Comprehensive exam for nursing students question
PDF
M C I F M G E Question Papers 2002 2008
DOCX
COUNCIL THEORY 2018 ANSWERED-1.docx
Jipmer solved 2000
Jipmer solved 2000 1
Jipmer 2000
Nclex test review
NIMSET 2013 Questions
Nepal nursing council licencing exam mcqs sample
Exam Qs for NURS 2536 Pathophysiology Pharmacology and Knowledge Integration ...
Multiple Choice Questions (MCQS) Basic and Clinical Science Course.doc
FMGE-RECALL-July2023-PrepLadder.pdf to ace the revision session
AIIMS 2001 quetions
523 ophthalmology MCQs
Nursing Competitive Exam Model Questions - 2018
Mcq for laboratory medicine
QUESTION SESSION -msn ashok baniwal.pptx
Krok 1 - 2014 Question Paper (General medicine)
Bpkihs 2012
Usmle step 2 ck september 2020 recalls (dr notes.com)
Comprehensive exam for nursing students question
M C I F M G E Question Papers 2002 2008
COUNCIL THEORY 2018 ANSWERED-1.docx
Ad

Recently uploaded (20)

PPTX
Approach to chest pain, SOB, palpitation and prolonged fever
PPTX
thio and propofol mechanism and uses.pptx
PDF
OSCE Series ( Questions & Answers ) - Set 6.pdf
PPTX
Neoplasia III.pptxjhghgjhfj fjfhgfgdfdfsrbvhv
PDF
MNEMONICS MNEMONICS MNEMONICS MNEMONICS s
PPT
Opthalmology presentation MRCP preparation.ppt
PPTX
Reading between the Rings: Imaging in Brain Infections
PDF
The_EHRA_Book_of_Interventional Electrophysiology.pdf
PPTX
Physiology of Thyroid Hormones.pptx
PPTX
Post Op complications in general surgery
PPTX
Hypertensive disorders in pregnancy.pptx
PPT
Rheumatology Member of Royal College of Physicians.ppt
PPTX
SHOCK- lectures on types of shock ,and complications w
PPTX
Critical Issues in Periodontal Research- An overview
PPTX
CARDIOVASCULAR AND RENAL DRUGS.pptx for health study
PPT
Infections Member of Royal College of Physicians.ppt
PDF
AGE(Acute Gastroenteritis)pdf. Specific.
PDF
04 dr. Rahajeng - dr.rahajeng-KOGI XIX 2025-ed1.pdf
PPT
neurology Member of Royal College of Physicians (MRCP).ppt
PPTX
Assessment of fetal wellbeing for nurses.
Approach to chest pain, SOB, palpitation and prolonged fever
thio and propofol mechanism and uses.pptx
OSCE Series ( Questions & Answers ) - Set 6.pdf
Neoplasia III.pptxjhghgjhfj fjfhgfgdfdfsrbvhv
MNEMONICS MNEMONICS MNEMONICS MNEMONICS s
Opthalmology presentation MRCP preparation.ppt
Reading between the Rings: Imaging in Brain Infections
The_EHRA_Book_of_Interventional Electrophysiology.pdf
Physiology of Thyroid Hormones.pptx
Post Op complications in general surgery
Hypertensive disorders in pregnancy.pptx
Rheumatology Member of Royal College of Physicians.ppt
SHOCK- lectures on types of shock ,and complications w
Critical Issues in Periodontal Research- An overview
CARDIOVASCULAR AND RENAL DRUGS.pptx for health study
Infections Member of Royal College of Physicians.ppt
AGE(Acute Gastroenteritis)pdf. Specific.
04 dr. Rahajeng - dr.rahajeng-KOGI XIX 2025-ed1.pdf
neurology Member of Royal College of Physicians (MRCP).ppt
Assessment of fetal wellbeing for nurses.

Step 1 PMDC paper compiled by Dr. Asif ali

  • 1. NEB/PMDC STEP-1 PAST PAPERS DR.ASIF ALI KHAN YANGTZE UNIVERSITY
  • 2. NEB/PMDC STEP-1 PAST PAPERS DR.ASIF ALI KHAN YANGTZE UNIVERSITY 02 JUNE 2016 MCQS OF NEB PMDC STEP-1 Community Medicine 1. Dental caries is due to lack of – Fluoride 2. Contain High Calories – Fat and Oil 3. Communication in School – Two way 4. Death rate declining, birth rate declining, stage? low expanding 5. Not included in Maternity Cycle – Weaning 6. Muscle stretching of factory worker, management – Ergonomics or Training 7. Not Included in PHC – Disequality 8. Not a prevention of Malaria – Vaccination 9. Polio schedule not included in EPI – 1 year 10. Pregnant Lady with anemia give specific protection – Secondary Protection/ Early diagnosis and Tx 11. 10 % of Population – Simple Cross sectional/ Stratified Cross Sectional 12. Topographic Accident – Avalances 13. Poor Overcrowded House – Rheumatic Heart Disease (Unemployment and overcrowding is associated with RHD) 14. Best Prevention of Non-Communicable Disease – Health Education/Genetic Counseling 15. Not risk factor for infant – 3 kg weight/ prematurity... 16. marasmus 17. Best teaching method in school, two way communication 18. Which one is not at risk of malnutrition 3 kg baby weight? at birth 19. Death rate decline, late expending 20. Polio vaccine not include in EPI schedule, zero at birth
  • 3. NEB/PMDC STEP-1 PAST PAPERS DR.ASIF ALI KHAN YANGTZE UNIVERSITY 8 JANUARY 2012 MCQS OF NEB PMDS STEP-1 1. In a hypertensive patient which of the following will least likely to reduce the hypertension A. Calcium intake B. Potassium intake C. Moderation in alcohol D. Weight loss E. Increase in physical activity 2. An example of acute inflammation is A. Pneumonia B. Tuberculosis C. Sarcoidosis D. Encephalitis 3. In Pakistan aedes egypti mosquito transmit A. Dengue fever B. Yellow fever C. Leishmaniasis D. Japanese encephalitis 4. Which of the following is the true regarding BCG vaccination: A. BCG is not fully protective in adults and it is a live attenuated vaccine B. It is protective in children’s C. It is a live attenuated vaccine D. Produce passive immunity 5. For the para follicular cells in thyroid and the parathyroid cells, best explanation is A. Calcitonin and PTH 6. The hormone that controls the level of calcium and phosphorus in blood is secreted by A. Thyroid gland B. Parathyroid gland C. Pituitary gland D. Thymus KEY: B, Parathyroid gland is tiny glands. It controls the body’s calcium level. Each gland is about the size of a grain of rice. 7. In a conflict between two equally competence medical witness the court will fallow A. The witness who seen the injured in the beginning B. The witness who favor in defense C. The witness who seen the injured in the end 8. If the vertical length of a window will increase what will happen in the light / shadows coming through it?
  • 4. NEB/PMDC STEP-1 PAST PAPERS DR.ASIF ALI KHAN YANGTZE UNIVERSITY 9. The privileges of a register medical practioner is A. can advertise in news paper B. can voluntarily appear in court 10. In younger children’s tetracycline is not used because A. It is hepatotoxic B. Cause bone calcification 11. In a child with bloody diarrhea which of the following organism will be in the stools found in stools lab exam. A. E-coli B. Shigella C. Campylobacter 12. In the adrenal gland mineralocorticoids comes from A. Zona fasciculate B. Adrenal cortex C. Zona glomerulosa D. Adrenal medulla E. Zona Reticularis 13. The size of particle to cause pneumoconiosis A. 0.5 to 3 micron B. 4 to 7 micron C. 10 to 12 micron 14. A patient with Bradykinesia, this is described as A. Cerebellar lesion B. Levy bodies C. Parkinsonism D. Substancia nigra 15. An abnormality in coordination movement is termed as A. Ataxia B. Cerebellar lesion C. Dysdiadochokinesia 16. Long term memories stored in which part of the brain A. Thalamus B. Cerebellum C. Cerebral cortex D. Hippocampus 17. Chop wound in? A. Revolver B. Short gun
  • 5. NEB/PMDC STEP-1 PAST PAPERS DR.ASIF ALI KHAN YANGTZE UNIVERSITY 18. Cookie cutter in fire arm injury? » Also known as: Rat hole injury / Cookie cutter phenomenon/ Moth eaten appearance of wound of entry » In case of shot gun fire, up till 1 meter (3feet) all the pellets make a single large hole. » After 3 feet the edges of the wound will become crenated. » From 2 meters upwards, the number of satellite pellet holes will progressively increase around the main wound. 19. Warm antibody in hemolytic anemia – IgG type of antibodies 20. Zona glomerulosa secrete – mineralocorticoid (aldosterone) 21. Erythropoietin is secreted from – peritubular capillary (interstitial fibroblasts in the kidney) 22. Under the Qisas and Diyat Act, injuries present on the neck are classified as: A. Jaifa Hurts B. Other hurts falling U/S 337 L II. C. Shajjah D. Jurh. E. Damiyah KEY: E 23. Swollen bleeding gums in??? A. Scurvy B. Vitamin d deficiency 24. BP 120/80 mean arterial pressure (MAP) is – 93mmHg » MAP = Diastolic pressure + 1/3 pulse pressure » At 120/80mmHg MAP = 80mmHg + 1/3 of pulse pressure (40mmHg) = 93mmHg 25. Complication of successful treatment of thrombophlebitis A. rupture B. Infection C. thrombus formation KEY: C 26. Incubation period of HCV A. 20 days B. 50 days C. 30 days D. 10 days KEY: B 27. Derivative of 4th pharyngeal arch – external laryngeal nerve 28. Muscle used in walking – Tabialis anterior
  • 6. NEB/PMDC STEP-1 PAST PAPERS DR.ASIF ALI KHAN YANGTZE UNIVERSITY 29. Enlargement of the prostate cells usually arise from A. periurethral area B. peripheral area C. transitional zone 30. A patient with only HBsAg positive will be called carrier if A. HBsAg positive for more than 6 months 31. If a person’s BP is 120/90, the mean arterial pressure will be A. 30 B. 20 C. 120 D. 90 KEY: D 32. In deafness, garment is inserted in the ear in A. conductive type of deafness B. Sensorineural type C. Presbycuses 33. ECG shows no P wave, where is the pace maker A. Ventricle B. AV node C. Atrium D. Purknje fibers E. Bundle of his 34. Counselling is a technique which aims at : A. Making people less emotional. B. Achieving a greater depth of understanding and clarification of the problem C. Comparing the patient’s experiences with one’s own. D. Giving sincere advice and solutions to the patients problems E. Breaking bad news in a professional manner. KEY: B 35. Patient may present with lucid interval in A. Subdural hematoma B. Extradural haematoma C. Sub arachnoid hemorrhage KEY: B 36. A highly infective but less pathogenic state is found in A. Sporadic outbreak B. Epidemic outbreak C. Carrier
  • 7. NEB/PMDC STEP-1 PAST PAPERS DR.ASIF ALI KHAN YANGTZE UNIVERSITY 37. Which of the following increase plasma level of phenytoin A. Penicillin B. Carbamazepine C. Cimetidine KEY: B » Phenytoin metabolism is decreased by: Cimetidine, isoniazid, chloramphenicol, Dicumarol, Sulfonamide » Phenytoin metabolism is increased by Carbamazepine 38. Which of the following states have least metastatic spread? A. Squamous cell carcinoma B. Basal cell carcinoma C. Melanoma 39. Case of kidney transplant, Patient on immunosuppressive therapy, he has severe headache and some other signs the organism was isolated they were branched and were stained with Indian Ink. A. Pneumocystis carniii B. Pneumocystis jervoci C. Cryptococcus neoformans D. Candida 40. Causative agent of urinary bladder Cancer A. Schistosoma Japonicum B. Schistosoma Mansoni C. Schistosoma Haematobium KEY: C 41. Patient had luminescent stools and vomit is due to A. Arsenic poisoning B. Copper poisoning C. Phosphorus poisoning KEY: C 42. Activated charcoal is used in following poisoning A. Metals B. Alkalis C. Acid bases D. Lithium KEY: A
  • 8. NEB/PMDC STEP-1 PAST PAPERS DR.ASIF ALI KHAN YANGTZE UNIVERSITY 43. Which of the product is recycled in urea cycle after urea is made? A. Citrulline B. Ornithine C. Arginosuccinate KEY: C 44. Arsenate poisoning blocks which step in TCA cycle A. Isocitrate dehydrogenase B. Alpha keto glutarate dehydrogenase C. Succinyl dehydrogenase KEY: B, Arsenate poisoning inhibit pyruvate dehydrogenase, alpha keto glutarate dehydrogenase 45. Epo-ophron and para-ophron are remnants of A. Mullerian duct B. Mesonephros C. Metanephros KEY: B 46. Which of the following medication for GI problems is contraindicated in pregnancy? A. Calcium carbonate B. Famotidine C. Lanzoprazole D. Misoprostol KEY: D, Contraindicated in pregnancy because it may stimulate uterine contractions. 47. An elderly woman with recent hx of myocardial infarction is seeking medication to help treat occasional heartburn. She is currently taking several meds, including aspirin, Clopidogrel, simvastatin, metoprolol, and Lisinopril. Which of the following choices should be avoided in this patient? A. Calcium citrate B. Omeprazole C. Ranitidine D. Calcium carbonate KEY: B, May possibly decrease the efficacy of clopidogrel because it inhibits the conversion of clopidogrel to its active form
  • 9. NEB/PMDC STEP-1 PAST PAPERS DR.ASIF ALI KHAN YANGTZE UNIVERSITY 19 SEPTEMBER 2015 UHS MCQS OF NEB PMDC STEP-l Physiology 1. Increase Cardiac output — cardiac reservoir 2. For renal function endogenous substance is — creatinine 3. For increasing concentration of local anesthesia — epinephrine 4. watery diarrhea gram negative increase — CAMP 5. Immune regulator are suppress — T cells 6. CSF protein — 350gm 7. MI ECG with one year history — Q wave 8. High K lead to release of — Aldosterone 9. Albumin — negative charge 10. Absent A wave — atrial fibrillation 11. 28 litters out of 42 store inside — cells of body 12. After 4th days chest pain increase with inspiration is — Fibrinoid pericarditis Biochemistry 1. Best for heart is — Hdl:LDL is 3:1 2. Enzyme of HMP shunt for hemolysis is — glutamate oxidase 3. Main regulator enzyme for Krebs cycle is – alpha ketoglutrate 4. Diabetes mellitus— glycosylate hb 5. Serotonin — 10 min 6. Enzyme competitive inhibitors – increase km 7. During fasting blood glucose level is main by – glycogenolysis 8. Darkness growth —Vit A 9. Cholelithiasis — vitamin E 10. Which among the following is a cardioprotective fatty acid – Omega-3 fatty acids » Oleic acid is a monounsaturated fatty acid – cardioprotective, reduce LDL. » Omega-3 fatty acids like linolenic acid and docosahexaenoic acid are cardioprotective.
  • 10. NEB/PMDC STEP-1 PAST PAPERS DR.ASIF ALI KHAN YANGTZE UNIVERSITY Pharmacology 1. Drug along with nitrates to overcome oxygen demand is—metoprolol 2. Drug for aspirin induce asthma— zafirleukast 3. Drug for MRSA — Vancomycin 4. Drug for mild Amebiasis is — metronidazole 5. Anti-tubercular drug in pregnancy which lead to deafness is — streptomycin » Streptomycin is contraindicated in pregnant women because of fetal ototoxicity. 6. Drug of choice for diabetic nephropathy is — captopril 7. Digoxin toxicity ECG – increase PR interval 8. Drug which is not metabolize in liver— diazepam 9. Case scenario and ask side effect of Ethambutol – Optic neuritis 10. Side effect of insulin is – hypoglycemia 11. Most potent Fluoroquinolones against Mycobacterium leprae is – Moxifloxacin 12. Drug which cause discoloration of teeth in the fetus – Tetracycline 13. Drug which inhibits insulin secretion and causes hyperglycemia – Phenytoin 14. Drug of choice in acute pulmonary edema due to left ventricular failure – Furosemide » Frusemide is a loop diuretic. It is also a pulmonary venodilator. In left ventricular failure, frusemide is given intravenously. It causes pulmonary venodilation and relieves pulmonary congestion. Also by increasing urine formation, it decreases blood volume and venous return thus decreasing preload of the heart. Pathology 1. Necrosis in granuloma of TB — Caseous necrosis 2. Long term Hypertension lead to— left ventricular hypertrophy 3. Diagnostic investigation for thalassemia is – Hb electrophoresis 4. Movements disorders plus dementia — Huntington disease 5. Ankyrin spectrin deficiency — hereditary spherocytosis 6. Collapsing pulse with diastolic murmur is aortic regurgitation 7. Case scenario of a worker who had been in the battery manufacturing unit for the last 23 years, reported to you with complaints of lost appetite and abdominal colic of 2 weeks duration. You would prefer to investigate him for: A. Cholecystitis B. Lead poisoning C. Appendicitis D. Ameobiasis E. Stomach cancer KEY: B
  • 11. NEB/PMDC STEP-1 PAST PAPERS DR.ASIF ALI KHAN YANGTZE UNIVERSITY 8. Epithelium of endocervix is mucus secreting — simple columnar epithelium 9. Pyloric gland cause bicarbonate secretion along with — mucus 10. Wound —tetanus 11. Vertical transmission — HIV/AIDS 12. Crohn disease— non caseating granuloma 13. Case scenario of patient with Gastroectomy, causes deficiency of —B12 14. ALT and AST arise with IgM anti HAV—fulminant hepatitis 15. Pigmentsion with low Na and ACTH is — Addison’s disease 16. Antidote for methanol is — Ethanol 17. Pinpoint purpura with low platelets high bleeding time and clotting time is Thrombotic thrombocytopenia 18. Electroencephalogram (EEG) petit mal seizures with – 3-Hz spike-and-wave discharges 19. Patient with centripetal obesity, acne and hairsutism – Cushing’s syndrome 20. Endotoxin—TNF Forensic medicine 1. Patient negligence is contributory 2. Most common cause of death in case of dental anesthesia is – Chocking 3. Exhumation done under order by A. Police officer B. Superintendent police C. First class magistrate KEY: C 4. Earliest sign of death is A. loss of skin elasticity B. Corneal clouding C. Cooling of body D. Postmortem lividity 5. Postmortem caloricity seen in poisoning from A. Arsenic B. Strychnine C. Cyanide D. Organophosphorus KEY: B
  • 12. NEB/PMDC STEP-1 PAST PAPERS DR.ASIF ALI KHAN YANGTZE UNIVERSITY 6. Blunt abdominal trauma commonest site of GI ruptures is A. Stomach B. Duodenum C. Jejunum D. Transverse colon KEY: D 7. Characters of smooth bored firearm are: A. It is generally a long barreled firearm B. The inside of barrel is marked by grooves C. It fires spinning missiles D. It usually fires single missile KEY: A 8. Medico legal system practiced in Pakistan: A. Coroner's system B. Continental system C. Modified continental system D. Judicial / police inquest KEY: D Anatomy 1. Only lateral ventricular enlargement – Intraventricular stenosis 2. Stab wound in axilla with lose of supination and flexion at elbow joint is – musculocutanoeus nerve 3. Fracture of shaft of humerus is — radial nerve 4. Fibular neck fracture —common peroneal nerve 5. Titling or positive Trendelenberg sign — gluteus maximum and minimus 6. Hit on lateral side of head artery damage is – frontal branch of meningeal artery (also called anterior branch 7. Appendicular artery is — posterior cecal artery 8. Deep venous thrombus – one of pulmonary arteries 9. Babinski dorsiflexion of greater toe plus fainting of other toe 10. Thrombus—greater saphenous vein 11. Lateral meniscus – hearing 12. Facial nerve palsy 13. Lymph from medial inferior side drained into – parasternal lymph node 14. structure behind the urinary bladder of male is – seminal vesicle 15. Lateral part of posterior wall of inguinal canal is formed by – transversalis
  • 13. NEB/PMDC STEP-1 PAST PAPERS DR.ASIF ALI KHAN YANGTZE UNIVERSITY 16. Loss of index finger flexion and loss of sensation on palmer aspect – Median nerve 17. Difficulty rising from standing position damage to – gluteus Maximus muscle 18. Right suprarenal vein drains in to – IVC 19. Laterally rotated thumb by which nerve – radial nerve 20. Thyroid is enclosed in which fascia – Pre tracheal Microbiology 1. In case of TB CSF — low glucose with lymphocytosis 2. Bloody diarrhoea with causative organism in mucosa – C. difficile 3. Yellow sputum with fever and on x ray infiltration abd neutrophilia is — S.aureus 4. Herpes simplex virus transmitted by – droplet 5. Currant jelly sputum – klebsiella 6. Pear shape eggs and causing diarrhoea – Giardia 7. Farmer with migratory lesion on foot — Cutaneous Larva migrans 8. Definitive diagnosis of TB is made through — sputum AFB Community medicine 1. A study was conducted to investigate the effect of HIV infection on mortality among people in Kenya with TB. Individuals with TB were recruited from hospitals and their HIV status determined. They were then followed-up over ten years to compare mortality rates in the HIV positive group and HIV negative group. A. Case-control study B. Cohort study C. Randomized controlled trial D. Cross-sectional study KEY: B 2. The non-clinical Vitamin A deficiency is more common as compared to the clinical, threatening the health of as many as one third of the world’s children. The best proxy indicator of this is: A. Infant mortality rate B. Maternal mortality rate C. Literacy rate D. Proportion of the rural population KEY: A
  • 14. NEB/PMDC STEP-1 PAST PAPERS DR.ASIF ALI KHAN YANGTZE UNIVERSITY 3. Which of the following is caused due to extreme lack of proteins? A. Malaria B. Typhoid C. Kwashiorkor D. None of these KEY: C 4. One gram of protein contains A. 4.3 kilocalories of energy B. 4.2 kilocalories of energy C. 5.2 kilocalories of energy D. 5.4 kilocalories of energy KEY: B 5. Community mean deviation 6. To limit the movements of patient – Quarantine 16 March 2017 (NUMS) MCQS OF NEB PMDC STEP-l Anatomy 1. A 70-year-old man has difficulty rising from a seated position and straightening his back, but he has no difficulty flexing his hip and knee. Which one of the following muscles is most likely to be weak? A. Hamstrings B. Ileo-psoas C. Obturator internus D. Gluteus maximus E. Gluteus minimus KEY: D 2. What exits from the Stylomastoid foramen A. Chorda tympanic nerve B. Accessory nerve C. Facial nerve D. Glossopharyngeal nerve E. Hypoglossal nerve KEY: C
  • 15. NEB/PMDC STEP-1 PAST PAPERS DR.ASIF ALI KHAN YANGTZE UNIVERSITY 3. Temporal lobe – Middle Cranial Fossa 4. Vagus nerve injury – least likely effect Sigmoid colon 5. Middle Nasal concha – Ethmoid bone 6. Right ear near shoulder – Sternocleidomastoid 7. Wernicke’s area – Middle cerebral artery 8. Main support of Uterus – Urogenital diaphragm?? 9. Vein used in graft — Greater Saphenous vein 10. If nystagmus is a prominent symptoms of a cerebellar lesion, the lesion within: A. The dentate nucleus B. The flocculonodular lobe C. The lateral cerebellum D. The cerebrocerebeller cortex E. The superior cerebellar peduncle KEY: B 11. The oesophageal opening in the diaphragm is at A. T6 B. T8 C. T10 D. T12 E. L4 KEY: C 12. The isthmus of thyroid gland – lies anterior to the 2nd , 3rd , and 4th Tracheal rings A. Lies directly anterior to the thyroid cartilage B. Lies directly anterior to the cricoid cartilage C. Lies directly anterior to the 2nd, 3rd, and 4th Tracheal rings D. Is the same as the pyramidal lobe E. Contains the superior pair of parathyroid glands KEY: C 13. The structure passing through the aortic opening of diaphragm A. Accessory nerve B. Esophagus C. Superior vena cava D. Azygous Vein E. L-R laryngeal KEY: D, Azygous Vein
  • 16. NEB/PMDC STEP-1 PAST PAPERS DR.ASIF ALI KHAN YANGTZE UNIVERSITY 14. Tennis player – Extensor Carpi Radialis 15. Volkmann canal— oblique 16. Myocardial infarction — Left anterior descending artery (LAD) » Most commonly involved coronary artery in myocardial infarction (MI) is left anterior descending artery. 17. Laterally displaced Tibia over Femur – Medial Collateral Ligament 18. Osteosarcoma – Knee joint 19. Suspensory ligament of axilla is derived from A. Clavipectoral fascia B. Deltoid C. Axillary sheath D. Platysma E. Pectoralis major KEY: A 20. The fetal period extends from: A. 12th week to term B. 3rd week to 8th week C. 4th week to term D. 5th week to term E. 9th week to term KEY: E 21. A 15 years old boy is brought to emergency after a fall from his bicycle x-ray reveals fracture of neck of fibula. On physical examination the patient a foot drop. Which of the following nerves is most likely injured A. Tibial B. Common fibular (peroneal) nerve C. Superficial fibular (peroneal) nerve D. Saphenous E. Deep fibular (peroneal) nerve KEY: B 22. A newborn presented with dribbling of urine from the umbilicus, he was diagnosed of having urachal fistula which is a remnant of A. Allantois B. Urogenital sinus C. Urorectal septum D. Cloaca E. Mesonephric duct KEY: A
  • 17. NEB/PMDC STEP-1 PAST PAPERS DR.ASIF ALI KHAN YANGTZE UNIVERSITY 23. A development of thyroid gland is marked by a depression called foramen cecum in the floor of the primitive pharynx. The foramen caecum gives rise to a diverticulum which later elongates and descends anterior to the hyoid bone and thyroid cartilage to assume the adult position. What is this diverticulum called? A. Hypoglossal duct B. Laryngeal diverticulum C. Respiratory diverticulum D. Thyroglossal duct E. Thyroglossal diverticulum KEY: D 24. A 60-year-old male executive who had a history of a chronic duodenal ulcer was admitted to the ER exhibiting signs of a severe internal hemorrhage. He was quickly diagnosed with perforation of the posterior wall of the first part of the duodenum and erosion of an artery behind it by the gastric expellant. The artery is most likely the: A. Common hepatic B. Gastroduodenal C. Left gastric D. Proper hepatic E. Superior mesenteric KEY: C, Gastroduodenal 25. A 43 year old women came with large abscess in the middle of right post triangle of the neck. The physician incised and drained the abscess. Five days later patient noticed that she could not extend her right hand above her head to brush her hair. Which of the following structure is damaged A. Damage to scaleneus Medius B. Spread of infection to shoulder joint C. Spinal accessory nerve injury D. Thoracodorsal nerve injury E. Suprascapular nerve injury KEY: C, Spinal accessory nerve injury 26. A women was diagnosed as having a cervical carcinoma. For exfoliative cytology cervical cells were taken from the junction of endocervix with ectocervix. Here the epithelium changes from simple columnar to: A. Stratified columnar B. Stratified cuboidal C. Stratified squamous keratinizing D. Stratified squamous non-keratinizing? E. Simple squamous epithelium KEY: D
  • 18. NEB/PMDC STEP-1 PAST PAPERS DR.ASIF ALI KHAN YANGTZE UNIVERSITY Biochemistry 1. A 10 years old was evaluated for burning sensation in his feet and clusters of small red purple spots on his skin. Lab studies reveal protein in his urine. Enzyme analysis revealed a deficiency of alpha galactosidase and enzyme replacement therapy was recommended. The most likely diagnose is A. Fabry disease B. Farber disease C. Gaucher disease D. Krabbe disease E. Niemann-Pick disease KEY: A. Fabry disease 2. A 2 years child with severe fasting hypoglycemia, hyperlacticedemia and hyperuricemia. Biopsy reveal hepatocytes with increased amount of glycogen – Von Griek disease (G6P deficiency) 3. Enzyme which is deficient or absent in lens, causing cataract in diabetes – Sorbitol dehydrogenase deficiency 4. Sorbitol accumulation 5. 37 years old male with signs and symptoms of hyperglycemia, diagnose is Diabetes Mellitus type-2 due to insulin resistance, lab show hypertriglyceridemia, hyperlipidemia – very low density lipoprotein (VLDL) 6. Which of the following lipoprotein particles are most likely responsible for the appearance of the patient's plasma? A. Chylomicrons B. HDL C. IDL D. LDL E. VLDL KEY: A, Chylomicrons 7. A term infant is born and does well with breast feeding. Two days later his mother calls frantically because the baby is bleeding from umbilicus as nostrils. The following vitamin is most probably deficient in this infants: A. Vitamin A B. Vitamin D2 C. Vitamin D3 D. Vitamin E E. Vitamin K KEY: E
  • 19. NEB/PMDC STEP-1 PAST PAPERS DR.ASIF ALI KHAN YANGTZE UNIVERSITY 8. Corn is continuously low in water soluble vitamin. In 1936, the corn meal was fortified with the vitamins, and the consequently large numbers of the insane asylum inmates in South Carolina (where corn is the staple diet were miraculously cured of mental illness. The vitamin that cured their pellagra and their mental illness was A. Folic acid B. Niacin C. Pyridoxine D. Riboflavin E. Thiamine KEY: B 9. Lactic acidosis – Thiamine 10. Semi-essential Amino Acid – Arginine 11. Pregnant with Cystic Acne – Vitamin A 12. Case scenario of Albinism is due to deficiency of the enzyme: A. Phenylalanine B. Tyrosinase C. P-Hydroxyphenylpyruvic acid oxidase D. Tyrosine dehydrogenase KEY: B 13. RNA does not contain A. Thymine B. Adenine C. Guanine D. Cytosine KEY: A, Thymine is present in DNA but not in RNA. Both DNA and RNA are composed of a mix of four bases. However, only three of the four bases are common to both nucleic acids - adenine, guanine, and cytosine. In DNA, the fourth base is thymine; in RNA, thymine is replaced by uracil. 14. In a new born presenting with refusal to feeds and irritability, a deficiency of Cystathionine –β- synthase has been diagnosed, which of the following compounds is expected to be elevated in blood? A. Serine B. Glutamate C. Homocysteine D. Valine KEY: C
  • 20. NEB/PMDC STEP-1 PAST PAPERS DR.ASIF ALI KHAN YANGTZE UNIVERSITY 15. A 3 months old girl is developing cataracts. Other than having social smile or being able to track objects visually, all other aspects of the girl examination are normal. Test on baby’s urine are positive for reducing sugar but negative for glucose. Which enzyme is most likely deficient? A. Aldolase A B. Aldolase B C. Fructokinase D. Galactokinase E. Galactose-1 P uridyltransferase KEY: D, The girl is deficient in galactokinase and is unable to appropriately phosphorylate galactose. Galactose accumulates in the blood (and urine). In the lens of the eye, galactose is reduced by aldose reductase to galactitol, a sugar alcohol, which causes osmotic effects that result in cataract formation. Deficiency of galactose 1-phosphate uridylyltransferase also results in cataracts but is characterized by liver damage and neurologic effects. Fructokinase deficiency is a benign condition. Aldolase B deficiency is severe, with effects on several tissues. Cataracts are not typically seen. 16. Meat restriction for 5-6 years + Anemia – Vitamin B12 deficiency 17. A 76 years old woman who is bed bound in a nursing home begins to develop swelling of her left leg. She is evaluated with venous Doppler ultrasound and is found a deep vein thrombosis. She is immediately started on heparin to further prevent the clot from enlargement. Heparin is an example of a A. Sphingolipid B. Cerebroside C. Ganglioside D. Glycoseaminoglycan E. Prostaglandin 18. Urea cycle and citric acid cycle are linked by A. Glutamate B. Fumarate C. Oxaloacetate D. Ketoglutarate E. Argininosucciate 19. Hyperammonemia lead to brain damage due to depletion of amphibo intermediate A. Oxaloacetate B. Fumarate C. Malate D. Citrate E. Ketoglutarate
  • 21. NEB/PMDC STEP-1 PAST PAPERS DR.ASIF ALI KHAN YANGTZE UNIVERSITY 20. A 55 year old male significantly reduced reabsorption of bile salts resulting from the action of bile salt- binding resins. This expected to lead to a significant decrease in which of the following steroid compound A. Aldosterone B. Cholesterol C. Cortisole D. Pregnenolone E. Progesterone 21. A teenager, concerned about his weight, attempts to maintain a fat-free diet for a period of several weeks. If his ability to synthesize various lipids were examined, he would be found to be most deficient in his ability to synthesize: A. cholesterol B. glycolipids C. phospholipids D. prostaglandins E. triacylglycerol KEY: D, Prostaglandins 22. Patient with H/O recurring attacks of pancreatitis, eruptive xanthomas and increased plasma triglycerides level (Hypertriglyceridemia) associated with chylomicrons A. Apolipoprotein E deficiency B. Lipoprotein lipase deficiency C. … D. … E. … KEY: B 23. Two enzymes have similar V max, but enzyme 1 has lower Km as compared to enzyme 2, this means: A. Less substrate is required by enzyme 1 to attain v max/2 B. Enzyme 1 has more active sites C. Enzyme 2 has lesser number of regulatory sites D. Enzyme 2 has lesser number binding sites E. Enzyme 2 is slowed down by substrate KEY: A
  • 22. NEB/PMDC STEP-1 PAST PAPERS DR.ASIF ALI KHAN YANGTZE UNIVERSITY Pharmacology 1. A 60 years old man had a myocardial infarction, which of the following should be used to prevent life- threatening arrhythmias that can occur post-myocardial infarction in this patient A. Digoxin B. Flecainide C. Procainamide D. Quinidine E. Metoprolol KEY: E Key: B - B-Blockers, such as metoprolol prevent cardiac arrhythmias that occur subsequent to a myocardial infarction. None of the other drugs has been shown to be particularly effective in preventing post infract arrhythmias 2. Chills, fever, and muscle aches are common reactions to which one of the following antiviral drugs? A. Acyclovir B. Gancyclovir C. Oseltamivir D. Interferon E. Ribavirin KEY: D 3. A 57-year-old man is being treated for an atrial arrhythmia. He complains of headache, dizziness, and tinnitus. Which one of the following antiarrhythmic drugs is the most likely cause? A. Amiodarone B. Procainamide C. Propranolol D. Quinidine E. Verapamil KEY: D - The clustered symptoms of headache, dizziness, and tinnitus are characteristic of cinchonism, 4. We have a patient with essential hypertension and unusually high circulating catecholamine level. Our goal is to block α- and β-adrenergic receptors using just one drug. Which of the following is capable to do that A. Labetalol B. Metoprolol C. Nadolol D. Pindolol E. Timolol KEY: A - Labetalol (Alpha + Beta blocker)
  • 23. NEB/PMDC STEP-1 PAST PAPERS DR.ASIF ALI KHAN YANGTZE UNIVERSITY 5. Which of the following agent(s) are associated with gingival hyperplasia? A. Beta blockers B. ACE inhibitors C. Phenytoin D. Statins E. Carbamazepine KEY: C, Phenytoin 6. A 16 years old boy presents to the ambulatory care clinic with persistent dry cough. He had cleaned out a barn attic that was full of bats about 2 week ago and has had the cough for 5 days. The physician suspects histoplasmosis, which would be the best treatment for this patient. A. Albendazole B. Fluconazole C. Ciprofloxacin D. Ribavirin E. Penicillin-G KEY: B - Fluconazole Histoplasmosis is a fungal infection caused by the dimorphic fungus Histoplasma capsulatum. Although it is found in the soil of the entire Ohio River basin, spores appear to be more concentrated in bird and bat droppings. This patient likely stirred up a large number of spores while cleaning an area frequented by bats, enough spores to cause an infection. A common treatment for less serious infections of histoplasmosis is ketoconazole or fluconazole, Amphotericin B for more serious systemic infections. MCQ copied from Lippincott's Illustrated Q&A Review of Pharmacology: page 206, chapter.6, MCQ #40 7. A 32 year old male presents to an outpatient clinic with a 5-day history of productive cough, purulent sputum, and shortness of breath. He is diagnosed with community Acquired pneumonia (CAP). It is noted that this patient has a severe Ampicillin allergy (anaphylaxis). Which of the following would be an acceptable treatment for this patient A. Levofloxacin B. Ciprofloxacin C. Penicillin-V D. Nitrofurantoin KEY: A – Streptococcus pneumoniae is a common cause of CAP, and the respiratory fluoroquinolones levofloxacin and Moxifloxacin provide good coverage. Ciprofloxacin does not cover S. pneumoniae well and is a poor choice for treatment of CAP. Penicillin would be a poor choice due to allergy. Nitrofurantoin has no clinical utility for respiratory tract infections.
  • 24. NEB/PMDC STEP-1 PAST PAPERS DR.ASIF ALI KHAN YANGTZE UNIVERSITY 8. A 40 years old woman on chemotherapy for ovarian carcinoma, developed hemorrhagic Cystitis. Which of the following anticancer drugs may be responsible for it? A. Methotrexate B. Cyclophosphamide C. 5-Fluorouracil D. Cytarabine E. Vinblastine KEY: B 9. Drug of choice for Cisplatin induced emesis is A. Metoclopramide B. Domperidone C. Ondansetron D. Octreotide KEY: C 10. Which of the following is euglycemic drug A. Chlorpropmide B. Metformin C. Glyburide D. Insulin E. Rapaglinide KEY: B 11. Carbidopa is used in the treatment of Parkinson’s disease because it A. Is a precursor of L-dopa B. Dopaminergic receptor agonist C. Prevents peripheral biotransformation of L-dopa D. Prevents break down of dopamine E. Prevents in decreased concentration of L-dopa in the nagrostratium KEY: C 12. Leukotriene inhibitor – Zafirleukast 13. Cinchonism – Quinidine
  • 25. NEB/PMDC STEP-1 PAST PAPERS DR.ASIF ALI KHAN YANGTZE UNIVERSITY 14. The macrolide antibiotics act by binding with A. 30-S ribosomal subunit B. 50-S ribosomal subunit C. 60-S ribosomal subunit D. 70-S ribosomal subunit E. 90-S ribosomal subunit KEY: B 15. Antihypnozoite – Primaquine 16. A 28 year old diabetic pregnant should be treated with A. Sulphonyl urea B. Biguanides C. Insulin D. Setagliptin 17. Benzodiazepine – Clonazepam 18. Insulin – Hypoglycemia 19. T4 conversion to T3 – Propylthiouracil (PTU) 20. Isoniazid – Pyridoxine 21. Taxi driver signs symptoms of allergy – Fexofenadine 22. Carbidopa – Inhibit L-dopa in periphery 23. Which of the following steroids can be administered for maturity of surfactant in preterm labour. A. Beclomethasone B. Betamethasone C. Prednisolone D. Hydrocortisone KEY: B 24. Which of the following agents is useful in treatment of malignant hyperthermia A. Baclofen B. Diazepam C. Dantrolene D. Tizanidine E. Tubocurarine KEY: C 25. Which of the following insulin preparation has the longest duration of action A. Insulin Detemir B. Insulin aspart C. NPH insulin D. Glulisine KEY: A
  • 26. NEB/PMDC STEP-1 PAST PAPERS DR.ASIF ALI KHAN YANGTZE UNIVERSITY 26. A 12 year old girl with a childhood history of asthma complained of cough, dyspnea, and wheezing after visiting a riding stable. Her symptoms became so severe that her parents brought her to the emergency room. Which of the following is the most appropriate drug to rapidly reverse her bronchoconstriction A. Inhaled fluticasone B. Inhaled Beclomethasone C. Intravenous propranolol D. Inhaled albuterol E. Oral theophylline KEY: D –Inhaled albuterol - inhalation of a rapid-acting B2 agonist, such as albuterol, usually provides immediate bronchodilation. 27. Pinpoint pupil – naloxone 28. Antifungal drug used for systemic fungal infection A. Griseofulvin B. Clotrimazole C. Amphotericin B D. Econazole E. Terbinafine KEY: C 29. Case scenario of Pulmonary Edema, ask about drug of choice – Furosemide 30. Case scenario of hypertensive on drugs, patient develop dry cough – Enalapril 31. A 22-year old man, who frequently backpacks, complains of diarrhea and fatigue. Examination of stool specimens shows bi-nucleate organisms with four flagellae. Which one of the following drugs would be effective in treating this patient’s infestation A. Metronidazole. B. Quinidine. C. Pentamidine. D. Sulfadoxine. E. Stibogluconate KEY: A 32. Case scenario of a child using fluoroquinolones – Arthropathy 33. Glucocorticoid – osteoporosis 34. A 45 year old female suffering from acute gouty attack of left big toa is in need of a drug which should not disturb her GIT A. Allopurinol B. Colchicine C. Indomethacin D. Acetaminophen KEY: A
  • 27. NEB/PMDC STEP-1 PAST PAPERS DR.ASIF ALI KHAN YANGTZE UNIVERSITY 35. Which of the following sulfonamide is used in the treatment of ulcerative colitis A. Sulfadiazine B. Sulfasalazine C. Sulfadimidine D. Sulfadoxine E. Mefenid KEY: B Physiology 1. In athletes, physical fitness is more closely correlated with A. Maximal pulse rate than with resting pulse rate B. Maximal oxygen uptake (VO2max) than with resting oxygen uptake C. Maximal minute ventilation than with maximal cardiac output D. Size of muscles E. Resting sympathetic tone than with maximal cardiac output KEY: 2. If muscle strength is increased with restrictive training, which of the following conditions is most likely to occurs A. Decrease in the number of myofibrils B. Increase in mitochonchondrial enzymes C. Decrease in the components of phosphagen system D. Decrease in stored triglycerides E. Hyperkalemia KEY: 3. The potassium in the renal tubule is A. Only filtered and secreted B. Only filtered and reabsorbed C. Filtrated, reabsorbed, and secreted D. Only secreted E. Only filtered KEY: C 4. During in the passage of an intravenous catheter, numerous endothelial cells are dislodged from lining of popliteal vein. What substance allows platelet adhesion to the exposed collagen A. Factor VIII B. Factor IX C. Fibronectin D. Tissue factor E. Von Willebrand factor KEY:
  • 28. NEB/PMDC STEP-1 PAST PAPERS DR.ASIF ALI KHAN YANGTZE UNIVERSITY 5. The hormone which has contraceptive role is: A. Luteinizing hormone B. Prolactin C. Testosterone D. Estrogen E. Human chorionic gonadotropin KEY: 6. A substance that is filtered, but not secreted or reabsorbed (substance X), is infused into a volunteer until a steady state plasma level of 0.1 mg/mL is achieved. The subject then empties his bladder and waits one hour, at which time he urinates again. The volume of urine in the second specimen is 60 mL and the concentration of substance X is 10 mg/mL. What is the glomerular filtration rate (GFR) in this individual? A. 30 mL/min B. 60 mL/min C. 100 mL/min D. 300 mL/min E. 600 mL/min KEY: C 7. A 45 years obese man feeling hungry and thirsty most of the time, which increase frequency of urination is recently diagnosed to have type 2 Diabetes mellitus. Considering the problem at glucose transporters the following tissue is likely to be affected the most A. Brain and liver B. Liver and intestine C. Muscle and adipose D. Red blood cell and brain E. Small intestine and brain KEY: D 8. Growth Hormone secretion increase – Exercise 9. Splitting of S2 during inspiration occurs due to A. Delayed closure of pulmonic valve B. Early closure of pulmonic valve C. Early closure of tricuspid valve D. Early closure of mitral valve KEY: A 10. 1st degree Heart block – syncope 11. Most sensitive cardiac marker – Troponin T 12. Presbyopia – Convex lenses
  • 29. NEB/PMDC STEP-1 PAST PAPERS DR.ASIF ALI KHAN YANGTZE UNIVERSITY 13. In a patient with a urine flow rate of 1 mL/minute, the tubular fluid with the lowest osmolarity would be found in the A. Beginning of the proximal tubules B. End of the cortical collecting tubule or duct C. End of the papillary collecting tubule or duct D. macula densa E. tip of the loop of Henle KEY: D, Tubular fluid first becomes hypotonic toward the end of the thick ascending limb of the loop of Henle and will therefore be hypotonic by the macula densa (which is the border between the thick ascending limb and the distal convoluted tubule). 14. A woodworker operating a bandsaw accidently injures his wrist, severing his radial artery and producing severe hemorrhage. As he loses blood, his body tries to compensate for the developing hypotension by increasing sympathetic outflow. The postganglionic signals carrying the impulses to constrict his arterioles are transmitted along which of the following fiber types? A. A & delta fibers B. B fibers C. C fibers D. Ia fibers E. Ib fibers KEY: C, Type C - Post Ganglionic Autonomic fib-= Type IV (Unmyelinated)--Slow Pain & Temp 15. Muscle spindle: A. Control body posture B. Maintains the length of skeletal muscle bundle C. Regulates muscle activity D. Is the sensory stretch receptor E. Relays motor impulse to the skeletal muscle KEY: B 16. Secretion of oxytocin is increased by A. milk ejection B. suckling of the cervix C. Increased prolactin levels D. Increased extracellular fluid (ECF) volume E. Increased serum osmolarity KEY: B, Suckling and dilation of the cervix are the physiologic stimuli for oxytocin secretion. Milk ejection is the result of oxytocin action, not the cause of its secretion. Prolactin secretion is also stimulated by suckling, but prolactin does not directly cause oxytocin secretion. Increased extracellular fluid (ECF) volume and hyperosmolarity are the stimuli for the secretion of the other posterior pituitary hormone, antidiuretic hormone (ADH).
  • 30. NEB/PMDC STEP-1 PAST PAPERS DR.ASIF ALI KHAN YANGTZE UNIVERSITY 17. Maintained lactation – Prolactin 18. Increase GFR…..efferent constriction 19. A 72-year-old woman with insomnia participates in a sleep study. As part of the study protocol, she has EEG leads attached, then goes to sleep. At one point during the evening, 12-16 Hz sleep spindles and K- complexes are observed. Which of the following stages of sleep is associated with this pattern? A. REM B. Stage 1 C. Stage 2 D. Stage 3 E. Stage 4 KEY: C, Explanation 1. Stage 2 has more theta waves than stage 1 and is associated with sleep spindles (short bursts of 12-16 Hz activity) and K-complexes (high amplitude slow waves with superposed sleep spindles) on the electroencephalogram 2. Transient large amplitude potentials in the occipital areas (ponto-geniculo-occipital [PGO] spikes) are associated with REM sleep (choice A). 3. Stage 1 (choice B), or drowsiness, is characterized by the attenuation of alpha rhythm (8-13Hz) and the appearance of 4-7-Hz theta waves. 4. Stages 3 (choice D) and 4 (choice E), or slow wave sleep, are characterized by high amplitude slow waves, especially in the delta (< 4 Hz) frequency range. Pathology 1. A 35 years female who got her cervical smear done, which reveals High Grade Squamous Intraepithelial Lesions (HSIL) which one of the following HPV types will be implicated in this lesion A. 6 and 11 B. 5 and 8 C. 16 and 18 D. 19 and 22 E. 15 and 17 KEY: C, HPV 16 and 18 2. Japanese have a higher rate of stomach cancer and a low rate for colon carcinoma than the U.S. However third generation descendants of Japanese immigrants to U.S have rates of stomach and colon cancer like that of U.S. This particular characteristic supports effects of: A. Environment B. Genetics C. Mutation D. Accidents E. Misinterpretation KEY: A, Environment
  • 31. NEB/PMDC STEP-1 PAST PAPERS DR.ASIF ALI KHAN YANGTZE UNIVERSITY 3. A 5 years old African male was chronically ill had swollen jaw and spleen and liver enlargement. The cells on biopsy were monotonous small non-cleaved lymphocytes. Most likely chromosomal translocation to be found in this disease is A. t(4;22) B. t(9;22) C. t(12;22) D. t(8;14) E. t(x;18) KEY: D - Burkett lymphoma case – t;(8;14) 1. ALL = 12;21 2. AML= 17;15 3. CML = 9;22 4. Burkitt lymphoma =8;14 5. Mantle cell 11;14 6. Follicular cell = 14;18 4. Deposition of calcium in nonviable or dying tissue in the presence of normal serum calcium levels is called A. Metastatic calcification B. Dystrophic calcification C. Intracellular calcification D. Calcium excess E. None of the above KEY: B 5. Most common karyotype seen in Down’s syndrome is: A. Trisomy 13 B. Trisomy 21 C. Trisomy 16 D. Monosomy 21 E. Trisomy 12 KEY: B 6. Insertion or deletion of one or two base pairs altering the reading frame of the DNA strand is known as Which of the following would result in a A. Point mutation B. Frame shift mutation C. Trinucleotide repeat mutation D. Missence mutation E. Single nucleotide polymorphism KEY: B - Frame shift mutations lead to lease effect when they lead to an insertion or deletion of 3 bases, as due to our codon being triplet this doesn’t shift the reading frame. However, if the codon thus lost or added is a very important one in the gene, there could be serious consequences.
  • 32. NEB/PMDC STEP-1 PAST PAPERS DR.ASIF ALI KHAN YANGTZE UNIVERSITY 7. A 30 years old male patient presented in outdoor with complain of oral ulceration. The consultant suspects a fungal infection. His medical record also showed that he is suffering from acquired immune deficiency syndrome (AIDS). What is the most likely cause of his oral lesions? A. Cryptococcosis B. coccidiomycosis C. Candidiasis D. Histoplasmosis E. Tinea KEY: C 8. A specific form of necrosis, also visible under light microscope, which occur after deposition of immune complexes in the walls of arteries is A. Coagulative necrosis B. Caseous necrosis C. Fibrinoid necrosis D. Fat necrosis E. Liquefactive necrosis KEY: C 9. Cytotoxic T-cells help in A. Destroying tissue invaded by viruses B. Producing antibodies C. Activation of mast cells D. Regulating T-cell activity E. Suppressing the T-cell activity KEY: A 20. A person with endemic goiter has A. High level of Plasma TSH B. High level of Plasma thyroxine C. High level of Plasma triiodothyronine D. Low production of thyroglobulin E. Low level of growth hormone KEY: A, High level of Plasma TSH (Decreased T3, T4 and Increased TSH) 10. A 42 years old obese women presented to the emergency room with complain of worsening nausea, vomiting and epigastric pain. The blood chemistry revealed a high Amylase level. Most likely diagnosis is: A. Acute appendicitis B. Acute Pancreatitis C. Gastritis D. Renal colic E. Viral hepatitis KEY: B,
  • 33. NEB/PMDC STEP-1 PAST PAPERS DR.ASIF ALI KHAN YANGTZE UNIVERSITY 11. Sterilization by autoclaving is best confirmed by A. Processing at temp of 120 o C B. Ten minutes incubation C. 10 lb/in2 pressure D. 90% air removal E. Bowie Dick test KEY: A 12. Incidence of Bile duct is increased in individuals with infestation by A. E. histolytica B. Schistosoma mansoni C. Fasiola hepatica D. Echinococcus E. Clonorchis sinensis KEY: E, Clonorchis sinensis (Cholangiocarcinoma) 13. The test that screens the extrinsic pathway is A. Prothrombin time (PT) B. Activated partial thromboplastin time (aPTT) C. Thrombin time D. Urea solubility test E. Clot lysis time KEY: A 14. A 65-year-old female with renal failure presents for hemodialysis. She is found to be anemic and is given a dose of erythropoietin along with her usual vitamin and mineral supplements. Erythropoietin stimulates which of the following intermediates in hematopoiesis? A. Basophilic erythroblasts B. Colony forming units-erythroid C. Multipotential stem cells D. Proerythroblasts E. Reticulocytes KEY: B, The colony forming unit-erythroid (CFU-E) is a unipotential stem cell that develops from a burst forming unit-erythroid (BFU-E), which develops eventually from the multipotential stem cell. The BFU-E is somewhat responsive to erythropoietin, but the CFU-E is completely dependent on erythropoietin. Erythropoietin is normally released from the kidney in response to hypoxic or anemic conditions. Its half-life is about 3-6 hours. Clinically it takes 5 days to see reticulocyte formation in the peripheral blood following erythropoietin administration
  • 34. NEB/PMDC STEP-1 PAST PAPERS DR.ASIF ALI KHAN YANGTZE UNIVERSITY 15. A 52 years old female having chronic viral hepatitis present with pain right hypochondrium. Ultrasound abdomen shows a space occupying lesion in the right lobe. The most likely diagnosis is A. Hepatic adenoma B. Hepatocelullar carcinoma C. Angiosarcoma D. Cyst of echinococcus granulossus E. Amoebiasis KEY: B, Hepatocellular carcinoma 16. A women smokes one pack/day, she drinks at least 5 cups of coffee/day. Her mother died of breast cancer. Which is the greatest risk factor for breast cancer in this patient A. Birth control pills B. Cigarette smoking C. Family history D. Caffeine intake E. Nulliparity KEY: C 17. A 27 years old man presents with a mass at the base of skull extending down in the oral cavity, which on biopsy was diagnosed as being a yolk sac tumour. Which of the following tumour marker is most likely to be increased in this patient’s serum as a result of being secreted from the cells of this tumour? A. Acid phosphatase B. Alpha-fetoprotein (AFP) C. Alkaline phosphatase D. Beta-human chorionic gonodotriphin (Beta HCG) E. Calcitonin KEY: B, Alpha-fetoprotein (AFP) » AFP is Measured in routine screening around 15-18 weeks (second trimester) » AFP and amniocentesis can help detect: neural tube defects and trisomies » AFP is synthesized in the fetal liver, GI tract, and yolk sac » Alpha-fetoprotein (AFP) is tumor marker for Hepatocellular carcinoma (HCC), Hepatoblastoma, Yolk sac tumor of the ovary (endodermal sinus tumor), Mixed germ cell tumor » Transient elevation during pregnancy » AFP increased in:  abdominal wall defects: anacephaly, meningomyelocele, encephalocele  neural tube defects: gastroschisis, omphalocele  twin gestations: if the measures twice the median value then evaluate by ultrasound and amniocentesis » AFP decreased in: associated with:  Trisomy 21 (Downs Syndrome)  Trisomy 18 (Edwards Syndrome)  Trisomy 13 (Patau Syndrome)
  • 35. NEB/PMDC STEP-1 PAST PAPERS DR.ASIF ALI KHAN YANGTZE UNIVERSITY 18. Aortic dissection occurs in a tall 31 years old male with long fingers. His ophthalmic examination reveals ocular lens dislocation. These features are found in A. Down’s syndrome B. Glycogen storage disease C. Neurofibromatosis D. Marfan syndrome E. Tay sachs disease KEY: D 19. A 4 year old boy presents with fever, an abdominal mass which has grown rapidly over the last four months accompanied by weight loss. Ultrasound confirms a solid space occupying lesion in the kidney. The diagnosis is: A. Renal cell carcinoma B. Transitional cell carcinoma of kidney C. Squamous cell carcinoma of renal pelvis D. Wilms tumor E. Hydronephrosis KEY: D 20. Which of the following organism is responsible of causing infective endocarditis after skin infection A. S. Aureus B. Strep. Viridians C. Haemophilus D. Eikenella E. Kingella KEY: A 21. A 35 years old truck driver presented in STI clinic with complain of painless of ulceration of on external genitalia. On examination there is chancre formation with inguinal and femoral lymphadenopathy. A. Neisseria gonorrhoea B. Human papilloma virus C. Chlamydia trachomatis D. Mycobacterium genitalium E. Treponema pallidum KEY: E
  • 36. NEB/PMDC STEP-1 PAST PAPERS DR.ASIF ALI KHAN YANGTZE UNIVERSITY 22. A 30 years old patient presented in emergency with multiple abscesses on different parts of his body. He also gave the history of fever with evening rise and productive cough. Blood picture showed lymphocytic predominance. Acid fast stain is also positive. X-ray showed an endobrochial mass. What is most likely diagnosis A. Pneumonia B. Cellulitis C. Nocardiasis D. Kaposi sarcoma E. Sarcoidosis KEY: C 23. A group of students developed vomiting, abdominal pain and diarrhea six hours after having lunch consisting of Chinese rice at restaurant. The most likely organism involved is: A. Clostridium botulinum B. Vabrio cholera C. Bacillus cereus D. Enterotoxicgenic escherchia coli E. Clostridium perfringens KEY: C 24. Malignancy – Telomerase 25. Peripheral cyanosis – Primary Reynaud Phenomenon 26. Cells against TB – CD4 T lymphocyte 27. Endothelium platelet aggregation – Von Willebrand Factor 28. Pregnant lady with hepatitis , how to confirm diagnosis — SGOT 29. A 30 years old active sex worker presented in the outdoor of tehsil headquarters hospital complaining of pain, itching and prulent vaginal discharge. Female medical officer took the sample of pus for lab examination also. Which organism is most likely cause of these symptoms A. C. Trachomatis B. H. Papilomavirus-2 C. H. Papilomavirus-7 D. Trichomonas Vaginalis E. U. Urealyticum KEY: D Systemic fungal 30. Beta Thalassemia – Hypochromic and Microcytic anemia 31. Increased An-ion gap is seen in – Metabolic Acidosis 32. Infant with vomiting – Pyloric Stenosis 33. Ovary mass with Caseous necrosis – Tuberculosis
  • 37. NEB/PMDC STEP-1 PAST PAPERS DR.ASIF ALI KHAN YANGTZE UNIVERSITY 34. A 70-year-oltl Negroid man from Makkran coast was stabbed on the upper arm, he received emergency treatment and his wound healed. He developed a raised hypertrophic scar with boundaries beyond the original wound, and it did not regress. Which of the following terms best describes this condition: A. Cicatrix B. Keloid C. Callus D. Granulation tissue E. Wound KEY: B 35. Myasthenia Graves – Abs against Ach receptors 36. Vasodilation – Septic shock 37. A transudate is edema fluid A. Rich in proteins B. Rich in neutrophils C. Rich in albumin D. Low in proteins E. Rich in fibrinogen KEY: D 38. A 50 years old non-smoker lady with alpha-1 antitrypsin deficiency developed severe lung disease and requires a double lung transplant. What is the most likely lung lesion she is having? A. Adenocarcinoma B. Bullous emphysema C. Centriacinar emphysema D. Interstitial emphysema E. panacinar emphysema KEY: E 39. Normal flora of intestine – E. coli 40. UV radiations – Oxygen free radicals 41. Specific with Cardiac Muscle – CK-MB 42. Steroid – increase neutrophil 43. Esophageal varices…cirrhosis 44. Allergy – IgE 45. transplant rejection – cytotoxic t cell
  • 38. NEB/PMDC STEP-1 PAST PAPERS DR.ASIF ALI KHAN YANGTZE UNIVERSITY Community Medicine 1. Low birth baby is one who has weight at birth less than A. 1.5 kg B. 2 kg C. 2.5 kg D. 3 kg E. 3.5 kg » KEY: C, 2.5 kg 2. A mother bought her six months old child to a BHU. She is worried about the growth of her child. The best single measure for assessing the physical growth in this age A. Weight for age B. Height for age C. Chest circumference D. Mild upper arm circumference E. Head circumference KEY: A, weight for age 3. BMI is calculated with the help of A. Age and weight B. Height and age C. Height and weight D. Mid upper area circumference and height E. Mid upper area circumference and weight KEY: C, height and weight 4. Most sensitive indicator of health of status of population is A. Crude death rate B. Infant mortality rate C. Maternal mortality rate D. Child mortality rate E. Dependency rate KEY: B, Infant mortality rate 5. In the data 15, 2, 9, 10, 18, 14 and 72 the median is A. 10 B. 14 C. 72 D. 15 E. 18 KEY: B, 14
  • 39. NEB/PMDC STEP-1 PAST PAPERS DR.ASIF ALI KHAN YANGTZE UNIVERSITY 6. Case fatality rate – 25% 7. Concept of primary Health Care and Health for All by 2000 was evolved in 1978 at Alma Ata is it A. China B. Russia C. America D. Egypt E. Kazakhstan KEY: E, Kazakhstan - The Declaration of Alma-Ata was adopted at the International Conference on Primary Health Care (PHC), Almaty (formerly Alma-Ata), Kazakhstan (formerly Kazakh Soviet Socialist Republic), 6–12 September 1978. It expressed the need for urgent action by all governments, all health and development workers, and the world community to protect and promote the health of all people. It was the first international declaration underlining the importance of primary health care. 8. Weaning should be started usually at A. Birth B. 4-6 months C. 1 year D. 1 month E. At any time KEY: B, 4-6 months 9. Pakistan is attached with WHO regional HQ of: A. South East Asia region B. Africa region C. Western Pacific region D. Eastern Mediterranean region E. None of the above KEY: A, South East Asia region 10. COCP – Ethylene Estradiol 11. Neonatal tetanus is an important preventable cause of neonatal mortality that account for 7% of neonatal deaths worldwide. This could be prevented by: A. Vaccination of women in reproductive age group B. Vaccination of newborn’s C. Vaccination of pregnant ladies D. Vaccination of females of child bearing age E. Vaccination of non-pregnant females KEY: D, Vaccination of females of child bearing age 12. Vaccination for those going abroad Saudi to perform hajj A. Meningitis B. Tetanus
  • 40. NEB/PMDC STEP-1 PAST PAPERS DR.ASIF ALI KHAN YANGTZE UNIVERSITY C. Pneumonia D. Cerebral Malaria E. Influenza KEY: A, Meningitis vaccination 13. A 46 year old male worker of a textile factory presented with 02 months history of cough, fever and hemoptysis. The exposure to cotton dust in the factory lead to a this condition known as A. Bagassosis B. Byssinosis C. Anthracosis D. Silicosis E. Asbestosis KEY: B, Byssinosis 14. Limitation of freedom movements of apparently healthy individuals for the period of not more than the longest incubation period of particular disease is A. Isolation B. Quarantine C. Surveillance D. Segregation E. All of the above KEY: B, Quarantine - Who has been exposed to a case of communicable disease for maximum incubation period, is termed as Quarantine 15. Dental caries is due to lack of A. Chlorine B. Fluorine C. Iodine D. Iron E. Calcium KEY: B, Fluorine 16. Bone breaking disease also known as A. Typhus fever B. Dengue fever C. Hay fever D. Congo hemorrhagic fever E. Yellow fever KEY: B, Dengue fever
  • 41. NEB/PMDC STEP-1 PAST PAPERS DR.ASIF ALI KHAN YANGTZE UNIVERSITY 17. A 26 years old female patient, at 3rd day of delivering a baby developed cyanosis, shortness of breath and orthopnea. What is the most likely underlying pathology causing these signs and symptoms -- Amniotic Fluid Embolism 18. 28 year old healthy mother of 2 with fear side effects of IUCD and COCP what should advice after proper counseling – COCP 19. Hospital waste is best disposed of by A. Composting B. Dumping C. Incineration D. Burial E. None of the above KEY: C, Incineration 20. If annual growth of population is 2.0% than it will be double in A. 35 years B. 28 years C. 23 years D. 47 years E. 50 years KEY: A, 35 years 21. If the population of a country grows at a rate of approximately 5 percent per year, the number of years required for the population to double is closest to A. 5 years B. 10 years C. 15 years D. 25 years E. 35 years KEY: C, 15 years 22. Shortly after a barbeque, Medical students of a medical college came back to their rooms and most of them (62 out of 74 students) experienced acute vomiting and diarrhea. This epidemic maybe classified as A. Point source B. Propagative C. Common source D. Serial direct E. None of the above KEY: A Point source
  • 42. NEB/PMDC STEP-1 PAST PAPERS DR.ASIF ALI KHAN YANGTZE UNIVERSITY 23. An operon is best described as A. A constitutively expressed gene system B. An unregulated gene system C. A co-ordinately regulated gene system D. A gene that produces monocistronic messenger RNA E. A combination of gene expressed independent of each other but under influence of KEY: 24. Geriatrics deals with the people A. Neonates B. Aldosence C. Teenagers D. Old age E. Orphan KEY: D, Old age 25. After a disaster, TRIAGE is an approach for A. Provision of food and shelter to the people B. Selection of injured who can be benefited the most from urgent medical treatment C. Rehabilitation of the affected people D. Removal of dead bodies E. Immunisation of people KEY: A, - The approach of rapidly classifying the injured on the basis of severity of their injuries and likelihood of their survival with prompt medical intervention after disaster is called as Triage 26. Sporadic epidemic Forensic Medicine 1. Munaqillah – Fracture with dislocation 2. Phossy jaw (mandibular necrosis) is associated with which of the following occupation is seen in which poisoning? A. Felt hat makers B. Leather manufacturers C. Match makers D. Shipyard workers E. Synthetic drivers KEY: C
  • 43. NEB/PMDC STEP-1 PAST PAPERS DR.ASIF ALI KHAN YANGTZE UNIVERSITY 3. Hatter shake is observed in chronic poisoning with? A. zinc B. Mercury C. Lead D. copper E. Arsenic KEY: B 4. Phosphorus poisoning most commonly seen in workers in the – Match industry 5. Shajjah-i-Munqaliba means: A. Causing Shajjah with exposing bone of the victim but without causing fracture B. Causing Shajjah by fracturing the bone of the victims but without dislocation C. Causing Shajjah by both fracturing and dislocation of bone of the victims D. Causing Shajjah by causing fracture of the skull and wound touches the membranes of the brain E. Causing Shajjah by causing fracture of the skull and the wound ruptures the membrane of the brain KEY: C, Explanation 1. Shajjah-i-Khafifa Causing Shajjah without exposing bone of the victim 2. Shajjah-i-Mudihah: Causing Shajjah with exposing bone of the victim but without causing fracture 3. Shajjah-i-Hashimah: Causing Shajjah by fracturing the bone of the victim but without dislocating it 4. Shajjah-i-Munaqqilah: Causing Shajjah by both fracturing and dislocating the bone of the victim 5. Shajjah-i-Ammah: Causing Shajjah by causing fracture of the skull and the wound touches the membranes of the brain 6. Shajjah-i-Damighah: Causing Shajjah by causing fracture of the skull and the wound ruptures the membranes of the brain 6. The direction of fire can be determined most reliably from: A. The nature of the tattooing around entrance wound B. The bullet track in the tissues C. Grease mark D. Collar of abrasion E. Preterm of damage on clothing KEY: 7. Following statement does not depict characteristic of a bullet wound A. Exit wound is usually irregular in shape B. Damage can be produced at a distance away from the original bullet track C. With low velocity gunshot, the main wound occurs due to cavitation phenomenon D. Entry produced a clean-cut hole in skull E. Entry produce a stellate shape wound on head KEY:
  • 44. NEB/PMDC STEP-1 PAST PAPERS DR.ASIF ALI KHAN YANGTZE UNIVERSITY 8. In Pakistan, a medical officer is authorized to perform medicolegal post-mortem examination under authority of following law: A. Section 302 of Pakistan penal code 1869 B. Section 174 of code of criminal procedure 1898, Pakistan C. Section 307 of Pakistan penal code D. Section 274 of code of criminal procedure 1898, Pakistan E. Section 374 of code of criminal procedure 1898, Pakistan KEY: 9. The snake bite of family viperidae is commonly identified by one of the following effect: A. Nephrotoxic B. Neurotoxic C. Mytoxic D. Heamatotoxic E. Vasculotoxic KEY: D 10. Whiplash is which form of injury A. Hyper flexion B. Hyperextension C. Lateral flexion D. Atlanto-axial dislocation E. .. KEY: B 11. Brush burn refers to A. Electric burn B. Lightening burn C. Sliding abrasion D. Pressure abrasion E. .. KEY: C 12. Which one of the following organic acids is made from methanol A. Acetic Acid B. Botanic acid C. Formic acid D. Propanoic acid E. .. KEY: C 13. A man survives after giving dying declaration? A. He has to give oral oath for evidence or something B. His declaration is valid for one year C. Declaration is null and void. D. No value E. ..
  • 45. NEB/PMDC STEP-1 PAST PAPERS DR.ASIF ALI KHAN YANGTZE UNIVERSITY 14. The legal right of consent is conferred to patient under the principle of A. Benevolence B. Autonomy C. Paternalism D. Justice E. Religion 15. First treatment for snake bite is A. Antisnake venom B. Application of tourniquet C. Clean the wound with soap & water D. Local incision & suction E. All KEY: B, Application of tourniquet 16. A 46 year old male worker of a textile factory presented with 02 months history of cough, fever and hemoptysis. The exposure to cotton dust in the factory lead to a this condition known as A. Bagassosis B. Byssinosis C. Anthracosis D. Silicosis E. Asbestosis KEY: B TYPES OF OCCUPATIONAL LUNG DISEASES DISEASE CAUSATIVE AGENT 1. Silicosis » Inhalation of free silica or silicon dioxide 2. Asbestosis » Inhalation of asbestos 3. Byssinosis » Cotton fiber dust 4. Bagassosis » sugarcane dust or fiber 5. Farmer’s lung » moldy hay or grain dust (fungal spore) 6. Siderosis » Iron dust 7. Coal worker’s pneumoconiosis (Anthracosis) » Coal worker 8. Berylliosis » Beryllium
  • 46. NEB/PMDC STEP-1 PAST PAPERS DR.ASIF ALI KHAN YANGTZE UNIVERSITY 17. The demographics cycle has five stages, the late expanding stage of the cycle indicates A. High birth rate and high death rate B. Decreasing birth rate and decreasing death rate C. Decreasing death rate and stationary birth rate D. Low birth rate and low death rate E. High birth rate and stationary death rate KEY: 18. A high school has experienced a series of incidents of sexual harassment in the hallways. One of the health education teacher assigns students to create posters in class encouraging positive peer pressure to prevent this behavior A. Group approach B. Individual approach C. Mass approach D. Personal approach E. Massive approach KEY: 19. The average number of children of a women will have if she were to pass through reproductive years bearing children at the same rate as the women in each age group is A. General fertility rate B. General marital fertility rate C. Total fertility rate D. Gross reproductive rate E. Total marital fertility rate KEY: 20. A researchers collects data to find the prevalence of smoking among school children in Lahore, he is using A. Sampling B. Test of significance C. Inferential statistics D. Descriptive statistics E. Complete enumeration KEY: 21. Orthotolidine test is carried out to A. Estimate the odour in water B. Estimate fluorine content in water C. Estimate chlorine content in water D. Estimate nitrate level of water E. Estimate nitrite level of water KEY:
  • 47. NEB/PMDC STEP-1 PAST PAPERS DR.ASIF ALI KHAN YANGTZE UNIVERSITY 22. Denominator in Crude death Rate is A. Population at risk B. Mild year population C. Total No of births D. Total No of death E. None of the above 15 APRIL 2018 NUMS MCQS OF NEB PMDC STEP-1 Community medicine 1. Case scenario of Cement industry worker reported to you with complaints of cough, dyspnea on exertion and chest pain. His X-ray chest showed 'snow storm' appearance. The diagnosis would be: A. Asbestosis B. Siderosis C. Silicosis D. Aspergillosis E. Byssinossis KEY: C 2. Case scenario of a worker, Ground glass appearance on X ray is seen in A. Asbestosis B. Silicosis C. Anthracosis D. Berylliosis E. Byssinosis KEY: A 3. Night sweats , productive cough — TB 4. Shock after IUCD— uterine perforation 5. Drug stopped mortality— prevalence will be increased 6. Hair tilt— spina biffida 7. Giving penicillin— secondary care 8. Psychotherapy/rehabilitation— tertiary 9. Non immunized child— BCG , OPV, DPT ,measles , Vit A 10. Infection avoided— hand washing 11. Tb diagnosed and OCP— rifampicin
  • 48. NEB/PMDC STEP-1 PAST PAPERS DR.ASIF ALI KHAN YANGTZE UNIVERSITY 12. Needle go to — sharp container 13. Rabies— Ig, vaccine , wound open 14. Calories count— 105 15. Folic acid dose— 0.5/0.4? 16. Niejeria to pak— yellow fever 17. Number of basic health units in Pak — 5400 18. Mumps – Parainfluenza virus Anatomy 1. The nerve which passes through the quadrangular space of the posterior shoulder A. Axillary nerve B. Ulnar nerve C. Median nerve D. Musculocutaneous nerve E. Vagus nerve KEY: A » Axillary nerve passes through the quadrangular space and then winds around the surgical neck of the humerus, it supplies teres minor and deltoid muscle » The quadrangular space is bounded medially by the tendon of the long head of the triceps, laterally by the humerus, superiorly by teres minor, and inferiorly by teres major. The posterior circumflex humeral artery and the axillary nerve traverse this space. The axillary nerve innervates 2 muscles: deltoid and teres minor. 2. Cavernous sinus — facial vein 3. The facial muscle that produces horizontal wrinkles on the forehead is A. Procerus B. Corrugator supercilii C. Frontal belly of occipitofrontalis D. Orbicularis oculi KEY: C , The frontal belly of occipitofrontalis elevates the skin of the forehead (panniculus carnosus) producing horizontal wrinkles on the forehead, thus helping the eyes to open wider and also to produce an expression of surprise. 4. Sole of foot – post tibial artery 5. Outer quadrant breast lymph – pectoral 6. Thyroid derived from – endoderm 7. Parathyroid – cheif cells
  • 49. NEB/PMDC STEP-1 PAST PAPERS DR.ASIF ALI KHAN YANGTZE UNIVERSITY 8. Scavenger cells – microglia 9. PNS myelination – Schwan cells 10. Frozen shoulder muscle – sub scapularis 11. Medial rotation – Erbs palsy 12. Motor area in cerebrum – primary motor 13. Organ of corti – vestibulocochlear 14. Permanent mydriasis— oculomotor nerve 15. Horner syndrome — sympathetic nerve 16. Tongue deviated on right side — right hypoglossal nerve damaged 17. Patellar reflex — L3 L4 18. Pain after appendectomy — iliohypogastric nerve 19. Medial thigh problem after disc herniation— L 4 20. Saturday night palsy — radial nerve 21. Duodenal ulcer— gastro duodenal artery 22. Moving touch along sides — ruffini 23. Half face weakness— same sided facial nerve 24. Nerve between c5 and c6 — c6 Pharmacology 1. interferon — flue like symptoms 2. Cholicine MOA — muscle spindle 3. Heart failure diuretic — spironolactone 4. A 60-year-old man is admitted to the hospital with acute heart failure and pulmonary edema. Which of the following drugs would be most useful in treating the pulmonary edema? A. Digoxin B. Lisinopril C. Dobutamine D. Furosemide E. Spironolactone KEY: D 5. Metallic taste— metronidazole » Metronidazole: it is used for treatment of anaerobic infections and certain ... The side effects include nausea, headache, a metallic taste and loss of appetite.
  • 50. NEB/PMDC STEP-1 PAST PAPERS DR.ASIF ALI KHAN YANGTZE UNIVERSITY 6. A 30 year old female developed antibiotics-induced colitis due to C. difficile; she was given metronidazole to treat it but it proved ineffective. Now she can respond to oral A. Amoxicillin B. Erythromycin C. Chloramphenicol D. Doxycycline E. Vancomycin KEY: E 7. One of the following oral hypoglycemic agent help the diabetic patient by promoting the release of endogenous insulin A. Acarbose B. Metformin C. Glipizide D. Pioglitazone E. Miglitol KEY: C 8. Hyperthyroidism – PTU 9. Patient with history of Prolactinoma treated by – Bromocriptine 10. Asthma by which drug— aspirin 11. Bradycardia due to vagal— atropine 12. Short procedure anesthesia— propofol 13. Sublingual drug— nitroglycerine 14. Angina Rx— Nifedipine 15. Asthma prophylaxis — cromolyn sodium 16. Case fatality — 10% 17. Antifungal for mouth — Nystatine 18. Oral antifungal – Griseofulvin 19. Antiepileptic drug interaction— sodium valproate 20. Anti-Peptic ulcer — PPI 21. Opioids — naloxone 22. Drug give with atropine — diphenoxylate 23. Add with amoxicillin — clavulanic acid 24. In pregnancy — amoxicillin 25. Diabetic ketoacidosis (DKA) treatment — regular insulin 26. Phyrngitis — Doxycycline 27. Tetracycline to give in renal insufficiency — Doxycycline or oxytetracycline?
  • 51. NEB/PMDC STEP-1 PAST PAPERS DR.ASIF ALI KHAN YANGTZE UNIVERSITY 28. Pregnancy combination— ticarcillin and tobramycin 29. SVT – adenosine 30. Potassium channel blocker – Amiodarone 31. Hyperkalemic hypertensive patient – thiazide diuretic 32. A 36-year old woman who is pregnant is diagnosed with hypertension. Which of the following agents would be the most suitable to treat her hypertension? A. Atenolol B. alpha-methyl dopa C. Nifedipine D. Clonidine E. Lisinopril KEY: B 33. A 67-year-old man is hospitalized recovering from a left wall myocardial infarction. He begins to show signs of fluid retention. His doctors want to start a drug regimen for congestive heart failure, including either an ACE inhibitor or an angiotensin receptor blocker (ARB). ACE inhibitors and ARBs treat hypertension in a similar fashion and have similar side effects. Which of the following is a side effect of ACE inhibitors only – dry cough 34. Tremors is side effect of—salbutamol/Salmeterol 35. Which of the following anti-seizures drug is most likely to elevate the serum concentration of other drugs administration concomitantly A. Carbamazepine B. Diazepam C. Phenobarbitone D. Phenytoin E. Valproic acid KEY: E 36. Abrupt withdrawal of anti-seizure drugs can result in increases in seizure frequency and severity. Withdrawal is most easily accomplished if the patient is treated with A. Carbamazepine B. Clonazepam C. Ethosuximide D. Phenobarbitone E. Phenytoin KEY: A 37. Case scenario of Parkinson’s disease, Antiviral used in the treatment of tremors A. Ribavirin B. Acyclovir C. Amantadine D. Oseltamivir KEY: C
  • 52. NEB/PMDC STEP-1 PAST PAPERS DR.ASIF ALI KHAN YANGTZE UNIVERSITY Physiology 1. Cardiac markers — 3 MCQS 2. Transudate — low protein in cells 3. L/S ratio— lung maturity 4. Running, cardiac blood flow increased by local arteriolar metabolites?? 5. Fast conduction pathway—bundle of his 6. Short acting blood control— barro receptor 7. Phosphate absorption —PCT 8. Cardiac output is – SV into HR 9. Cardiogenic shock— decreased cardiac output 10. Saw tooth waves on ECG is seen in A. Atrial flutter B. Arterial fibrillation C. Digoxin toxicity D. Hyperkalemia E. Ventricular fibrillation 11. Lazy man — 30 mins walk / day or thrice a week? Pathology 1. A 52 years old female having chronic viral hepatitis present with pain right hypochondrium. Ultrasound abdomen shows a space occupying lesion in the right lobe. The most likely diagnosis is A. Hepatic adenoma B. Hepatocelullar carcinoma C. Angiosarcoma D. Cyst of echinococcus granulossus E. Amoebiasis KEY: B, Hepatocellular carcinoma 2. Liver abscess — Echinococcus? 3. Bloody stools — iron deficiency anemia 4. IgG + HbeAg — chronic infectivity 5. Gall stones obstruction— bile duct 6. Germ cell tumor—dysgerminoma 7. Pregnant female present with Acne, which is Teratogenic Rx for Acne — vitamin A
  • 53. NEB/PMDC STEP-1 PAST PAPERS DR.ASIF ALI KHAN YANGTZE UNIVERSITY 8. A term infant is born and does well with breast feeding. Two days later his mother calls frantically because the baby is bleeding from umbilicus as nostrils. The following vitamin is most probably deficient in this infants: A. Vitamin A B. Vitamin D2 C. Vitamin D3 D. Vitamin E E. Vitamin K KEY: E 9. Black water fever— falciparum 10. H-pylori— adenocarcinoma 11. Abdominal aorta — atherosclerosis 12. Blood transfusion — O Negative blood 13. Exophthalmos — graves’ disease 14. Conns syndrome — hypokalemia 15. Hypoglycemia— insulinoma 16. Liver dx — ALT will increase 17. Pernicious anemia— intrinsic factor deficiency 18. Microcytic anemia —iron 19. Megaloblastic anemia —b12 20. Investigation after Hb in macrocytic anemia—Hb electrophoresis 21. Auer rods is seen in — AML 22. Commonest leukemia in kids — ALL 23. Hepatitis B — HBsAg 24. Hepatitis infectivity — Hbe 25. To check in Thalassemia — HbA2 26. Splenomegaly—sickle cell anemia 27. For monitoring Warfarin drug check — Prothrombin Time (PT) 28. Swollen abdomen, thin legs n arms — hepatic cirrhosis 29. First line defense — skin 30. Tb test HSV type — four
  • 54. NEB/PMDC STEP-1 PAST PAPERS DR.ASIF ALI KHAN YANGTZE UNIVERSITY 31. Following an IV dose of penicillin, a middle-aged man develops allergy such as skin rash and breathing difficulty followed by hypotension and shock. The hypersensitivity reaction is most likely mediated by which of the following? A. C3b B. IgE C. IgM D. Prostaglandins E. CD 4+ lymphocytes KEY: B 32. Anaphylactic shock — IgE 33. Placenta cross — IgG 34. Which of the following is the leading cause of severe diarrhoea among young children? A. RSV B. Adenovirus C. Rota virus D. EBV E. Echovirus KEY: C Forensic medicine 1. Patient death, Doctor was drunk — criminal 2. Didn’t do x-rays — civil negligence 3. Didn’t tell anything to patient — blanket consent 4. Arsenic is used for — homicidal 5. Multiple throat cuts—suicide 6. Stab wound patient — inform police 7. Blast injury— blast lung 8. Wedding firing—ricochet 9. Murree, cherry red— CO poisoning 10. Bugs under skin—coccaine poisoning 11. Chronic poisoning check — long bones 12. Farmers came with poison — OP poisoning 13. Wrestlers eye — contusion 14. Bluish black bruise —2-4 days 15. More bleeding in— incised wounds
  • 55. NEB/PMDC STEP-1 PAST PAPERS DR.ASIF ALI KHAN YANGTZE UNIVERSITY 16. Case: A 6 years old girl ate her mother’s anxiety drugs, she may be best treated by giving her an antagonist like: A. Flumazenil. B. Naloxone. C. Fomepizole. D. Nalorphine. E. Naltrexone. KEY: A 17. Vanilla flavor custard— food additive 18. Birds foot injury— driver 19. Consent from injured — written 20. Gunshot on right side—ipsilateral lower motor neuron injury Biochemistry 1. The diet of a child suffering from Maple syrup urine disease (an amino acid disorder), should be low, in which out of the following amino acids content? A. Branched chain amino acids B. Phenylalanine Alanine C. Methionine D. Tryptophan KEY: A 2. Lactate — cori cycle 3. Phenyl ketone urea, what to avoid — phenylalanine 4. Maximum substance made by—phenylalanine 5. Alcoholic patient— thiamine deficiency 6. Alcoholic with neurological problems—vit B 12 7. Cheilosis —riboflavin 8. Dementia — niacin 9. Hemopoitic vit— vitamin C 10. Vitamin in fats— vitamin E 11. In severe starvation energy comes from — amino acids 12. Patient on vegetable diet — b12 deficiency 13. Urea cycle enzyme deficiency — glutamine will increase 14. Bloating with milk— lactase deficiency 15. Exercise will reduce— LdL ? Vldl ?
  • 56. NEB/PMDC STEP-1 PAST PAPERS DR.ASIF ALI KHAN YANGTZE UNIVERSITY 16. Common sugar made her obese.. due to glucose or fructose? ]Number of BHUs (Basic health units) in Pakistan – According to Economic Survey of Pakistan 2016, currently there are 5464 basic health units in the country. The country's public health care system comprises of 1167 hospitals, 5695 dispensaries, 675 rural health centers(RHCs), 733 mother and child health centers and allied medical professionals) – 2017-2018 -- By the year 2017, the number of public sector hospitals has increased to 1211, 5508 basic health units (BHUs), 676 rural health centers (RHCs) and 5,697 dispensaries. PMDC Neb Step-1 Exam held on 25 November 2018 1. Basic health units (BHU) in Pakistan — 5000 - 6000 2. weakness of thenar muscles & sensation lost lateral aspect — Median nerve 3. Medial epicondyle fracture — ulnar nerve injury 4. In fracture of medial epicondyle of humerus, the patient suffers claw-hand deformity and sensory loss in medial 1/3' of palm and medial 11/2 fingers. The damaged nerve is: A. Musculocutaneous nerve. B. Ulnar nerve C. Median nerve D. Medial coetaneous nerve of forearm E. Radial nerve. KEY: B 5. A 23-year-old lady presents with abnormal spoon shaped nails (Koilonychia), pallor, weakness and palpitations and glossitis—Iron deficiency anemia 6. A 2 years old child presented with weakness and bowing of legs on x-ray — rickets 7. Laceration look a like incisions seen — on head/scalp 8. Patient with amenorrhea and nipples spread widely — Turner 9. Post UTI infection 10. 5th intercostal space which valve — Mitral 11. Thyroid – thyroglossal duct 12. BMI — 24.5 13. Alma-Ata declaration — physician oath 14. Isoniazid — pyridoxine deficiency
  • 57. NEB/PMDC STEP-1 PAST PAPERS DR.ASIF ALI KHAN YANGTZE UNIVERSITY 15. A 32-year-old body builder has decided to go on a diet consisting of egg whites to ensure only proteins for muscle growth. After a few weeks he experiences decreased energy and is found to be hypoglycemic. A nutritionist tells the patient that he most likely has the deficiency of vitamin biotin. Which of the following enzymes is unable to catalyze its step in synthesizing glucose from pyruvate. A. Pyruvate carboxylase B. Phospho enol pyruvate carboxykinase C. glucose-6-phosphatase D. Fructose 1, 6 bisphosphatase E. phosphoglycerate kinase KEY: A, Pyruvate carboxylase 16. 36-48 hr. — Falciparum malaria 17. Fatty acid synthesis — acetyl coA carboxylase 18. Biceps shoulder can't touch — musculocutanoeus 19. A child travelling in car uphill having nausea and vomiting — Scopolamine 20. Hydrochlorothiazide side effect — Gout 21. Koplik spot — measles 22. Antimalarial for falciparum malaria — Artemether 23. Enalapril — Dry cough 24. Acute pancreatitis diagnose — serum amylase 25. Sulbuterol — Tremor/seizure 26. Benzodiazepines— Flumazenil 27. Doxycycline— Renal failure 28. Green light tag— Ambulatory 29. Tetracycline— Teeth discoloration
  • 58. NEB/PMDC STEP-1 PAST PAPERS DR.ASIF ALI KHAN YANGTZE UNIVERSITY 30. A tissue graft between two people who are between genetically identical individuals is termed a: A. Isograft B. Heterograft C. Xenograft D. Autograft E. Allograft KEY: A, Explanation: » An “isograft“is performed between genetically identical individuals (i.e. monozygotic twins). » A “heterograft”is not a word that is used in transplantation immunology. » A “xenograft” is a transplant that is performed across species. » An “autograft” is a transplant from one location in the body to another. » An allograft is a transplant between two members of the same species who are not genetically identical. 31. Third pharyngeal pouch— thymus 32. Transitional epithelium—bladder 33. A male patient is taking cimetidine. Which adverse effect is more likely to occur with cimetidine than with other histamine2 receptor antagonists? A. Seizures B. Gynecomastia C. Hypoxia D. Hypertension E. Heart block KEY: B, 34. Which of the following drugs is most likely to cause systemic lupus-like syndrome? A. Isoniazid B. Methotrexate C. Procainamide D. Sulfasalazine KEY: C, Procainamide 35. Finger adduction and abduction is controlled by which of the following nerves? A. Radial B. Ulnar C. Musculocutaneous D. Median E. Axillary KEY: B
  • 59. NEB/PMDC STEP-1 PAST PAPERS DR.ASIF ALI KHAN YANGTZE UNIVERSITY 36. A 35-year old man is brought to a Tehsil Headquarter hospital in southern Punjab following a road traffic accident. He requires urgent blood transfusion and is transfused unscreened whole blood. His medical record retrieved later reveals that he is a chronic carrier of Hepatitis B virus. Two weeks later the patient dies of fulminant hepatic failure. What transfusion related incidence do you suspect most in this case? A. ABO incompatible blood transfused B. Rh incompatible blood transfused C. Transmission of HDV D. Transmission of HCV E. Transmission of HGV KEY: C 37. Pregnant lady — iron deficiency anemia 38. Pellagra— niacin 39. Sign of drowning— Ansa cutis 40. Leishmaniasis – Balochistan sandfly 41. Biguanide (Metformin) side effect— lactic acidosis 42. Fluorine – dental 43. Pregnant lady fell in washroom, severe bleeding — tertiary health unit 44. Bone marrow sample should be from iliac crest or sternum 45. In autopsy blood can be taken from— Femoral vein 46. Anthelminthic— Praziquantel 47. Mechanism of penicillin— transpeptidase cross link 48. SIADH causing hyponatremia in— Lung carcinoma 49. GFR increase by – decrease Afferent arterial resistance 50. Gynecomastia — Spironolactone 51. Hyperuricemia — Thiazide diuretic (2 questions) 52. Case of increase ICP — Mannitol 53. DM ketoacidosis— Regular insulin 54. Anti-thyroid — Agranulocytosis 55. Microcytic with splenomegaly—Thalassemia major 56. ALT — Viral hepatitis 57. Fatty liver changes— alcoholic hepatitis 58. Mallory bodies —Alcoholic hepatitis 59. After ATTB develops fever malaise and hemoptysis—Pyrazinamide 60. Alcoholic case scenario — Thiamine deficiency 61. Iron studies in blood — Ferritin
  • 60. NEB/PMDC STEP-1 PAST PAPERS DR.ASIF ALI KHAN YANGTZE UNIVERSITY 62. gluten hypersensitivity — Anti-IGA transglutamic AB 63. Pseudomembranous colitis— Vancomycin 64. Case scenario of heart failure patient, drug for pulmonary edema — Furosemide 65. Which of the following agents is useful in treatment of A. Baclofen B. Diazepam C. Cyclobenzaprine D. Dantrolene E. Halothane KEY: D 66. A 50 years old non-smoker lady with alpha-1 antitrypsin deficiency developed severe lung disease and requires a double lung transplant. What is the most likely lung lesion she is having? A. Adenocarcinome B. Bullous emphysema C. Centriacinar emphysema D. Interstitial emphysema E. panacinar emphysema KEY: 67. Tuft of hair neural tube defect— spina bifida 68. Neural tube defect having membrane covering outside — meningo 69. Facial nerve doesn’t supply Masseter muscle 70. Lateral ventricle with third ventricle with foramen monro 71. Patellar reflex — sciatic nerve » The muscles involved in the Achilles tendon reflex are supplied by the tibial nerve (a branch of the sciatic nerve), which is derived from the S1 and S2 nerve roots. The tibial nerve forms in the popliteal fossa, and leaves it running inferiorly on the tibialis posterior muscle. It supplies the posterior muscles of the leg and knee joint, and terminates by dividing into the medial and lateral plantar nerves. 72. The prophylactic treatment of malaria: A. Mefloquine B. Primaquine KEY: B 73. Drug which belongs to class-IA anti-arrhythmic A. Lidocaine B. Propranolol C. Procainamide D. Amiodarone KEY: C
  • 61. NEB/PMDC STEP-1 PAST PAPERS DR.ASIF ALI KHAN YANGTZE UNIVERSITY 74. Erythroblastosis fetalis can be prevented if the mother is injected, at parturition, with an antibody called: A. Blocking antibody B. Rh0 (D) immunoglobulin (RhoGAM) C. Antilymphocyte serum D. Antithymocyte serum E. Univalent antiserum KEY: B 75. The surgical neck of the humerus is related to the: A. Radial nerve. B. Axillary nerve. C. Ulnar nerve. D. Median nerve. KEY: B 76. Which one of the following nerve palsy may result in winging of the scapula? A. Long thoracic nerve palsy B. Suprascapular nerve palsy C. Spinal accessory nerve palsy D. Radial nerve palsy KEY: A 77. What is the serious adverse effect of metformin? A. Obesity B. Lactic acidosis C. Hypoglycemia D. Hypercalcemia KEY: B 78. A 54 year old female with an abdominal mass undergoes exploration laparoscopy. Both ovaries are enlarged hence resected. Pathology report is Krukenberg tumor, indicating which of the following: A. Ectopic pregnancy B. Endometriosis C. Hyperestrogenic state D. Immunosuppression E. Metastatic carcinoma KEY: E
  • 62. NEB/PMDC STEP-1 PAST PAPERS DR.ASIF ALI KHAN YANGTZE UNIVERSITY 79. Chills, fever, and muscle aches are common reactions to which one of the following antiviral drugs? A. Acyclovir B. Gancyclovir C. Oseltamivir D. Interferon E. Ribavirin KEY: D 80. Which of the following sulfonamide is used in the treatment of ulcerative colitis A. Sulfadiazine B. Sulfasalazine C. Salfadimidine D. Sulfadoxine E. Metronidazole KEY: B 81. Orthotolidine test is used to determine A. Nitrates in water B. Nitrites chlorine in water C. Free and combined chlorine in water D. Ammonia content in water E. None KEY: C 82. A mother brought her infant to the pediatrician for delayed milestones, hepatosplenomegaly and pallor. Another sibling had similar problems for which repeated blood transfusions was advised. Confirmation of the diagnosis can be made by: A. Full blood profile B. Iron studies C. Hb electrophoresis D. Bone marrow study E. Biopsy KEY: C 83. Antiemetic like effect— propo 84. Dissociative sedation— ketamine 85. A person died in police custody: magistrate 86. fasting plasma glucose _> 126 87. HbA1c _>6.5 to 7.5 88. Adenocarcinoma— Barrett’s esophagus
  • 63. NEB/PMDC STEP-1 PAST PAPERS DR.ASIF ALI KHAN YANGTZE UNIVERSITY 89. Which of the following steroids can be administered for maturity of surfactant in preterm labour. A. Beclomethasone B. Betamethasone C. Prednisolone D. Hydrocortisone KEY: B 90. A patient needs antibiotic treatment for culture-positive infective streptococcal endocarditis. His medical history includes a severe anaphylactic reaction to penicillin G during the last year. The best approach would be treatment with A. Amoxicillin-clavulanate B. Aztreonam C. Penicillin G D. Ticarcillin E. Vancomycin KEY: B 91. mRNA codon—AUG 92. Black guy with skin problem— ?fatty acid defect 93. Vitamin causing hypercreatinine— vit D? 94. Pulmonary HTN— Frusemide 95. Patient with HF present in emergency with dyspnea, rales on auscultation— Furosemide 96. Sublingual Anti anginal— Glyceryl trinitrate 97. Anti-peptic ulcer PPI—Omeprazole 98. Femur neck fracture, hx of long term drugs taking— Prednisolone 99. Asthmatic presents with tremors — Salbutamol 100. Long term use, seizures—Theophylline 101. Cromolyn Sodium A. Is given by oral route B. Produce bronchodilation in asthmatic patient C. Act by preventing potassium efflux from the mast cells D. Is useful for Prophylaxis of asthma KEY: D 102. Loperamide — Opioid Derivative
  • 64. NEB/PMDC STEP-1 PAST PAPERS DR.ASIF ALI KHAN YANGTZE UNIVERSITY 103. The most serious adverse reaction that may occur with the use of Thionamide antithyroid drugs is A. Agranulocytosis B. Joint pains C. Paresthesias D. Alopecia E. Skin rashes KEY: A 104. GIT motility — Domperidone 105. Drug which is used in the treatment of Bipolar Disorder A. Fluoxetine B. Benzodiazepine C. Lithium D. Clozapine KEY: C 106. Extrapyramidal symptoms (EPS) have been associated with which of the following drugs? A. Metoclopramide B. Alprazolam C. Aprepitant D. Loperamide KEY: A, Metoclopramide 107. Benzodiazepines Antidote— Flumazenil 108. fat emboli— long bone fracture 109. After criminal abortion— Hemorrhagic shock 110. Babinski positive— UMN and corticospinal tract damage 111. A teenager concerned about his weight, attempts to maintain a fat free diet for a period of several week. If his ability to make lipids were examined, he would be found to be the most deficient in his ability to make A. Cholesterol B. Glyco lipids C. Phospholipids D. Prostaglandins E. Triacyl Glycerol KEY: E
  • 65. NEB/PMDC STEP-1 PAST PAPERS DR.ASIF ALI KHAN YANGTZE UNIVERSITY 112. A young girl with history of severe abdominal pain was taken to local hospital at 5 am in severe distress. Blood was drawn the plasma appeared milky with TAG more than 2000 mg/dl (N = 4-150 mg/dl). The patient was placed on diet limited on fat but supplements with medium chain TAG. Which of the following protein is deficient in this patient. A. Apo A-1 B. Apo B-48 C. Apo C-II D. Cholestryl Ester transfer protein E. Microsomal triglycerides transfer protein KEY: C 113. A couple of American-African descent, after an otherwise uneven pregnancy, gives birth to an exceptionally fair skinned baby boy with also white hair. Further examination reveals red pupils. A postnatal screen likely to confirm the deficiency of the following enzyme A. Glutathione reductase B. Glutathione peroxidase C. Tyrosinase D. Methionine synthase E. Cystathionine beta synthase 114. A 24 year old women presents with diarrhea, dysphagia, jaundice and white transverse line on fingernails (Mee’s lines). The patient is diagnosed with arsenic poisoning which inhibit the following enzyme of TCA cycle A. Aconitase B. Alpha ketoglutarate dehydrogenase complex C. Citrate synthase D. Isocitrate dehydrogenase E. Succinate dehydrogenase KEY: B, Arsenic is toxic at many levels due to its broad specificity of enzyme inhibition. The mode of action of arsenic-containing compounds, primarily arsenic trioxide, is via it reacting with biological ligands that possess available sulfur groups. Arsenic interferes with the activity of the TCA cycle by inhibiting the α- ketoglutarate dehydrogenase complex, which requires the activity of the sulfhydryl group associated with the dihydrolipoamide moiety of the enzyme complex. 115. A 40 years old male presented with chest pain that radiates to his left jaws and shoulder, he has diagnosed with myocardial infarction and is prescribed a statin medication. Statin are competitive inhibitors of HMG-CoA-Reducatse. Which convert HMG-CoA to the following A. Cholesterol B. Famesyl pyrophosphate C. Geranyl pyrophosphate D. Isopententyl pyrophosphate E. Mevalomate
  • 66. NEB/PMDC STEP-1 PAST PAPERS DR.ASIF ALI KHAN YANGTZE UNIVERSITY 116. Three days after a full-term normal delivery, a neonate vagina to hyperventilate, develops hypothermia and cerebral edema and becomes comatose, urinalysis reveals high levels of glutamine and orotic acid. The BUN is below normal. Which enzyme is most likely to be deficient in this child? A. Cytoplasmic glutaminase B. Cytoplasmic carbamoyl phosphate synthetase C. Cytoplasmic orotidylate decarboxylase D. Mitochondrial carbamoyl phosphate synthetase E. Mitochondrial orinthine transcarbamoyolase 117. Screening of disease is a part of A. Primary prevention B. Secondary prevention C. Tertiary prevention D. All of the above E. None of the above KEY: B 118. Commonest cause of blindness in preschool children’s is A. Maternal rubella B. Vitamin A deficiency C. Accidental injury D. Congenital cataract E. Trachoma neonatorum KEY: B 119. Clavulanic acid is a beta-lactam antibiotic that A. That easily penetrates gram negative microorganisms B. Is specific for gram positive microorganisms C. Is a potent inhibitor of cell transpeptidase? D. Inactivates bacterial B-lactamases E. Has a spectrum of activity similar to penicillin G KEY: D 120. Secondary intension of wound healing differs from primary intention in which of the following aspects A. Neutropenia accumulation B. Mitoses C. Fibrous reaction D. Wound contraction E. Clot formation KEY: D 121. The following is inhibitor of prokaryotic transcription A. Ciprofloxacin B. Etoposide C. Erythromycin D. Rifampicin E. Methotrexate KEY: D
  • 67. NEB/PMDC STEP-1 PAST PAPERS DR.ASIF ALI KHAN YANGTZE UNIVERSITY 122. The following subunit of bacterial RNA polymerase is responsible promoter recognition A. Alpha B. Beta C. Beta prime D. Sigma E. Alpha prime KEY 123. A specific form of necrosis, also visible under light microscope, which occur after deposition of immune complexes in the walls of the arteries is A. Coagulative necrosis B. Caseous necrosis C. Fibrinoid necrosis D. Fat necrosis E. Liquefactive necrosis 124. A 70 years old man presented in medical emergency with c/o severe chest pain with profuse sweating and shortness of breath. He was diagnosed as a case of acute myocardial infarction. Which isoenzyme of creatine kinase (CK) raised in this case A. CK 1 B. CK 2 C. CK 3 D. CK MB E. Both b and d KEY: D 125. A known patient of haemophilia a being treated by factor VIII concentrates has stopped responding to this treatment and his condition is worsening. the most likely explanation to this phenomenon is A. Neutralizing antibodies against factor VIII B. Low dose of factor VIII C. Expired factor VIII D. Wrong diagnosis E. Infection 126. Over the counter medications containing bismuth are quite popular for managing diarrhea and several other common GI maladies. However, the bismuth salt they contain is the subsalicylate. Given the presence of salicylate, these should not be administered to children A. Under 14 years old B. With diarrhea lasting more than 18h C. Having diarrhea with flatus D. Child with flu, common cold, chickenpox E. With otitis media KEY
  • 68. NEB/PMDC STEP-1 PAST PAPERS DR.ASIF ALI KHAN YANGTZE UNIVERSITY 127. Following system is in vogue in Pakistan A. Coroner system B. Medical examiner system C. Continental system D. Modified continental system E. Proctor fiscal system KEY: D 128. Cytological examination of FNA breast of 45 years old female showed groups of cells which were dyscohesive and were showing dyspolarity, individually scattered cells with open chromatin and nuclear membrane irregularity were also found. Most likely cytological category of lesion is: A. Atypical probably benign lesion (C-3) B. Benign breast lesion (C-2) C. Malignant lesion (C-5) D. Suspicious malignant lesion (C-4) E. Smear inadequate (C-1) KEY: 129. The drug most effective against malarial parasites in the liver but not effective against parasites within erythrocytes is A. Primaquine B. Pyremithamine C. Quinidine D. Chloroquine E. Chloroguanide KEY: A 130. Four hours after abdominal surgery a 65 years old male was found to have some equivocal ECG changes and markedly raised serum CK (>1500 U/L). which of the following cardiac markers would be the most specific investigations to rule out myocardial infarction in this patient: A. CK-MB activity measurement B. CK-MB mass measurement C. CK-MM activity measurement D. LDH levels E. Estimation of cardiac troponin T KEY: D 131. Flow like syndrome is seen with: A. Rifampin B. Isoniazid C. Streptomycin D. Pyrazinamide E. Ethambutol KEY: B
  • 69. NEB/PMDC STEP-1 PAST PAPERS DR.ASIF ALI KHAN YANGTZE UNIVERSITY 132. A 55 year old woman presents in OPD with inability to adduct and medially rotate her arm after right mastectomy. Examination showed loss of function of latissmiss dorsi muscle. The nerve supplying in the muscle derived from: A. Lateral cord of brachial plexus B. Medial cord of brachial plexus C. Posterior cord of brachial plexus D. Upper trunk of brachial plexus E. Lumbar plexus 133. Septic shocks results from severe inflammatory response to bacterial infection. It has a high mortality rate and is associated with changes in the level of nitric oxide. Which statement concerning septic shock is mostly likely accurate A. Activation of endothelial nitric oxide synthase cases an increase in NO B. High mortality is the result of the long half-life of nitric oxide C. Lysine, the nitrogen source of nitric oxide synthesis, is deaminated bacteria D. Over production of nitric oxide by Ca – independent enzyme is the cause of the hypotension E. High mortality is the result of short half-life of NO KEY: A 134. Separation of skull suture in young persons by blunt trauma is classified as A. Comminuted fracture B. Diastatic fracture C. Hinge fracture D. Ring fracture E. Signature fracture KEY: A 135. A 26 year old female is using injectable Medroxyprogesterone as a method of contraception. Which of the following adverse effects is a concern if she wishes to use this therapy long-term? A. Hyperkalemia B. Male pattern baldness C. Osteoporosis D. Weight gain E. Pulmonary fibrosis KEY: C 136. Myasthenia gravis is an autoimmune disease in which antibodies damage or destroy: A. Acetylcholine molecules in the synaptic cleft B. Acetylcholine receptors on the muscle membrane C. Acetylcholine vesicles in the neuromuscular junction D. Acetylcholinesterase molecules in the presynaptic terminal E. Acetylcholinesterase molecules in the synaptic cleft KEY: B
  • 70. NEB/PMDC STEP-1 PAST PAPERS DR.ASIF ALI KHAN YANGTZE UNIVERSITY 137. The pap smear of patient showed enlarged cells having orarangophilic cytoplasm and central vascular nucleus which was surrounded by perinuclear hallow. These cells are most likely indicative of A. CIN-1 B. Invasive squamous cell carcinoma C. Invasive adenocarcinoma of cervix D. Invasive adenocarcinoma of endometrium E. Koilocystic cells 138. A six-year-old child, whose medical history includes a rather difficult birth, has a permanently tilted head posture, with the right ear near the right shoulder and the face turned upward and to the left. Which of the following muscles was very likely damaged during birth? A. Anterior scalene B. Omohyoid C. Sternocleidomastoid D. Trapezius E. Platysma KEY: C 139. Which of the following antiemetics functioning through inhibition of neurokinin-1 (NK-1) receptor A. Aprepitant B. Domperidone C. Granisteron D. Ondansetron 140. Which of the following drugs is leukotriene-modifying drug indicated in the management of bronchial asthma A. Triamcinolone acetonide B. Budesonide C. Zafirleukast D. Flunisolide E. Fluticasone KEY: C 141. There is inverse relationship between hardness of drinking water and A. Diabetes mellitus B. Cardiovascular disease C. malignancy D. Obesity E. All of the above KEY: B
  • 71. NEB/PMDC STEP-1 PAST PAPERS DR.ASIF ALI KHAN YANGTZE UNIVERSITY 142. Vasodilation caused by A. Parasympathetic stimulation B. Sympathetic blockade C. Localized hypoxia D. Anaphylactic shock E. Stimulation of chemoreceptors 143. A 56 year old female develop massive pulmonary embolism. The most likely chances that embolus must be from A. Deep calf veins B. Femoral veins C. Iliac veins D. Pelvic veins E. Portal veins KEY: B 144. Surgical removal of adrenal tumour of a 47-year old man, is planned. It is anticipated that the procedure will take approximately 16h. The patient has a history of hypertension controlled with a beta-blocker. Which of the following agents, used intraoperatively, will provide efficacious blood pressure control for the duration of the procedure A. Acebutolol B. Esmolol C. Metoprolol D. Nadolol E. Pindolol KEY: B 145. Two enzyme have similar V max, but enzyme 1 has lower Km as compared to enzyme 2, this means A. Less substrate is required by enzyme 1 to attain V max/2 B. Enzyme 1 has more active sites C. Enzyme 2 has lesser number of binding sites D. Enzyme 2 has lesser number of regulatory sites E. Enzyme 2 his slowed down by substrate 146. A 25 years old male suffering from high grade fever, rigors and severe headache. Examination of the peripheral blood film reveals presence of rings in the red cells and crescent shaped gametes. What is the most likely pathogen A. Plasmodium vivax B. Plasmodium malariae C. Plasmodium ovale D. Plasmodium falciparum E. Plasmodium knowlesii
  • 72. NEB/PMDC STEP-1 PAST PAPERS DR.ASIF ALI KHAN YANGTZE UNIVERSITY 147. A 50 years old patient suffering from chronic cough, weight loss and low grade fever is admitted with probable diagnosis of tuberculosis. Immunity to tuberculous infection is primarily mediated by A. DC4+ve T cells B. CD8 + T cells C. Macrophages D. Neutrophils E. B Cells 148. The most likely cause of intermittent diarrhea of 3weeks duration in a 26 year old male with space occupying lesion in the right hypochondrium most likely is due to A. Giardia Lamblia B. Entamoeba histolytica C. Balantidium coli D. Entamoeba coli E. Cryptosporidium parvum 149. Exposure to 4 atmosphere of oxygen (PO2 = 3040 mmhg) can cause d duration within an hour or so due to acute oxygen poisoning. The lethal effect of acute oxygen poisoning can be attributed to the dysfunction of which of the following agents A. Brain B. Heart C. Kidney D. Liver E. Lungs 150. An aviator is flaying at 30,000 feet, where the barometric pressure is 226 mm hg. He is breathing 100% oxygen. His alveolar PCO2 is 40mm hg, and alveolar water vapor pressure is 47 47 mm hg. What is the alveolar PO2 of the aviator A. 43 mm Hg B. 75 mm Hg C. 99 mm Hg D. 139 mm Hg E. 100 mm Hg
  • 73. NEB/PMDC STEP-1 PAST PAPERS DR.ASIF ALI KHAN YANGTZE UNIVERSITY 23 JUNE 2019 NUMS MCQS OF NEB PMDC STEP-1 1. Histopathological examination of lymph nodes of 18 years old diagnosed patient of Hodgkin’s lymphoma, involving supraclavicular and lower cervical lymph nodes, showed a lacunar variant of Reed-sternburg cells. In which sub type of Hodgkin’s lymphoma this variant is found. A. Nodular sclerosing H.L B. Mixed cellularity H.L C. Lymphocytic predominance H.L D. Lymphocytic rich H.L KEY: A 2. An 83 years old female has a biopsy of an ulcerated nipple lesion that is interpreted as Paget’s disease. A biopsy of the breast tissue with an ulcerated lesion on the nipple diagnoses to be Paget’s disease will most likely show the following disease: A. Acute mastitis B. Ductal carcinoma in situ C. Intraductal papilloma D. Invasive lobular carcinoma KEY: B 3. X-ray of a patient show skull thickening with narrowing of foramina and the proximal humerus- typical sun burst or sun ray appearance. There is bowing of femur and tibia as well and raised alkaline phosphates. Bone biopsy reveals mosaic pattern of bone spicules with prominent osteoid seams. Which neoplasm will it be? A. Astrocytoma. B. Osteosarcoma. C. Meningioma. D. Hodgkin's lymphoma KEY: B 4. A person with endemic goiter has A. High level of Plasma TSH B. High level of Plasma thyroxine C. High level of Plasma triiodothyronine D. Low production of thyroglobulin E. Low level of growth hormone KEY: A, High level of Plasma TSH (Decreased T3, T4 and Increased TSH)
  • 74. NEB/PMDC STEP-1 PAST PAPERS DR.ASIF ALI KHAN YANGTZE UNIVERSITY 5. A 60-year old businessman returns to Pakistan after a 2-month long trip to the Middle East. A week later he presents with gross hematuria. Urine examination shows pear shaped ova with large terminal spine. Which of the following is most likely responsible for this patient’s symptoms? A. Schistosoma haematobium B. Schistosoma Japonicum C. Schistosoma mansoni D. Wuchereria Bancrofti KEY: A 6. A 42 years old obese women presented to the emergency room with complain of worsening nausea, vomiting and epigastric pain. The blood chemistry revealed a high Amylase level. Most likely diagnosis is: A. Acute appendicitis B. Acute Pancreatitis C. Gastritis D. Renal colic E. Viral hepatitis KEY: B, 7. After prolonged active hepatitis C, a 50 years old patient developed ascites and esophageal varices. These complication are secondary to: A. Cholangitis B. Hepatocellular carcinoma C. Pancreatitis D. Cirrhosis KEY: D 8. A women smokes one pack/day, she drinks at least 5 cups of coffee/day. Her mother died of breast cancer. Which is the greatest risk factor for breast cancer in this patient A. Birth control pills B. Cigarette smoking C. Family history D. Caffeine intake KEY: C 9. A 33 year old female presented with pain abdomen for 3-6 months, low grade fever and lethargy. On ultrasound examination right ovary was four to be enlarged and was removed for examination. On microscopy inflammation with epithelioid cells, Langerhans’s giant cells and caseous necrosis was seen. The most likely diagnosis is: A. Mumps B. Syphilis C. Tuberculosis D. Sarcoidosis KEY: C
  • 75. NEB/PMDC STEP-1 PAST PAPERS DR.ASIF ALI KHAN YANGTZE UNIVERSITY 10. Deposition of calcium in nonviable or dying tissue in the presence of normal serum calcium levels is called A. Metastatic calcification B. Dystrophic calcification C. Intracellular calcification D. Calcium excess E. None of the above KEY: B 11. A 15-year-old girl is brought to the physician by her parents, who are concerned because she has developed multiple nodules on her skin. On physical examination, there are 20 scattered, 0.3- to 1-cm, firm nodules on the patient's trunk and extremities. There are 12 light brown macules averaging 2 to 5 cm in diameter on the skin of the trunk. Slit-lamp examination shows pigmented nodules in the iris. These findings are most likely to be associated with which of the following types of neoplasm? A. Dermatofibroma B. Leiomyoma C. Neurofibroma D. Lipoma KEY: C, The patient has neurofibromatosis type 1 (NF-1), characterized by the development of multiple neurofibromas and pigmented skin lesions. Neurofibromas are most numerous in the dermis but also may occur in visceral organs. Patients with NF-1 also may develop a type of sarcomatous neoplasm known as a malignant peripheral nerve sheath tumor. Dermatofibromas are subcutaneous masses that typically are small and solitary. Leiomyomas occur most frequently in the uterus. Lipomas can occur almost anywhere in the body, but do so sporadically. Hemangiomas may occur sporadically on the skin; they typically are red- blue masses. 12. A 70-year-old Negroid man from Makkran coast was stabbed on the upper arm, he received emergency treatment and his wound healed. He developed a raised hypertrophic scar with boundaries beyond the original wound, and it did not regress. Which of the following terms best describes this condition: A. Cicatrix B. Keloid C. Callus D. Granulation tissue KEY: B 13. Which of the following arrhythmia is most commonly associated with syncope A. Sinus arrhythmia B. First degree heart block C. Second degree heart block D. Third degree heart block KEY: B
  • 76. NEB/PMDC STEP-1 PAST PAPERS DR.ASIF ALI KHAN YANGTZE UNIVERSITY 14. Which of the following corticosteroids is most appropriate to administer to a women in preterm labor to accelerate fetal lung maturation. A. Beclomethasone B. Betamethasone C. Prednisolone D. Hydrocortisone KEY: B 15. A 55 year old women stop menstruating approximately 3 months ago. Worried that she is maybe pregnant, she decided to have a pregnancy test. The test came back negative. Which of the following series of tests results would confirm that the women is post-menopausal A. Decreased LH, decreased FSH, increased estrogen B. Decreased LH, increased FSH, decreased estrogen C. Increased LH, decreased FSH, decreased estrogen D. Increased LH, increased FSH, decreased estrogen KEY: D, FSH and LH can no longer perform their usual functions to regulate estrogen, progesterone and testosterone. These inevitable changes in hormones and natural decline of estrogen levels causes menopause 16. PTH A. Acts directly on bone cells to increase calcium deposition B. Acts indirectly on intestinal cells to increase calcium absorption C. Causes decrease in cAMP concentration within renal proximal tubular cells D. Is secreted in response of an increase in plasma calcium concentration KEY: 17. The part of male reproductive tract which carries only semen within prostate gland is the A. Prostatic urethra B. Seminal vesicles C. Ductus deferens D. Ejaculatory duct KEY: D, Ejaculatory duct-the ejaculatory duct is a duct which courses through the prostate gland and contains only semen. Remember, semen is the combination of sperm from the ductus deferens, seminal fluid from the seminal vesicle, and secretions of the prostate gland. The ejaculatory duct is formed by the union of the duct of the seminal vesicle and the ampulla of the ductus deferens, and it is the site where sperm and seminal fluid mix. The prostatic urethra is also contained in the prostate gland, and it carries semen, but it also carries urine out of the bladder. The membranous urethra is the continuation of the prostatic urethra outside of the prostate gland, and it carries both semen and urine as well. The seminal vesicle is a structure on the posterior surface of the bladder that produces seminal fluid. The ductus deferens is a passageway that carries sperm from the epididymis to the ejaculatory duct.
  • 77. NEB/PMDC STEP-1 PAST PAPERS DR.ASIF ALI KHAN YANGTZE UNIVERSITY 18. The structure passing through the aortic opening of diaphragm A. Accessory nerve B. Esophagus C. Superior vena cava D. Azygous Vein KEY: D, Azygous Vein 19. Cytotoxic T-cells help in A. Destroying tissue invaded by viruses B. Activation of mast cells C. Regulating T-cell activity D. Suppressing the T-cell activity KEY: A 20. A 43 year old women came with large abscess in the middle of right post triangle of the neck. The physician incised and drained the abscess. Five days later patient noticed that she could not extend her right hand above her head to brush her hair. Which of the following structure is damaged A. Long thoracic nerve B. Suprascapular nerve C. Accessory nerve D. Dorsal scapular nerve KEY: C, Accessory nerve (spinal accessory nerve) 21. Knee jerk nerve involvement — sciatic nerve ✅ » The muscles involved in the Achilles tendon reflex are supplied by the tibial nerve (a branch of the sciatic nerve), which is derived from the S1 and S2 nerve roots. The tibial nerve forms in the popliteal fossa, and leaves it running inferiorly on the tibialis posterior muscle. It supplies the posterior muscles of the leg and knee joint, and terminates by dividing into the medial and lateral plantar nerves. 22. A Case scenario of and asked about association of CNS axon - Multiple Sclerosis is a disease characterized by preferential damage to myelin that becomes detached from axon in the central nervous system. Which associated cells are involved? A. Oligodendrocytes. B. Microglia. C. Protoplasmic astrocytes. D. Satellite cells. KEY: A 23. Premature closure of the sagittal suture A. Anencephaly B. Scaphocephaly C. Brachycephaly D. ….. KEY: B
  • 78. NEB/PMDC STEP-1 PAST PAPERS DR.ASIF ALI KHAN YANGTZE UNIVERSITY 24. A patient who has suffered severe chest trauma in an automobile accident is found to have fluid in the right pleural space. A thoracentesis reveals the presence of chylous fluid in the pleural space, suggesting a rupture of the thoracic duct. In which regions of the thorax is the thoracic duct found. A. Anterior and middle mediastinum B. Anterior and superior mediastinum C. Middle and superior mediastinum D. Posterior and superior mediastinum KEY: D, Posterior and superior mediastinum 25. During a full workup on a 2-month old with a history of intermittent gastrointestinal pain and vomiting, physician discovered that the cause was lack of emptying of the stomach. They immediately suspected that the use was spasmodic contraction of which of the following parts of the stomach A. Cardiac notch B. Fundus C. Lessar curvature D. Pylorus KEY: D 26. Neonatal tetanus is an important preventable cause of neonatal mortality that account for 7% of neonatal deaths worldwide. This could be prevented by: A. Vaccination of women in reproductive age group B. Vaccination of newborn’s C. Vaccination of pregnant ladies D. Vaccination of females of child bearing age E. Vaccination of non-pregnant females KEY: A, NOT CONFIRMED 27. A mother bought her six months old child to a BHU. She is worried about the growth of her child. The best single measure for assessing the physical growth in this age A. Weight for age B. Height for age C. Chest circumference D. Mild upper arm circumference KEY: A, Weight for age 28. In study carried out in the hospital ward, every 10th admitted patient was included in the sample, which sample procedure is this A. Random sampling B. Stratified sampling C. Quota sampling D. Systemic sampling KEY: D
  • 79. NEB/PMDC STEP-1 PAST PAPERS DR.ASIF ALI KHAN YANGTZE UNIVERSITY 29. Secular trend ✅? Cyclic trend? » Secular trends are those trends which describe the change in occurrence of a disease over prolonged periods, usually years. It depends on the multiple factors such as degree of immunity, socioeconomic status and nutritional levels in the population. 30. Hospital waste is best disposed off by A. Composting B. Dumping C. Incineration D. Burial KEY: C, Incineration 31. Equity in health service means A. Provision of health services on equal basis B. Provision of health services according to the demands C. Provision of health services according the needs D. Provision of health services according to the resources KEY: C 32. A term infant is born and does well with breast feeding. Two days later his mother calls frantically because the baby is bleeding from umbilicus as nostrils. The following vitamin is most probably deficient in this infants: A. Vitamin A B. Vitamin D3 C. Vitamin E D. Vitamin K KEY: E 33. Pap-smear is graded as cervical intra-epithelial neoplasia, Grade II (CIN II). Which of the following viruses is related to this neoplastic growth? A. Epstein Barr virus. B. Hepatitis B virus. C. Human papilloma virus. D. Human herpes virus 8 KEY: C 34. Which of the following is a common compound shared by the TCA cycle and the Urea cycle? A. α- Keto glutarate B. Succinyl co A C. Oxalo acetate D. Fumarate KEY: D
  • 80. NEB/PMDC STEP-1 PAST PAPERS DR.ASIF ALI KHAN YANGTZE UNIVERSITY 35. Aortic dissection occurs in a tall 31 years old male with long fingers. His ophthalmic examination reveals ocular lens dislocation. These features are found in A. Down’s syndrome B. Glycogen storage disease C. Neurofibromatosis D. Marfan syndrome KEY: D 36. A 40-year old woman complains of deceased energy, significant weight gain and cold intolerance. She is seen by her family physician, who has diagnosed her to be having hypothyroidism (Low level of thyroid hormone). Which of the following is a precursor for thyroid hormone? A. DOPA B. Glutamine C. Tyrosine D. Tryptophan KEY: C 37. A 55-year-old man suffers from cirrhosis of liver. Toxins such as ammonia are not properly metabolized by the liver and can damage brain. Which of the following compounds is expected to be in highest concentration in brain as a result of detoxification of ammonia? A. Alpha keto glutarate B. Glutamate C. Glutamine D. GABA E. Asparagine KEY: C 38. The predominant effect of Glucocorticoids on the circulating white blood cells is A. Increase in T lymphocytes B. Increase in monocytes C. Increase in basophils D. Increase neutrophils KEY: D 39. A 68 year old male has COPD with moderate airway obstruction, Depsite using salmeterol twice daily as prescribed, his reports continued symptoms of shortness of breath with mild exertion. Which one of the following agents would be an appropriate addition to his current therapy? A. Systemic corticosteroids B. Montelukast C. Tiotropium D. Oral theophylline Key: C – Tiotropium
  • 81. NEB/PMDC STEP-1 PAST PAPERS DR.ASIF ALI KHAN YANGTZE UNIVERSITY 40. A 22-year old man, who frequently backpacks, complains of diarrhea and fatigue. Examination of stool specimens shows bi-nucleate organisms with four flagellae. Which one of the following drugs would be effective in treating this patient’s infestation A. Metronidazole. B. Quinidine. C. Pentamidine. D. Sulfadoxine. KEY: A 41. A 32 year old male presents to an outpatient clinic with a 5-day history of productive cough, purulent sputum, and shortness of breath. He is diagnosed with community Acquired pneumonia (CAP). It is noted that this patient has a severe Ampicillin allergy (anaphylaxis). Which of the following would be an acceptable treatment for this patient A. Levofloxacin B. Ciprofloxacin C. Penicillin-V D. Nitrofurantoin KEY: A – Streptococcus pneumoniae is a common cause of CAP, and the respiratory fluoroquinolones levofloxacin and Moxifloxacin provide good coverage. Ciprofloxacin does not cover S. pneumoniae well and is a poor choice for treatment of CAP. Penicillin would be a poor choice due to allergy. Nitrofurantoin has no clinical utility for respiratory tract infections. 42. A 56 years old female present with vomiting, diarrhea, headache, vertigo, tinnitus, her temperature is 98.8o F, BP 106/60mmHg, PR is 56, P/E skin rashes and photosensitivity etc — Quinine ✅ » This patient presents with symptoms of cinchonism, a syndrome caused by the ingestion of derivatives of cinchona bark such as quinine and quinidine. The key features of cinchonism include tinnitus (and often hearing loss), blurred vision, headache, vomiting, and diarrhea. Confusion, delirium and psychosis may occur. Quinidine toxicity will also cause bradycardia and hypotension, and a lichenoid photosensitivity reaction is possible. 43. A 60-year-old man is admitted to the hospital with acute heart failure and pulmonary edema. Which of the following drugs would be most useful in treating the pulmonary edema? A. Digoxin B. Dobutamine C. Furosemide D. Spironolactone KEY: C
  • 82. NEB/PMDC STEP-1 PAST PAPERS DR.ASIF ALI KHAN YANGTZE UNIVERSITY 44. A 42-year-old woman presents to the emergency department with fevers and headaches for the past 3 days. The fevers are off and on but are usually at least 102.5°F. Her headaches usually occur at the same time as the fevers. Splenomegaly is noted on examination. Peripheral blood smear confirms the presence of Plasmodium vivax. What is the most appropriate treatment —Primaquine (Tertian malaria) » Primaquine is added to the regimen for treatment of Plasmodium vivax and Plasmodium ovale to eradicate the dormant stages in the liver. Without Primaquine, once the blood stream infection is cleared, it is likely that recurrence will still occur. However, Primaquine only will not cure malaria, so both Primaquine and quinine is usually prescribed. 45. A 66-year-old woman with a history of Cushing’s disease treated with oral glucocorticoids presents to her primary care physician for follow-up. She was recently hospitalized for a hip fracture following a fall, the most common side effect of prolong use of glucocorticoid. — Osteoporosis ✅ » Osteoporosis is the most common adverse effect because of the ability of glucocorticoids to suppress intestinal Ca2+ absorption, inhibit bone formation, and decrease sex hormone synthesis. Alternate-day dosing does not prevent osteoporosis. Patients are advised to take calcium and vitamin D supplements. Drugs that are effective in treating osteoporosis may also be beneficial. 46. Drug combination of Sulbactam + with A. Ciprofloxacin B. Amoxicillin C. Erythromycin D. Piperacillin KEY: B, Sulbactam + Amoxicillin Available antibiotics drug combination in Pakistan 1. Ampicillin + Cloxacillin Ampiclox (GSK) 2. Amoxicillin + Sulbactam Inj. Bactamox plus (Bosch) 3. Amoxicillin + Clavulanic acid Augmentin (GSK) 4. Ampicillin + Sulbactam Inj. Unasyn (Pfizer/GSK) 5. Sulbactam + Cephalosporin group (Ceftriaxone, Ceftazidime, cefoperazone, Cefepime) 6. Sulbactam + cefoperazone Inj. Cebac (Bosch) / Inj. xorbact (highnoon) 7. Piperacillin + Tazobactam Inj. Tazocin EF 4.5mg (Pfizer) 8. Ticarcillin + Clavulanic acid Inj. Timentin 3.1g (GSK) 47. Chills, fever, and muscle aches are common reactions to which one of the following antiviral drugs A. Acyclovir B. Gancyclovir C. Oseltamivir D. Interferon KEY: D
  • 83. NEB/PMDC STEP-1 PAST PAPERS DR.ASIF ALI KHAN YANGTZE UNIVERSITY 48. Calcium magnesium inhibit the absorption of which drug in intestine A. Tetracycline B. Ciprofloxacin C. Benzyl penicillin 49. Which of the following sulfonamide is used in the treatment of ulcerative colitis A. Sulfadiazine B. Sulfasalazine C. Salfadimidine D. Sulfadoxine KEY: B 50. We have a patient with essential hypertension and unusually high circulating catecholamine level. Our goal is to block α- and β-adrenergic receptors using just one drug. Which of the following is capable to do that A. Labetalol B. Metoprolol C. Nadolol D. Pindolol KEY: A - Labetalol (Alpha + Beta blocker) 51. A 12 year old girl with a childhood history of asthma complained of cough, dyspnea, and wheezing after visiting a riding stable. Her symptoms became so severe that her parents brought her to the emergency room. Which of the following is the most appropriate drug to rapidly reverse her bronchoconstriction A. Inhaled fluticasone B. Inhaled Beclomethasone C. Inhaled albuterol D. Oral theophylline Key: C, Inhaled albuterol - inhalation of a rapid-acting B2 agonist, such as albuterol, usually provides immediate bronchodilation. 52. Whiplash is which form of injury A. Hyper flexion B. Hyperextension C. Lateral flexion D. Atlanto-axial dislocation KEY: B
  • 84. NEB/PMDC STEP-1 PAST PAPERS DR.ASIF ALI KHAN YANGTZE UNIVERSITY 53. During medico-legal autopsy, bullet is picked with A. Toothed forceps B. Hand C. Rubbed ended forceps D. Scissors KEY: C 54. Bleeding from nose and ears is commonly seen at autopsy in A. Hanging B. Lynching C. Strangulations by ligature D. Smothering KEY: C 55. Under the Pakistan Panel code, the minimum age of criminal responsibility is A. 18 Years B. 13 Years C. 7 Years D. 5 Years KEY: C, 7 Years, The age of full criminal responsibility is 18 years 56. A 10 year old develops a bruise on his arm after a fall, which changes its color to yellow to brown a few days later due to accumulation of: A. Lipofuscin B. Bilirubin C. melanin D. Hemosiderin KEY: D 57. Dr. sana is operating a patient with myocardial infarction, she will prefer following structure for the bypass graft A. Great saphenous vein B. Small saphenous vein C. Profundafemoris vein D. Popliteal vein KEY: D
  • 85. NEB/PMDC STEP-1 PAST PAPERS DR.ASIF ALI KHAN YANGTZE UNIVERSITY 58. A 26 years old female patient, at 3rd day of delivering a baby developed cyanosis, shortness of breath and orthopnea. What is the most likely underlying pathology causing these signs and symptoms A. Fat embolism B. Amniotic Fluid Embolism C. Air embolism D. Myocardial infarct KEY: B 59. A girl receiving treatment for asthma developed muscle tremors, which drug may be responsible A. Salbutamol B. Beclomethasone C. Cromolyn sodium D. Montelukast KEY: A 60. Pulmonary surfactant is formed by: A. Goblet cells B. Type I alveolar cells C. Type II alveolar cells D. Alveolar macrophages KEY: C 61. Which of the following is the risk of digoxin toxicity A. Hypernatremia B. Hypokalemia C. Hyperkalemia D. Hyponatremia KEY: B 62. When the respiratory drive for increased pulmonary ventilation becomes greater than normal, a special set of respiratory neurons, that are inactive during normal quiet breathing, then become active contributing to the respiratory drive. These neurons are located in which of the following structures? A. Apneustic center B. Nucleus of the tractus solitaries C. Pneumotaxic center D. Ventral respiratory group KEY: D
  • 86. NEB/PMDC STEP-1 PAST PAPERS DR.ASIF ALI KHAN YANGTZE UNIVERSITY 63. The oesophageal opening in the diaphragm is at A. T6 B. T8 C. T10 D. T12 KEY: C, Esophageal opening at T10 - It is located in the right crus, one of the two tendinous structures that connect the diaphragm to the spine. Fibers of the right crus cross one another below the hiatus. It is located approximately at level of the tenth thoracic vertebra (T10). 64. The isthmus of the thyroid gland A. Lies directly anterior to the thyroid cartilage B. Lies directly anterior to the cricoid cartilage C. Lies directly anterior to the 2nd, 3rd, and 4th Tracheal rings D. Contains the superior pair of parathyroid glands KEY: C 65. What exits from the Stylomastoid foramen A. Chorda tympanic nerve B. Accessory nerve C. Facial nerve D. Glossopharyngeal nerve KEY: C 66. A 22-year-old woman who is a professional tennis player presents to the physician with pain on lateral aspect of her elbow radiating down her forearm. Repetitive use of which of the following muscle most likely lead to this patient’s condition? A. Biceps brachii B. Extensor carpi radialis C. Extensor carpi ulnaris D. Flexor carpi ulnaris KEY: B, Due to repetitive use of Extensor like flexor carpi Radialis (due to Backhand shots), he developed lateral Epicondylitis. *However note that both (medial, lateral) overuse injuries can be idiopathic. 67. The fetal period extends from: A. 12th week to term B. 3rd week to 8th week C. 4th week to term D. 9th week to term KEY: D
  • 87. NEB/PMDC STEP-1 PAST PAPERS DR.ASIF ALI KHAN YANGTZE UNIVERSITY 68. The secretion of growth hormone is increased by A. Hyperglycemia B. Exercise C. Somatostatin D. Hypothermia KEY: B 69. If a patient with metabolic alkalosis A. HCO3 of arterial blood is decreased B. Kidney retained bicarbonate C. Urine becomes alkaline D. H+ is secreted from blood to rental tubules E. H+ is excreted in urine KEY: A 70. Anticholinergic drugs may be used in pre anesthetic medication to: A. Slow down the gastric emptying B. Dilate the pupils C. Prevent bradycardia D. Cause bronchodilation E. Reduces peristalsis KEY: A 71. Which of the following is 1st generation antihistamine A. Promethazine B. Loratidine C. Fexofenadine D. Levocetrizine KEY: A 72. A 68-year-old women is diagnosed as a case of ovarian cancer. She begins using Cisplatin but becomes nauseous and suffers from severe vomiting. Which of the following medication would be most effective to counteract the emesis in this patient: A. Droperidal B. Dimenhydramine C. Prochlorperazine D. Ondensetron KEY: D, DOC FOR Chemotherapy induce vomiting is Ondansetron
  • 88. NEB/PMDC STEP-1 PAST PAPERS DR.ASIF ALI KHAN YANGTZE UNIVERSITY 73. A 45-year-old man was just started on therapy for hypertension and developed a persistent, dry cough. Which is most likely responsible for this side effect A. Enalapril B. Losartan C. Nifedipine D. Prazosin KEY: A 74. A 40 years old female diagnosed with short bowel syndrome in which, fat cannot be properly absorbed so long-chain fatty acid are mobilised from adipose tissue to generate energy, the following is the substrate from fatty acid oxidation A. Long chain fatty acid B. Fatty acyl carnitine C. Fatty acyl coA D. Beta-hydroxyacyl CoA E. Acetyl CoA KEY: 75. Which of the following organism is responsible of causing infective endocarditis after skin infection A. S. Aureus B. Strep. Viridians C. Haemophilus D. E.coli KEY: A 76. A registered medical practitioner attempted pregnancy of a 35 35 years old women because she did not want another child. However the woman died due to complication of the procedure. In Pakistan, the medical practitioner would be guilty of the offense of: A. Medical negligence B. Qatl-e-Khata C. Qatl-e-Shibh-Amd D. Qatl-e-Bisabab E. Isqat-e-Hamal 24. A teenager, concerned about his weight, attempts to maintain a fat-free diet for a period of several weeks. If his ability to synthesize various lipids were examined, he would be found to be most deficient in his ability to synthesize: A. cholesterol B. phospholipids C. prostaglandins D. triacylglycerol KEY: C, Prostaglandins
  • 89. NEB/PMDC STEP-1 PAST PAPERS DR.ASIF ALI KHAN YANGTZE UNIVERSITY 77. Which of the following gram negative bacilli mostly colonize healthy GI tract as non-pathogenic organisms A. Campylobacter enterocolitis B. Shigella C. Salmonella D. Escherichia colli KEY: D 78. The amino acid that undergoes oxidative deamination at a highest rate is A. Arginine B. Urea C. Asparagine D. Glutamine KEY: D 79. In hyperammonemia which one of the following is also elevated in blood A. Glutamine B. Arginine C. Urea D. Asparagine KEY: A 80. A lady gives birth to a baby with small abnormally shaped ears, mandibular hypoplasia, cleft Palate and heart defects. She gives the history of treatment for cystic acne throughout the pregnancy. Which chemical gents is responsible for the above mentioned abnormalities in the baby? A. Lead B. Valproic acid C. Lithium D. Vitamin A KEY: D 81. Addition of an epinephrine to a local anesthetic results in A. Early onset of local anesthesia B. Rapid metabolism of local anesthesia C. Prolonged duration of local anesthesia D. Increased toxicity of local anesthesia KEY: C
  • 90. NEB/PMDC STEP-1 PAST PAPERS DR.ASIF ALI KHAN YANGTZE UNIVERSITY 82. You are caring for a patient who has suffered a stroke and has issue comprehending any spoken language. Branches of what artery do you know have been compromised? A. Posterior cerebral artery B. Middle cerebral artery C. Anterior cerebral artery D. Anterior choroidal artery KEY: B 83. A 15 years old boy is brought to emergency after a fall from his bicycle x-ray reveals fracture of neck of fibula. On physical examination the patient a foot drop. Which of the following nerves is most likely injured A. Tibial B. Common fibular (peroneal) nerve C. Superficial fibular (peroneal) nerve D. Deep fibular (peroneal) nerve KEY: B 84. The middle nasal concha is part of what bone? A. Ethmoid bone B. Maxilla C. Palatine bone D. Sphenoid bone KEY: A 85. A 40 years old female developed a tumour, which was labeled as grade-2 on grading. This grading of his lesion is most likely based on: A. Cell size & mitoses B. Cell shape & mitoses C. Degree of differentiation and mitosis D. Nuclear Pleomorphism KEY: C, Staging is by size of tumor but grading is by differentiation 86. Which of the following values are the best Indexes of the preload on Heart? A. Blood volume B. Pulmonary capillary wedge pressure C. Left ventricular end-diastolic volume D. Left ventricular end-diastolic pressure KEY: C
  • 91. NEB/PMDC STEP-1 PAST PAPERS DR.ASIF ALI KHAN YANGTZE UNIVERSITY 87. While playing football a person falls on the ground because of foot abduction of the tibia on femur. Dr. Sheharyar is of the opinion that ligamentum has been injured. The most likely structure is: A. Lateral collateral ligament B. Capsule of Knee joint C. Medial collateral ligament D. Ligament patellae KEY: A 88. Antianginal drug administered by inhalation: A. Isosorbide dinitrate B. Amyl nitrite C. Glyceryl trinitrate D. … KEY: B, Amyl nitrate – Anti-anginal effects of nitrites and nitrates are usually due to decrease preload leading to decreased ventricular wall tension (decreased oxygen demand) » Isosorbide dinitrate: sub-lingual or chewed » Glyceryl trinitrate: sub-lingual 89. A 45-year-old chronic alcoholic male has been brought to medical emergency in a semi-conscious state. Blood lactate level is high. Which of the following vitamins can be used as a part of the treatment? A. Vitamin C B. Vitamin B12 C. Thiamine D. Folic acid KEY: C 90. Pantothenic acid is important for which of the following steps or pathways? A. Pyruvate carboxylase B. Fatty acid synthesis C. Gluconeogenesis D. Glycolysis KEY: B 91. Activation of the parasympathetic nervous system resulting A. Vasoconstriction B. Broncho-dilation C. Bradycardia D. Relaxation of GI tract KEY: C
  • 92. NEB/PMDC STEP-1 PAST PAPERS DR.ASIF ALI KHAN YANGTZE UNIVERSITY 92. Which of the following vitamins is an antioxidant action A. Vitamin B6 B. Vitamin E C. Vitamin D D. Cyanocobalamin KEY: B 93. Osteonic canals communicating with the medullary cavity obliquely transversely are known as A. Volkmann’s canals B. Haversian canals C. Canaculli D. Interstitial lamellae KEY: A 94. Case scenario in patient with increased bleeding time, what could be the cause of bleeding? A. Prothrombin deficiency B. Hemophilia – A C. Vitamin – K deficiency D. Von Willebrand’s disease KEY: D 95. Case scenario of Patient with bone pains having normal Ca, increase alkaline phosphatase. Most likely suffering from: A. Paget’s disease B. Hyper PTH C. Hyper Vit D D. Osteomalacia KEY: A 96. Calories intake during pregnancy and lactation A. 200-400 B. 350-600 C. 650-100 D. >1000 KEY: B? 97. A transudative fluid in edema A. Rich in proteins B. Rich in neutrophils C. Rich in albumin D. Low in proteins KEY: D
  • 93. NEB/PMDC STEP-1 PAST PAPERS DR.ASIF ALI KHAN YANGTZE UNIVERSITY 98. A 70-year-old man has difficulty rising from a seated position and straightening his back, but he has no difficulty flexing his hip and knee. Which one of the following muscles is most likely to be weak? A. Hamstrings B. Obturator internus C. Gluteus maximus D. Gluteus minimus KEY: C 99. In Severe starvation energy comes from A. Amino acid B. … C. … D. … KEY: A 100. The nerve between C5, C6 is A. C6 B. C5 C. C4 D. C2 KEY: A 101. Tall-tented T waves and widened QRS are seen in: A. Hyponatremia B. Hyperglycemia C. Hyperkalemia D. Hyperphosphatemia KEY: C 102. Regarding the CNS A. The tentorium cerebelli divided the right and left of the cerebellum B. The temporal lobe the middle cranial fossa C. … D. … 103. With regard to fetal circulation: A. Fetal umbilical vein has higher PO2 than maternal vein B. Fetal umbilical vein has higher PO2 than fetal umbilical artery C. Foetal haemoglobin has a lower affinity for O2 than which increases delivery to foetal tissues. D. The stroke volume of the left ventricle is the same as the stroke volume of the right ventricle. KEY: B, Umbilical Vein PO2 30mmHg > Umbilical Artery PO2 20mmHg
  • 94. NEB/PMDC STEP-1 PAST PAPERS DR.ASIF ALI KHAN YANGTZE UNIVERSITY 104. Loop of Henle is called as A. The countercurrent multiplier B. The countercurrent exchanger C. ….. D. ….. KEY: A 105. Which one of the following most likely causes Increase in GFR A. Constriction of afferent arteriole B. Constriction Of efferent arteriole C. Decreased angiotensin D. Dilatation of efferent arteriole KEY: B. GFR increases with increase in glomerular pressure. Afferent constriction decreases blood flow to glomeruli, thus decreasing glomerular pressure and GFR. Efferent constriction decreases blood flow from glomeruli, thus increasing glomerular pressure and GFR. 106. A taxi driver comes to the clinic with complaints of runny nose, effused eyes, fever and malaise. He wishes to continue his work while taking treatment for these symptoms. Which one of the following anti-histamines would be most suitable for him? A. Chlorphenamine B. Diphenhydramine C. Fexofenadine D. Meclizine KEY: C 107. A 42-year-old woman is evaluated for a 3-week history of vaginal discharge. The discharge is described as yellowish and malodorous and is accompanied by burning, itchy and dyspareunia. She is sexually active with a new male partner. Her last Pap smear was 2 years ago and was normal. A. Trichomonas Vaginalis » Trichomoniasis is caused by the organism called Trichomonas vaginalis and characterized by copious, malodorous, pale yellow or gray frothy discharge with vulvar itching, burning, and postcoital bleeding; it is effectively treated with a single dose of metronidazole. Treat the patient's sexual partner as well. 108. Case scenario of asthmatic patient and history of seasonal symptom’s A. IgG B. IgM C. IgE D. IgD KEY: C
  • 95. NEB/PMDC STEP-1 PAST PAPERS DR.ASIF ALI KHAN YANGTZE UNIVERSITY 109. In case of typhoid fever of 06 days. Which investigation is of first choice? A. Widal test B. Blood culture C. Typhidot test D. Urine culture KEY: B 110. Case scenario: Within one hour of the myocardial infarction, which of the following enzymes serum will be raised? A. CK-MB B. LDH C. Alkaline Phosphatase D. Troponin – T KEY: A 111. Case scenario about Increased ICP — Cushing reflex ✅ » The triad of cushing’s is Increase ICP, hypertension, Bradycardia » The Cushing reflex is a combination of hypertension and bradycardia. » Raised intra-cranial pressure (ICP) leads to reduced cerebral perfusion and cerebral ischemic. This causes a hypothalamic response leading to sympathetic activation, causing an increased cardiac output and with peripheral vasoconstriction. This leads to systemic hypertension, which helps to increase the cerebral perfusion pressure. » The increased systemic blood pressure stimulates the carotid baroreceptors, causing a reflex parasympathetic response causing the bradycardia. » The Cushing reflex is usually seen in the terminal stages of an acute head injury. It is a sign of impending brainstem herniation due to raised ICP. 112. Death certificate is written by A. Authorized medical officer B. Magistrate C. … D. … 113. A 55 year old obese male patient of type-ll diabetes mellitus was given metformin, but he became at high risk of A. Hypoglycemia B. Excessive weight gain C. Hepatotoxicity D. Lactic acidosis KEY: D
  • 96. NEB/PMDC STEP-1 PAST PAPERS DR.ASIF ALI KHAN YANGTZE UNIVERSITY 114. S2 heart sound is produced due to: A. Closure of aortic and pulmonary valves B. Opening of aortic and pulmonary valves C. Opening of tricuspid valve D. Closure of tricuspid valve KEY: A 115. A 30 year old female developed antibiotic-induced colitis due to C. difficile she was given Metronidazole to treat it but it proved ineffective. Now she can respond to which oral drug A. Erythromycin B. Doxycycline C. Vancomycin D. amoxicillin KEY: C 116. Case scenario with sarcoidosis microscopically by A. Granoma with Asteroid B. Non-caseating granuloma C. Caseating granuloma D. Macrophages and giant cells KEY: B 117. Drug for AV nodal delay is A. Propranolol B. Adenosine C. Flacinnide D. Digoxin KEY: B 118. Case scenario of Patient with polycythemia increase BP. The kidney secretes an enzyme-hormone which raises blood pressure. A. aldosterone B. renin C. angiotensinogen D. angiotensin II KEY: B
  • 97. NEB/PMDC STEP-1 PAST PAPERS DR.ASIF ALI KHAN YANGTZE UNIVERSITY 119. A 15 year old boy with Type-I Diabetes Mellitus has been admitted in emergency with Severe Ketoacidosis which needs rapid control with some appropriate antidiabetic drugs like A. Crystalline regular insulin B. NPH insulin C. Insulin Lispro D. Iultralente insulin KEY: A 120. In a chronic carrier of hepatitis B virus (HBV), which positive test is most indicative of high infectivity of viral replication? A. HbsAg B. HbcAg C. HbeAg D. Anti-HbeAg KEY: C, HBeAg is detected in the serum of persons with high virus titers and indicates high infectivity. 121. Community medicine is the system of delivery of A. Basic health care B. Selective health care C. Essential health care D. Comprehensive health care KEY: D 122. Case scenario of a chronic smoker and there is change in squamous epithelium A. Metaplasia B. Dysplasia C. Hyperplasia D. Neoplasia KEY: A, change of columnar epithelium of the bronchi into mature squamous epithelium is called metaplasia 123. Aspirin inhibits platelet aggregation by inhibiting: A. Thromboxane A2 (TXA2) B. Prostaglandins C. Endothelial cells D. Leukotrienes KEY: A
  • 98. NEB/PMDC STEP-1 PAST PAPERS DR.ASIF ALI KHAN YANGTZE UNIVERSITY 124. Mechanism of action of Penicillin is A. transpeptidase cross link B. protein synthesis inhibitors C. nucleic acid synthesis inhibitors D. act on DNA gyras KEY: A 125. Splitting of S2 during inspiration occurs due to A. Delayed closure of pulmonic valve B. Early closure of pulmonic valve C. Early closure of tricuspid valve D. Early closure of mitral valve KEY: A 126. Case scenario , infection of GI tract , bacteria or virus will first line defense against that microbial organism A. Chemotactic B. Skin C. Complement D. Opsonin KEY: B 127. Cytotoxic cells are A. B cells B. Memory cells C. T cells D. Mast cells KEY: C, T cells 128. Hormone which is responsible for the maintenance of corpus luteum A. Testosterone B. FSH and LH C. HCG D. Prolactin KEY: C 129. Exogenous steroids in adrenal insufficiency—decrease ACTH 130. cavernous sinus thrombosis and Emissary vein related mcqs 131. Accidental injury – JG cells 132. Type 2 alveolar cell - Lung maturation ✅ 133. Old and young age divide by — Median age✅ 134. BMI wala age 55 , height 172cm , weight 88kg — 30✅
  • 99. NEB/PMDC STEP-1 PAST PAPERS DR.ASIF ALI KHAN YANGTZE UNIVERSITY 135. Boxer hit a person on front of face , bleeding and smelly discharge which bone is fractured 136. Organ kept in during autopsy in — formaldehyde ✅ 137. Cytotoxic T cells 138. Patient death in in illegal delivery—criminal negligence? 139. A person trying to lose weight by no-fat diet but consuming same amount of calories as like before. Which of the following is contributing towards failure to lose weight — Carbohydrates 140. After milk intake abdominal bloating — Lactase deficiency ✅ 141. Case scenario Behavior changes, Skin rashes, diarrhea - tryptophan ✅ 142. Patient with H/O recurring attacks of pancreatitis, eruptive xanthomas and increased plasma triglycerides level (Hypertriglyceridemia) associated with chylomicrons – Lipoprotein lipase deficiency 143. Lipoprotein particles are most likely responsible for the milky appearance of the patient's plasma — chylomicron ✅ 144. Case scenario no breast — Turner syndrome ✅ 145. Severe starvation—Amino acid ✅ 146. Case scenario Previous history of fetal loss — Anti-phospholipid syndrome✅ 147. A 39-year-old woman gives birth to a term infant with a right transverse palmar crease, low-set ears, oblique palpebral fissures, and a heart murmur and mental retardation. Which of the following chromosomal abnormalities is most likely to be present in this child – Down syndrome 21 148. Glycogen synthase + by – insulin✅ 149. Maximum substance made by —phenylalanine ✅ 150. Postmortem (exhumation) — no limit ✅ 151. Oral glucose test - impaired ✅ 152. Road traffic Accident frequency — Enforcement of traffic law 153. Something about poisoning, guys pupil were constricted and pulling his earlobe dilated them for a short while — alcohol 154. Diagnosis of DM HbA1c — 7-13%✅ 155. Bradycardia results from increased vagal tone, drug should be given - atropine ✅ 156. a case was of anti-ulcer drug which act slowly but once daily given -omeprazole✅ 157. Chelation use for — heavy metal ✅ 158. A person drink poison and vomiting due to— salt ✅ 159. Thyroid swelling - pre tracheal ✅ 160. Prostatic urethra related? 161. Scenario on Sea water (ship ) — direct to kidney damage✅ 162. Oral thrush treatment — Nystatine✅ 163. Insulin side effect— hypoglycemia ✅ 164. More bleeding — incised wound✅ 165. Blast injury —lung stab wound 166. In case of stab injury from other person or poisoning — inform police ✅ 167. Multiple throat cut — suicide attempt ✅ 168. Bird foot injury —driver car accident ✅
  • 100. NEB/PMDC STEP-1 PAST PAPERS DR.ASIF ALI KHAN YANGTZE UNIVERSITY 169. Antisnake venom administration route— IV✅ 170. Patient meal after poisoning , died — fatty meal ✅ 171. Gunshot—8yard✅ 172. WHO recommended salt —2g✅ 173. Log phase 174. Eggshell appearance— silicosis✅ 175. Prevent congenital rubella prevention – vaccinate female 176. Child with lose of subcutaneous fats and muscle wasting e hx of some intake of protein and other calories — under nutrition? Marasmus 177. Night vision - cod liver oil✅ 178. Vitamin A deficiency causes- Xerophthalmia ✅ 179. Gestational DM - regular insulin ✅ 180. Ephedrine- Bronchiole 181. COPD patient Moxifloxacin use for—14 days✅ 182. Protective layer in stomach by which drug - sucralfate (gastric protective) ✅ 183. A patient present with prolong use in seizure states, he has gum hyperplasia, hairsutism, Osteomalacia etc. the following drug is responsible — Phenytoin ✅ 184. Fluoroquinolones—arthropathy ✅ 185. 186. Strong prokinatic — metoclopramide✅ 187. Haloperidol induce tremors (drug induce Parkinsonism) — procyclidine✅ 188. cromolyn sodium administration— inhalation ✅ 189. Isoniazid—peripheral neuropathy ✅ 190. Hyperuricemia is the side-effect of —pyrazinamide✅ 191. Migraine pathophysiology – vasospasm ✅ 192. Ewing sarcoma— onion skin✅ 193. WBC 2000, RBC2milion , platelet 35k - aplastic anemia ✅ 194. AB crosses placenta – IgG ✅ 195. Hydropes fetalis ✅ 196. Patient present with abdominal pain , amylase 100 time more normal — Acute pancreatitis 197. Most common tumor of parotid gland - mucoepdermoid carcinoma✅ 198. Mycoplasma associated with anemia?? ✅ 199. Compensated heart failure and Decompensated heart failure related mcqs 200. Liver injury – SGPT✅ 201. Heinz bodies – G6PD deficiency ✅ » Heinz bodies appear in G6PD deficiency and oxidative stress 202. CKMB – MI 1MCQS✅ 203. Paget disease — increase ALP ✅ 204. Case scenario APTT prolonged — Factor Vlll deficiency ✅ 205. Primary Raynaud’s phenomena
  • 101. NEB/PMDC STEP-1 PAST PAPERS DR.ASIF ALI KHAN YANGTZE UNIVERSITY 206. Pencil cells, poikylicytosis- further Ix — hb electrophoresis? 207. Detective hepatitis – hepatitis D 208. 50 year old present with central chest pain, profuse sweeting, ST elevation on ECG next step – Troponin T 209. Epidemic goiter – potassium iodine✅ 210. 10 year old traveling to Murree hills with family suddenly develop nausea and vomiting — scopolamine✅ 211. Notochord derivative — nucleus pulposus ✅ 212. Epistaxis location — septum 213. Body builder muscle atrophy —axillary?? 214. Scaleus muscle — subclavian artery 215. Child foreign body obstruction —right main bronchus ✅ 216. Boca’s area — MCA✅ 217. Patient has H/O orbital cellulitis and result was cavernous sinus thrombosis and spread of infection to brain is — ophthalmic vein 218. Left Bundle Branch is supplied by — Right coronary artery ✅ 219. Loss of arm abduction — axillary nerve✅ 220. Outstretch injury – ulnar Nerve ✅ 221. During excercise blood supply decrease—cerebral ✅ 222. Beta HCG - maintenance of corpus leutium ✅ 223. Intra vascular hemolysis — heptoglobin 224. A malformation of the hand or foot resulting in fusion of two or more digits – syndactyly✅ 225. Repolarization of membrane caused by — K ✅ 226. 100ml hb in RBC - HB index✅ 227. 3 days child dyspnea and nasal Filariasis – type 2 pneumocytes not present✅ 228. Nessel bodies - RER in neuron ✅ 229. Blood transfusion to infant – O negative 230. Resting tremors — occurs in resting state 231. Naegleria fowleri is transmitted by - swimming pool water (fresh water)✅ 232. Diphtheria transmit through - via droplet ✅ 233. Parasite rate - infant parasite rate✅ 234. Vitamin D – Osteomalacia ✅ 235. Muscle wasting and lower leg edema — Kwashiorkor ✅ 236. Treatment of acne – retinol✅ 237. hypoalbuminemia e effusion khawshioker disease✅ 238. Vitamin b12 — methylmalonic acid 239. A lady is worried about breast cancer her mother also has breast cancer in order to prevent her method should be —Self breast examination 240. Girl playing in bushes— contact dermatitis 241. Maternal infection lead to peri carditis - Rubella (German measles) 242. Further elongation of fatty acid take place in — peroxisome ✅ 243. Pulmonary larevea — Ascaris✅ 244. Case scenario on Measles — paramyxo virus ✅
  • 102. NEB/PMDC STEP-1 PAST PAPERS DR.ASIF ALI KHAN YANGTZE UNIVERSITY 245. Retroperitoneal pain back — supra renal gland✅ 246. Mild moderate severe - ordinal ✅ 247. Polio vaccine when not to give — Inner dark, outer light ✅ 248. The cases occur irregularly and haphazardly — Sporadic ✅ 249. Antenatal care – investigation for fatal anomalies ✅ 250. Contraception of choice for married with 1 child – IUCD 251. Basic health unit (BHU) in Pakistan 5400✅ 252. Case scenario previous delivery of women child died due to meningocele now she is pregnant Rx – folic acid ✅ 253. One mcqs was Brownsquard syndrome and Hemisection of spinal cord 254. Liver vitamin k dependent factor ✅ 255. lecithin–sphingomyelin ratio used to check – Lung maturity ✅ 256. Rh -ve mother with Rh + child and child is anemic so blood transfusions – O Negative ?? 257. Episodic hypertension – chromaffin cells ✅ » Episodic hypertension is a feature of Pheochromocytoma and Pheochromocytoma is a tumor that arises from chromaffin cells. 258. A person hit by medial malleolus which action damage — Planter-flexion✅ 259. Mallary bodies — alcoholic hepatitis ✅ 260. Iron deficiency anemia – ferritin 261. Thoracic duct lies in which mediastinal compartment— posterior compartment✅ 262. Atherosclerosis which artery damage — LAD 263. Hematogenous spread – ovarian CA? Sarcoma? 264. Pancrease CA spread – Common Bile Duct 265. Jaundice , pruritis, marker will be increase – ALP ✅ 266. chronic HBV replication – HbeAg 267. patient with hx of chills and fever now hb uria – G6PD deficiency ✅ 268. Hyperproteineuria ki reason—A. Bowman capsule? B. Endothelial c. injury Podocyte D. injury GBM 269. Paget disease ALP increase, nipple discharge – invasive ductal CA? 270. Allergic symptoms— mast cells activation✅ 271. Increase anion gap – metabolic acidosis✅
  • 103. NEB/PMDC STEP-1 PAST PAPERS DR.ASIF ALI KHAN YANGTZE UNIVERSITY KINDLEY IF CORRECT ANY KEY IF MISTAKENLEY PUT WRONG I WILL SHARE PAST PMDC STEP-2 AND STEP 3 OSCE QUESTION WITH COMPLETE EXPLAINATION INSHA-ALLAH SOON THIS NOTES IS AVAILABLE IN LAHORE AT STUDENTS PHOTOCOPY KHWAJA ARCADE WAHDAT ROAD MAYBE IT WILL HELP IN YOUR STEPS EXAM, this NOTES IS FREE OF CAST FOR MY ALL FRIENDS REMEMBER ME AND MY FAMILY IN YOUR PRAYERS BEST OF LUCK FOR THE STEPS EXAME DR. ASIF ALI KHAN